Thomas Calculus
Thomas Calculus
Thomas Calculus
Maron
Problems
in
Calculus
of
One Variabl
H. A. MAPOH
HHOOEPEHUHAJlbHOE
W MHTErPAJlbHOE
HCMHCJ1EHME
B nPMMEPAX H 3AiXAHAX
<DyHKUHH OJ^HOfl nEPEMEHHOM
H3/lATEJlbCTBO «H AY K A»
MOCKBA
I. A. MA RON
Problems
in Calculus
of One Variable
TRANSLATED
LEONID LEVANT
MIR PUBLISHERS
MOSCOW
UDC 517=20
Ha aneAUucKOM R3biKe
0223-200
041(01)-73
Contents
Functions 131
Limits 147
Function • • 159
§ 3.8. Solving Problems in Geometry and Physics .... 162
§ 3.9. Convexity and Concavity of a Curve. Points of
Inflection 166
§ 3.10. Asymptotes 170
§ 3.11. General Plan for Investigating Functions and Sket-
ching Graphs 174
§ 3.12. Approximate Solution of Algebraic and Transcen-
dental Equations 183
§ 3.13. Additional Problems 190
cluded.
The book is designed along the following lines. Each section
opens with a concise theoretical introduction containing the prin-
cipal definitions, theorems and formulas. Then follows a detailed
solution of one or more typical problems. Finally, problems with-
out solution which are similar to those solved but
are given,
contain certain peculiarities. Some of them are provided with hints.
Each chapter (except Chap. IV and V) closes with a separate
section of supplementary problems and questions aimed at reviewing
and extending the material of the chapter. These sections should
prove of interest to the inquiring student, and possibly also to
lecturers in selecting material for class work or seminars.
The full solutions developed in the text pursue two aims: (1)
to provide lecturers with a time-saver, since they can refer the
students to the textbook for most of the standard exercises of a
computational character and concentrate mainly on the solution
of more sophisticated problems, thus gaining time for more rewar-
10 From the Author
ding work; and (2) to meet the needs of those who are working
on their own or following correspondence courses, providing a sub-
stitute for the oral explanations given to full-time students.
The student will find the book most useful if he uses it acti-
INTRODUCTION
TO MATHEMATICAL ANALYSIS
where a and b are fixed numbers, is called an open interval (a, b).
A set of real numbers x satisfying the inequalities is a^x^b
called a closed interval [a, b]. A set of real numbers satisfying
x,
the inequalities a^x<b or a<x^.b, is called a half-open in-
terval [a, b) or (a, b]. Open, closed, and half -open intervals are
covered by a single term interval.
Any real number may be depicted as a certain point on the
coordinate axis which is called a proper point. We may also intro-
duce two more, so-called improper points, +°° and oo infinitely —
removed from the origin of coordinates in the positive and nega-
tive directions, respectively. By definition, the inequalities oo — <
<x<+ oo hold true for any real number x.
The interval {a e, a — +
s) is called the e-neighbourhood of the
number a.
The set of real numbers x > M is called the M -neighbourhood
of the improper point +00.
The set of real numbers x < M is called the M- neighbourhood
of t e improper point —-00.
The absolute value of a number x (denoted \x\) is a number
that satisfies the conditions
\x\ = —x if x < 0;
\x\ =x if x^O.
12 Ch. I . Introduction to Mathematical Analysis
is irrational.
Solution, To prove this, it is necessary to ascertain that the
given decimal fraction is not a periodic one. Indeed, there are n
zeros between the nth and (n+\)th unities, which cannot occur
in a periodic fraction.
1.1.2. Prove that any number, with zeros standing in all deci-
mal places numbered 10" and only in these places, is irrational.
1.1.3. Prove that the sum of, or the difference between, a ra-
tional number a and an irrational number P is an irrational
number.
Solution. Consider the sum of a and p. Suppose a P y is a + =
rational number, then P Y =
a is also a rational number, since
it is the difference between two rational numbers, which contra-
dicts the condition. Hence, the supposition is wrong and the number
a+ P is irrational.
1.1.4. Prove that the product a|J and the quotient a/[i of a
rational number a ^= and an irrational number P is an irrational
number.
1.1.5. (a) Find all rational values of x at which y=\/ x* + x +3
is a rational number.
Solution, (a) Suppose x and y=\/~x <iJr x Jr 3 are rational num-
bers. Then the difference y x — =q is also a rational number. Let
us now express x through q
y — x=\/ x + x-{-3 — x —
r 2
q,
Vx
2
+ x+3 = q + x,
x2 + x + 3 = q* + 2qx + x\
x ~ \-2q
•
Indeed,
(a)
x+ 1
(b) 5* + 6| = — (x — 5*+6). 2
(a) + 4x + 9) + (2x — 3) - x + 4x + 9 +
(x
2
| |
2
1 |
2x —3 |;
4 2 4 2
|
2 2
(b) \x |jc |
(b) x2
Solution, (a) This equation will hold true only for those values
of x at which sin*<0, that is why we may rewrite it in the
§ 1.2. Function. Domain of Definition 15
following way:
Substituting y for |
x |, we obtain
1.2.1.
2
Given the function / (x) =(*+ \)/(x— 1). Find f(2x), 2f(x),
fix ), [f(x))>.
Solution.
f(20=§±i; 2f W _2£±};
/<«>-£!; tfW]'-(f±{)'.
1.2.2. (a) Given the function
/(*>+/(*)=/ (t££-).
16 Ch. I. Introduction to Mathematical Analysis
On
f(x l)+ f
^1 + ^2
^
*l + *2 \ __
j Qg
1 ~f- XXX 2 _ j Qg
1 —X —X x 2
^
\l-jrX1X2J -^l ~l~ ^2 1 -\~ x i x 2~i~~ x i ~h x 2
j
1 -j- XiX 2
(1— *i) (\—x 2 ]
log
(!+*!) (l+x 2 )
(
3-^—1, —\<Cx< 0,
= J tan(*/2), 0<*<jx,
[
x/(jc
2
— 2), jt<*<6.
Find /(-l), f(ji/2), /(2ji/3), /(4) f /(6).
Solution. The point x = — 1 lies within the interval [ — 1, 0).
Hence
^ ( — = 3-1)
(
-J
> — = 1 2.
Hence
/ (ji/2) - tan (ji/4) - / (2n/3) = tan (jt/3) - 1 ;
\^3.
The points jc = 4, x = 6 belong to the interval [ji, 6]. Hence
f (
4) = 16^2 = T ;
= 36-2
= 17 '
1.2.6. The function f (x) is defined over the whole number scale
by the following law:
2x 3 + 1. if jc<2,
f(x)-{ l/(*-2), if 2<*<3,
2* — 5, if x> 3.
§ 1.2. Function. Domain of Definition 17
If x> V2 y
then
Thus, Fig. 1
S(*) = 0<x</2",
-x \\x\f2 —4,
2
V2< x^2\/2.
Since |/T/2 < K2~ = {V~2 j2) = V Since
, S (V2/2) 2
2 . 2 > J/
2~,
1.2.8. Bring the number a„, which is equal to the nth decimal
place expansion of V~2 into a decimal fraction, into cor-
in the
respondence with each natural number n. This gives us a certain
function an = tp (ri). Calculate q)(l), cp (2), q)(3), q)(4).
Solution. Extracting a square root, we find \f2 =1.4142... .
Hence
q>(l) = 4; cp(2)-l; q>(3) = 4; cp (4) = 2.
f (x) = 49/x + x at the points for which 7/x + x=3.
2 2
1.2.9. Calculate
Solution. f(x) = 49/x + x = (7/x + x) —\4,but 7/x + x = 3, hence
2 2 2
/ (a') =9 —
14 = — 5.
1.2.10. Find a function of the form / (x)
= ax' 1
-\- bx -f c, if it is
f(—l)= I0^a — b + c,
/(l)=6 = a + 6 + c.
18 Ch. I. Introduction to Mathematical Analysis
cp (jc) =x 2
and ^(x) = 2x .
Solution.
i|)[<p(*)] = 2* < * ,
= 2*".
1.2.13. Given the function
5x2+1
f(x)~ 2—x '
3
Find f(3x); f (* ); 3f (x); [f (x)]\
1.2.14. Let
(3*
at — < x<1 0,
4 at 0<*<1,
3x— 1 at l<x<3.
Find f(2), /(0), /(0.5), /(-0.5), /(3).
/(*+l) =* a
— +2 3jc .
/ (
X) = x + l/x 2 2
and (p (x) = x* + 1 /*
4
(e) log^^
(0 f(x) = \og x 5\
— 5x-\-6 x'2
(g) /W = log- -4x + 6 v
(j) /W = logcosx;
3
(k)/(x)^arccos
4 + 2 sin;c ;
(i) y
log^^->0.
This inequality will be satisfied if
-5^1>1, or x 2 — 5x + 4<0.
Solving the latter inequality, we find 1^a:^4. Thus, the seg-
ment [1,4] the domain of definition of the function.
is
(f) The function is defined for all positive x different from unity,
-1 ^ 4+2 A ,
3
.
sin x
<
^ 1.
4 + 2 sin x ^
Whence
3<4+-2sinx, i.e. sinx>-— 7 2
.
(C) / W
= l + 2 a-in, + _J_.
(d) f (x) = log|4-x 2
|;
f
( ) &
= 2- cos 3* ;
(g) y=j^ •
2x
< 1.
-1
< cos3x< 1, we have — —-< 1,
x - Ty
'
-\-x = 0.
2
whence x
Thus, the left member of the equation attains real values only
at x =0 and # 2
t
=— 1. By a directcheck we ascertain that they
are the roots of the given equation.
This problem shows that a study of domains of definition of a
function facilitates the solution of equations, inequalities, etc.
1.2.24. Find the domains of definition of the following functions:
, 2x— 3
(a)
x
y= r « _ ;
1.2.25. The function f (x) is defined on the interval [0, 1]. What
are the domains of definition of the following functions:
(a) /(3r>); (b) f(x-5); (c) f(tan*)?
2
function f(3x )j=f(u) is defined if i.e. 2
<l, 0<w<l, 0<3x
whence — 1/]/3<*<; l/|/3.
(c) Similarly: O^tanx^l, whence
fcrt< + (k = 0, +1, ±2, ...).
1.2.26. The function f (x) is defined on the interval [0, 1]. What
are the domains of definition of the functions
(a) f (sin*); (b) f(2x + 3)?
§ 1.3. Investigation of Functions
At a >
the function has no maximum value.
Similarly, at a <
the function f (x) will increase in the inter-
y = 3x + 5x — 1.
2
(b) Find the rectangle with the maximum area from among all
rectangles of a given perimeter.
Solution, (a) Apply the results of Problem 1.3.1: a — 3 0, b=5,
>
c =— 1. The minimum value is attained by the function at the
point x = — 5/6
4ac — b* 37
4a 12'
= (*2 — *l) (A + X X + X\ + 3) = 1 2
Since x 2 xx — >
and the expression in the brackets is positive
at allx 1 and jc 2 then / (x 2 )
,
f (xj 0, —
i.e. / (* 2 ) >f(Xj), which>
means that the function f (x) increases for all values of x.
1.3.4. Find the intervals of increase and decrease for the follo-
wing functions:
(a) / (x) = sin #-|-cos x;
(b) tan(*+n/3).
Solution, (a) Using the familiar trigonometric formulas, we find
f (x)
— ]/^ 2 cos (x— ji/4).
It is known that the function cos a: decreases in the intervals
2nn ^.x^(2n+\) n
and increases in the intervals
j (x)
= a cos x-\-b sin x (u b 2 > 0).
1
-J-
§ 1.3. Investigation of Functions 25
f (x) = Vcl 1
-V b cos (x
2
— a),
where cos a = a/)/ a + b 2 2
, sin a = 6/ Ka + b'\ 2
Since |cos(x — a)|^ 1,
cos(a: — a)-=l),
r
the maximum value of f (x) equals +\/ a 2
+ t)
z
(at
(p(*) = (*
a
— 2)
3
+ 8.
f (x) = ( x - ai )
2
+ (x-a + 2)
2
. . .
+ (x-a n f
takes on the minimum value.
Solution. Rewrite the function f (x) in the following way:
f(x) = nx 2
— 2(a, + a + 2
. . . +a n )x+(a +al+ 2
l
. .
x= (aj +a + 2
. . . +a n )/n.
Thus, the sum of the squares of deviations of the value of x
from n given numbers attains the minimum value when x is the
mean arithmetic value for these numbers.
1.3.9. Which of the given functions is (are) even, odd; and
which of them is (are) neither even, nor odd?
26 Ch. I. Introduction to Mathematical Analysis
f(x) = 2x — x+l;
s
(c)
(d) /W = *S^T-
Solution, (a) It can be seen that /(+ x) + f (— x) ^=0. Indeed,
= log ( 1 +* 2 -- x 2 )=0,
hence, / (x) = — / — *) for all ( a:, which means that the function is odd.
v =—
1
(b) m_ ^ = iog = log \+x)
io 6
g
1
1+jc'
then the function / (ax + b), where a > 0, is periodic with period T/a.
Solution. Firstly,
= f(ax' + b + aT = f(x + aT l) l ).
Note. The periodic function (x) = A sin (ayx + where 9 / cp), /I, co,
and initial phase (p. Since the function sin* has a period 2k, the
function A sin(o)A: + (p) has a period T 2n/(s>. =
1.3.12. Indicate the amplitude \A\, frequency co, initial phase (p
(a) / (a;)
= tan 2x;
(b) f(x)=cot(*/2);
(c) \(x)^s\x\ 2nx.
Solution, (a) Since the function tanx has a period Jt, the function
tan 2x has a period n/2.
= 1— y sin 2
2x = 1 — cos 4x) = -| + -j-sin ^4x + y) ;
2
f(x)= r = .
1.3.17. Which of the following functions are even, and which are
odd:
3
(a) /W=^(l-x) + /(l+x) 2 2
;
28 Ch. I. Introduction to Mathematical Analysis
called inverse with respect to the given function y f(x). The suf- =
ficient condition for the existence of an inverse function is a strict
monotony of the original function y f{x). If the function increases =
(decreases), then the inverse function also increases (decreases).
The graph of the inverse function x g(y) coincides with that of =
the function y =
f(x) if the independent variable is marked off along
the (/-axis. If the independent variable is laid off along the x-axis,
i.e. if the inverse function is written in the form y = g(x) then y
since ]fx
2
+ >
1 \x\ y
and is odd [see Problem 1.3.9 (a)]. It increases
§ 1.4. Inverse Functions 29
y=\og a (x + Vlfi+l)
with respect to x, we find
a? = x + Vx + 2
l; a-y= — x+Vx + 2
1,
whence
x = y (a^ — = sinh (*/ lna).
x*-x + {l-y) = 0.
Solving the equation with respect to x, we obtain
not fulfilled.
Divide the x-axis into intervals tin n/2. If n is — n/2^x^nn +
even, then the function increases on the intervals tin — n/2^x^
^nn + n/2; if n is odd, the function decreases on the intervals
—
nn ji/2^ x^nn +
n/2. Hence, on each of the indicated intervals
there exists an inverse function defined on the interval [—1, 1].
30 Ch. I. Introduction to Mathematical Analysis
xix '
y=2
1}
(d) .
(*>)
f(*)=TT#>
(c) f(x) = s\n x — 2sinx; 2
f( X = X
(f)
°*x.x ' l
)
-3 "2
I
-1
1
/
/
12
i i
3'
i
-/
Fig. 3
fl,)
(l+*S)0 + *i)
^ 2 (x — s 2 t ) (1— x^)
(l+*I)(l+*f)
and f(x 2 ) >f(x l ).
Similarly, we can show that on the interval (1, oo) the function
decreases. Finally,
f (*)
= (1— sin*) 2
— 1,
f(x)= l
*. 0<x< n ,
\ 2n — x, n^x^ 2jt.
X
-2% -% Tt 2lt
Fig. 5
Fig. 6
—2 at x>0,
(b)0= 1/2 at x= 0,
—x 3
at x < 0; Fig. 7
9,
2
1/2
{
1
-1 O 1
.1 1— -1
12
I i
3 4
-2
-i
Fig. 8 Fig. 9
i/ =2
on the interval (0, 1] and a part of the branch of the hyper-
bola y=\l{x—\)
on the interval (1,4] (see Fig. 8).
(b) The graph of the function consists of a portion of a cubic
parabola, an isolated point and a half-line (see Fig. 9).
(c) The function may be given by two formulas:
I 2*. if x>0,
y==
\ 0, if x<0.
34 Ch. I. Introduction to Mathematical Analysis
Thus, the graph of our function is a polygonal line (see Fig. 10).
(d) From (c) it follows that the function defined only in the is
Fig. 11
X —
N
x
-7C Sic JL v % s'3n 2% Jax
' 2 2 ~T~ ~7T
Fig. 12
the points where cosx<0, we can sketch the desired graph (the
solid line in the same figure).
(b) The function + 2|x may be
|x given by two formulas:
=/ +
(x 2)x at — 2,
# \ —(x+2)x at — 2.
and y = [(*— l)
2
+1], retain —
only the parts corresponding to
the above indicated intervals. Drawn in a solid line in Fig. 13 is
the graph of the given function, the broken line showing the de-
leted parts of the constructed parabolas.
§ 1.5. Graphical Representation of Functions 35
y = 2\x—2\ — x+ \
l\ + x.
Solution. At x ^2
y= 2(x—2) — (x + l) + x = 2x— 5.
At — l<x<2
y== — 2(x — 2) — (x+ + x = — \) 2x + 3.
19
\ 5
i /
-1
— \\V//'
1
'2/
1
3
>
-1
Fig. 13 Fig. 14
Finally, at x<.~ 1
Fig. 16
y x
=x
and y = x —4. 2
2
Fig. 17 Fig. 18
l
incre-
ases. The maximum value y 1 =
n is attained at *--=— 1. Accor-
dingly, the minimum value of the function is 1/jx. The solid line
in Fig. 18 represents the general outline of the graph.
u>-0 / vr ,o yuf(x)
y=f(x)+l,l<0
Fig. 20
Fig. 21 Fig. 22
of the function y =
f(x) lying above the x-axis remains unchanged,
its other portion located below the x-axis being transformed sym-
metrically about the x-axis (see Fig. 23).
VIII. The graphs of the more complicated functions
y^'kf (kx+a) +b
are drawn from the graph of y = f(x) ap-
plying consecutively transformations I to V.
Fig. 24
By shifting the obtained graph 2.5 scale units leftward and then
0.8 u nit downwar d draw the desired graph of the function
y=3V— 2(jc + 2.5)— 0.8 (see Fig. 24, d).
= 2 K3 cos
#= 2VTcos (x + ji/6),
which is the graph of the function */ 2 K 3cosjc translated by ji/6 =
leftward. The function has a period of 2ji, hence it is sufficient to
draw its graph for — n ^Zx ^ ji
ZVT
\
// \ \
/ /
1
/' \ \
1
// \\ 1
%
> •X
/ fa/2 f\\7t/2
I l
\\
>^ -2V3
Fig. 25 Fig. 26
(a) y—^i
< b> f-JTZ^
(c) ^ x2 + a:+ 1, if— l<x<0,
s sin 2 x if < x < k,
(x-l)/(x+l), if ft <x^ 5;
(d) = x+ \/x;
y
(e) y = x —x 2 3
;
(f) = sin
(g) y = 1/cos*;
(h) j/=3s in (2jc — 4);
(i) t/=2K — 3(x+ 1.5)— 1.2;
§ 1.6. Number Sequences. Limit of a Sequence 41
n -+ co
N(e)>0 such that the inequality |jt„ a\<& holds true for all
n> Af(e).
A sequence which has a finite limit is said to be convergent.
A sequence {x n } is called infinitely small if lim.*^ 0, and infi- —
nitely large if lim^^oo.
1.6.1. Given the general term of the sequence {xn }:
sin (nn/2)
1
~~
3 3
sin (4it/2) ~
: "" U;
4
sin(5jt/2) _
~~
1
*
5 5
42 Ch. I. Introduction to Mathematical Analysis
1.6.2. Knowing the first several terms of the sequence, write one
of the possible expressions for the general term:
(3 '
I' 8 »
13 »
18 '
23
;
thus we have
_ n* + l
*»~~5/i —2 '
(b) Here the general term of the sequence can be written with
the aid of two formulas: one for the terms standing in odd places,
the other for those in even places:
j k at n = 2k—l 9
xn =\ 1/(6+1) at n = 2k.
^=^[i-(-i)i+^- [i+(-i)"]- 2
1.6.3. Find the first several terms of the sequence if the general
term is given by one of the following formulas:
(a) jc„ = sin (nn/3)\
(b) x n = 2~ n cos nn\
(c) x n =(l +
l/n)\
Solution, (a) For any e > let us try to find a natural number
Af(e) such that for any natural number n>N(z) the inequality
\x n — l|<e is fulfilled.
§ 1.6. Number Sequences. Limit of a Sequence 43
For this purpose let us find the absolute value of the difference
2/1—1
-1
2/1+1 2/i+ 1
I
2n+
Thus, the inequality |
jc„ — <e 1 1
is satisfied if
2
^qrf <8 » whence
n > 1/e— V 2 - Hence the integral part of the number 1/e — V, may
be taken as jV(e), i.e. W = £(1/8 — 7,).
So, for each e > we can find a number A/ such that from the
inequality n >N it will follow that \x n 1 e, which means that — < 1
lim f^ = l.
5(5/i 2 -l)
is fulfilled.
Solving this inequality, we find
_ 8 . 1 .1 t/8 + 5b
Putting
N = £ (i/?±5),
we conclude that at n> N
K-3/5|<e,
which completes the proof.
If e = 0.01, then
*-*(•{- -5.
and all terras of the sequence, beginning with the 6th, are contai-
ned in the interval (3/5—0.01; 3/5 0.01). +
1.6.5. Given a sequence with the general term x n = 9n _^ 4 . It is
known that lim x„= 1/3. Find the number of points xn lying out-
side the open interval
44 Ch. I. Introduction to Mathematical Analysis
X»
1 19 _ 19
3 3(9/z + 4) 3(9/z + 4)
Outside the interval L there will appear those terms of the se-
quence for which this distance exceeds 0.001, i.e.
19
3(9/z + 4) ^ 1 000
whence
18988 ?n 7
1 <; n <
Hence, 703 points (x lt x2 , .
,
x 103 ) are found outside the inter-
val L.
2n 2 —9 |2/2
2 —9 |
At n ^
3 the absolute value of the difference remains greater
than the constant number 1 /2 hence, there exists such e > 0, say,
;
e= 1
/ 29 that the inequality
2ril —9 — >
1
1,
112
^
1
1
J_ _L i. _L L
2 '
2 '
4 '
4 '
8
I
l
-^rm if n is odd *
*»=f — i
j77
n/2
if n
.
is even,
{ 2
has no limit.
xn = a n /nl
Solution. Let a natural number k > 2a. Then at n >k
an a \ a a
a a a f a a ( a
^x
n\~ \ 2 n \ 1 2 k ) \k+l k +2 n
<a.(i)--W.(^"
Since lim(l/2)" = (prove it!), then at a sufficiently large n we
have: (4-) <—— and, hence, a n
/n\ < e, which means that
V 2 J (2a) k
Y\m(a n /n\) = 0.
1.6.12. Test the following sequences for limits:
(a) x„=1/(2jx);
i 1 for an even n,
(b) xn — <j
^
(C) Xn = ~ C0S nn
1
~2" I
(d) ^„ = ai [!-(-!)»].
46 Ch. I. Introduction to Mathematical Analysis
x n =l/n* (k>0)
is an infinitely small sequence.
Solution. To prove that the sequence xn is infinitely small is to
pro\e that = 0.
lim x n
n -* oo
(a) xn = ,
(b) xn = ± sin [(2/i - 1)
I
JC l-
»l""
2
^
^
2
3/-
^ 3/-
2
- 1
3/-'
l
c 3/ .
f
-
\
X n\ = 3/- < TO •
Solution, (a) x n
. .
=3 "s^
— — 3+ ,
,
3
.
is
as the
Solution. Let us prove that the variable
sum l+a„, where a n is an infinitesimal as n oo.
Yn can be represented
—
Let us put \/n= 1 + a n Raising to the nth power we obtain .
n>l + njn^l) a%
(since all the terms on the right are non-negative). Transposing the
unity to the left and reducing the inequality by n 1 we obtain —
whence
r
it follows that 2/n >a n
2
or > a n > 0. Since
lim V 2/n = i
lim an also equals zero, i. e. an is an infinitesimal.
n -* oo n oo
lim yn=l.
n -* oo
z V n
> AJ.
Taking the logarithm, we obtain
infinitely large.
^
W xvn~
3n 2 +5n
2 +n
+4
2
,
'
W n ~
5b» + 2«»-3/i + 7
4n 3 -2n+ll
.
'
— + +
4n 2 -4n 3 l»+2»+ ...+««
+ 4'
2/i3 3/i
.
w n_ 5/i3 + n+l
.
(e) *n =
1+2
+r + " -
3+A+l
n
Solution, (a) ~ 2_
S+
n*
1
lim x = — lim (3 +
7-o
5//z + 4//z
x
2
— -3.)
Hence
_ + ^ A-f-l
2
/i(/i+1)(2/i+1)_ 2n 3 + 3n 2 + Ai n^ n 2
lim #„ = 1/15.
/ 3n + n— \32 2n 3 + 2n +l \« 2
W
, ,
— ^4„2 + 2n + 7j
.
' W /
U" + 7 " + 3n + 4j
3 2 •
«i « U« + 2n+7j U« + 2« + 7j U/t + + 7j 2 2 2
2/l
We have
lim xn = lim i/5n = lim lim \/n,
n co n -+ <x> n -* co n -* co
but from (1) it follows that limv^5=l and lim y^/i = 1 ; hence
lim xn = 1 • 1 = 1
1.7.3. Find
' 2ft 3 .
1— 5ft 2 \
xn—
2ft
3 — 13ft 2 +3
i Oft 3 + 2ft 2 + +3
15ft
*
Whence
_ 2ft 3 — 13ft 2 +3 _ 1
lim x n - r
iirn^
10n3+ 2 n 2 + i5 n+3- 5
•
n -+ oo
Note. If we put
_ 3
_ 1— 5ft 2
^ — 5ft+
2ft
— 2ft
2
+3 ;
1
'
then the limit of their sum ) though each of the \im{y n + z n = 1/5,
summands an infinitely large quantity. Thus, from the conver-
is
n
(b) xn = Vn + n+ —]/ n — n+ 1;2
\
2
(d) xn =¥n — n + n\ 2 3
(ej xn _
— ]Tn*+\ + Vn— ,
.
4 /
(h) xn = ± + ± + ±+...+.
1-2 1
2-3 1
3-4 1
' ' * 1
ft(ft+l)
50 Ch. I. Introduction to Mathematical Analysis
(c) Xn
_ n*(n-VlF+\ _ ) -n* _
1
n+K/iHl
=—n 1
— — oo as n — oo.
(d) xn = ~ . =
(n 2 —n 3 2/3
)
— n y n — n* + n* *
It means, x n — >•
1/3.
Factoring out the terms of the highest power in the numera-
(e)
tor and denominator, we have:
1/4 >
+ oo as n — oo.
(c) xn =Y — n l
3
+n; (d) *„ = -^ cos n3 — ;
(f) xn = j j j- .
1+T+T+-+*
§ 1.8. Testing Sequences for Convergence
Bolzano-Weierstrass* theorem. A monotonic bounded sequence has
a finite limit.
Theorem on passing to the limit in inequalities. If xn ^y ^z n n
and liin x^lim zn = c, then \imy n c too (c = is a number,
n -y cc n -> cc n -* cc
e>n
2
+ 5n+\ > 6m + 5n — 4. 2
Xn ~ n\
'
Xn+1 ~~
_ 10"+ 1
_ 10" 10 __ 10
(n+ 1)! n\
'
/i+ 1 n+ 1
'
( a ) Xn= ^j'>
#„ = (-l) ~sinft;
n
(b)
5+1
1
1
52
11
+l ^5 3 +l 1
' • • 1
i
5»H-1
^i.e. x1 — b+{ ;
x2
1,1
~ 5+1 + 5 +1 a ;
x2 — 5+1+ 1,1,1 5H-1 ^ 5a +l '
converges.
Solution. The sequence \x n ) increases, since x n+1 xn 1/(5"
4-1
= + + 1)
and, hence, #„ +1 >
x n Besides, it is bounded above, since 1/(5"+
. 1) <
< 1/5" at any n and
+ _L_ + _^
^3 +l +
+-J—
1
" 5+1 I
1
5 +l 5*22 i 1 I
1
3 l
1 I1 '
'
'
' '
' I
1
Kn
5»+l ^ _L_ 1
iii 1/5-1/5" + * _ W,
^ ^ili^
/ 1 1\ 1
(a) xn = —r- ;
(b) * B = 2 + l + l+...+I.
1.8.7. Prove that the following sequences converge and find their
limits:
n radicals
2n
(b) *„
(/i + 2)! '
(c) x
We have = V 2 + x n _ n = 2 3,
xn Sinc e x = V~2<2 l9 y
. . .
1
ft = 2; y2 =— 1-
The negative root does not suit here, since x„>0. Hence, lim xn =y l
= 2.
n -* oo
sequences j#
— -^-j and {y} converge, their limit being y, that is
why lim x n =
(fl-f-l)! _ n\ _n\ nn __ nn
Xn+1 ~~
(n+\) n Xn
' ~~~ '
~~nP (n+l)»
Since
(n+\)*
< l
>
X» + 1 <X »*
nn \ n J \ n J n
54 Ch. I. Introduction to Mathematical Analysis
Hence,
^n
< -y and xn+l <-^x n .
Passing over to the limit,
we obtain
/ = 0.
1.8.8. Find the limits of the sequences with the following gene-
ral terms:
n n
zn = -
1 1 1
r
-
*-...+
V n*+\ ]fn*+2 Vn* + n
Solution. Let us prove that lim#„=l. Indeed,
n -* oo
= n— Vn + n 2
l*»-l| 1
}Tn? +n
V n 2 + n(n+ V n 2 + n) 2n'
Then,
+ = 2M .
l
=x ri
lim yn =\.
n -* qo
x1 = Va\ xa =----j/ a + V a\
r
xs = V<* + V a + Va; xn --= ]f a + Va+ . . .
+ Va
n radicals
(a>0)
has the limit b = {VAa+\ + l)/2.
_ 1 1 l_
*n
~3+l + 3 +2 + 2 ' ' ' ^~?> n
+n
has a finite limit.
A= lim f (x), if for any e > there exists a number 8(e) > such
x -+ a
that for all x satisfying the inequality and belon- 0<\x — a|<8
ging to the domain of definition of the function f (x) the inequality
\f(x) — A\<s holds true (the "e-8 definition").
If a=+oo, the definition is as follows. A number A is called
the limit of a function f (x) as x «>, A lim f (x), if for any —+ =
X -* + 00
e >there exists a number M(e) such that for all x satisfying >
the inequality M(e) and belonging to the domain of definition
of the function f (x) the inequality \f(x) A\ holds true (the — <e
"e-M definition").
The notation lim / (x) = 00 means that lim | f (x) \
= + 00. The
x -+ a x -* a
rest of the cases are considered similarly.
The definition of the limit of a function after Heine. The nota-
tion limf(x) = A means that for any sequence of values of x con-
x -+ a
verging to the number a
-^2> • • •
•^l> • • * •
the following two conditions: (1) the numbers x lt x2y ... belong
to the domain of definition of the function f (x) =(3x+ l)/(5* 4) +
(i.e. x n =£ —
4/5); (2) the sequence {x n } converges to the number 2,
i.e. lim x n 2. =
n -+ 00
5*1 +4 ;
5* 2 +4 » ' ' '
0^l) a j±i a l
l
„T.'
im fi xW )= Hm
3*„+l_ lim
ii.5^ + 4- lim (5*„ + 4) 10 + 4- 2
•
§ 1.9. The Limit of a Fund ion 57
5^+4= 2
•
xn = 1 H1
nn
1
and xn = 1
—j-t—
+ (4n+l)ji
Ta
2
(n
v
= 1 ,
»
2,
»
. .
. ),
/»
for which
lim x n = lim a:^ = 1.
n -+ <x> n -* oo
)
l l
and
f «) - sin + g/[(4w ^
{ t) n] _ = sin f£+i = sin
{
ji
(
2/m +
f) = 1
Hence,
lim / (x n ) = and lim f (x'n )
= 1
n
lim
-* cp
sin<= lim sin (2jw
n -* 00
+ ji/2) = 1,
Note. The above examples show that one cannot draw the con-
clusion about the existence of the limit of a function proceeding
from the sequence of values of x of a particular form (for example,
proceeding from x n 1 +2/((4n+ 1) n) =
in the item (a) of this prob-
lem), but it is necessary to consider an arbitrary sequence x 19
x 2) ...,#„, ... having a given limit.
The latter inequality shows that the required inequality /(jt)-f5 |<e
is fulfilled as soon as \x —
1 < e/3 = 6. Hence, lim (3x 8) = 5.
1
|
— —
According to the "e-M" definition of
(b) the limit one has to
show that for any e > it is possible to find a number M>
such that for all x > the inequality M
will be fulfilled.
Transforming this inequality, we obtain
5* -j-l 5 14
3x +9 3 |3* + 9| <8.
Since x > 0, it remains to solve the inequality
§ 1.9. The Limit of a Function 59
whence
^ 14—9e
14 7" 98
hence M= 3e
.
values of x >M the inequality (*) is fulfilled, and this means that
(c) We have to prove that for any K> there exists 8 >
such that from the inequality
|x-l|<8
there always follows the inequality
1 1
I
K, (**)
(l-*) 2
whence
|l_x|<-L: (K>0).
(d) We have to prove that for any there exists K> M>
such that from the inequality x there always follows the ine- >M
quality \og a x>K.Let us choose an arbitrary number and K>
consider the inequality \og a K. If we put a K x>
M, then at =
x>M the inequality \oga holds true. Hence, x>K
lim log a *= +oo.
X -»» + 0O
1.9.5. Using the sequences of the roots of the equations sin (\/x)=l
and sin (l/x)= —
1, show that the function / (x) = sin (l/x) has no
limit as x >0.—
60 Ch. I . Introduction to Mathematical Analysis
(g) lim —= 0.
I. If the limits \\mu{x) and limy (a;) exist, then the following
x -» a x-+a
theorems hold true:
^
x^ a a a
x -* x -*
lim U (x)
II. For all main elementary functions at any point of their do-
main of definition the equality lim / (x) = / (lim x) = f (a) holds true.
x -+ a x-+a
III. If for all values of a; in a certain neighbourhood of a point a
(except for, perhaps, x a) the functions f(x) and q) (x) are equal =
and one of them has a limit as x approaches a, then the other one
has the same limit.
IV. The following limits are frequently used:
(1) hm
x-+0
— x
=1;
(2) lim(l
x -* <x>
+ l/jc)* =alim(l +a) '* = e = 2. 71828.
-*
l
.
.;
(3) lim
lo ^ il+x) = log a e (a>0;a^l);
x^Q x
/+ =
... ,. In (1 a:) «
(4) hm x
'
1;
x^O
i \ i; +
W I™m 3*.+-f*r+T
ixb 9x 7 /u\ r1 xs + 3x* — 9x—2
'
(b
>
I
™ *3_ x _ 6 ••
m P and q
(C)
^VeZU^' (d)
il
,^_J
integers);
(e) Hm ^+^- 3
;
(f)lim^^;
|/*Fp7 —3 J/" 2* —3 .
.
r f\ x —3
(g)
*-+22 r/ — 2o 3/0=
3/-Xft
j/Ff6-2j/3^5
x+6 r/ 3x— 5
5 '
(h)
*-3 L
*-*3|_
l0g «
VHT+6-3J'
> — x — a:+
2 1
r r
v + K8a:+ 1
Solution, (a) Since there exist limits of the numerator and deno-
minator and the limit of the denominator is different from zero,
we can use the theorem on the limit of a quotient:
+ 9* + 7 ^ ^i
lim V(4* 5 t 9x+7)
+ t ;
+9+7
lim 4*» J
4 ,
(b) The above theorem cannot be directly used here, since the
limit of the denominator equals zero as x—-»2. Here the limit of
the numerator also equals zero as x —»2. Hence, we have the
indeterminate form -jj- . For ^^2 we have
x — x— 6
3 ~~ (x—2) (x + 2x + 3) --jc + 2*+3" 2 a
Thus, in any domain which does not contain the point x =2 the
functions
r, x £3 + 3x — 9x— 2
2 , , . x2 + 5x+l
are equal; hence, their limits are also equal. The limit of the
function y(x) is found directly:
hm /vi- * + 5x+ = 15
cp(^)^ im
2
1
hence,
£/ x
x3 + 3x — 9x— 2 2 15
X-+2 X-+2 X X O 11
62 Ch. I. Introduction to Mathematical Analysis
— 4 — 3x + 2
X3 X2
(a) lim 2
( 3a:
(b)
*
lim
-* + 00
(j/9je 2 + — 3x); l
(c) lim
2^+3^/7+5^/7
v^ + oo V 3*— 2+ 3/2x—
(d) lim 0/2x 2 — 3— 5x);
* -* — 00
/f , ,. V2x^f3 , ]/~2x 2 +3
X -* OO
lim
jc^oo
f^_^)=lim
—
\3* 2+ 4 3x 2 y ^ 9x
3
+
2x3+4x2
6a: 2 — \2x— 8
-
2 + 4/x
= lim 9 —
2
'
/ux ,.
—
(}^9x 2
+i — 3x) = ,. 1
n
= 0.
(b) lim -J-
'
lim r
+ 1
x -+ + cc J^-j- 1+3*
(c) In handling such examples bear in mind that the function
f{x)= \/p n where p n (x) is a polynomial ofjdegree n, tending
(x),
n
to infinity in the same way as the function "j/ This allows us .
to single out the superior power of x and divide both the nume-
rator and denominator by this power of x. In the given example
§ 1. 10. Calculation of Limits of Functions 63
Urn
- cc
(1/2j?^3— 5*)= lim [V2x* — 3 + (—5*)] -
X -> X-*-oo
, im
y* a (2 + 3/**) = Hm ^2 + 3/^ _ ]/"2~
not exist.
lim 2x/(* + 3)
* -*• oo
teger);
,
x ,. sin (x— ji/6)
^3 — 2cosx' U *-jt/2 sin*) 2 '
, v
y x—
2 sin 2 x-f sin 1
*i™/6 2sin
2
x— 3sinx+l '
lim 3/ ^Z_
* 1; hence
2
= lim ^ - lim ^-3)(^+3*+9) _
= lim2(z + 3z + 9) = 54.
2
2^3
64 Ch. I. Introduction to Mathematical Analysis
lim
^T^~ l
-i; m _£z± *
(S ee Problem 1.10.1 (d)).
X-y 2 -* 1
lim
t-jT/6 l/"3
y^U
— lim—— 2 cos a: 2-0 /"3
sin2
2 cos (z -f ji/6)
sin z 2 sin (z/2) cos (2/2)
lim
—
= lim
>-o J^~3 "K" 3 cos 2+ sin 2 2 - o 2
/" 3 sin 2 (z/2) + 2 sin (z/2) cos (z/2)
CQS(2/2)
= lim _ =1.
z - o yA 3 sin (2/2) + cos (z/2)
1.10.4. Find the limits:
,
(a)
x ,.
hm
x^O
—— —
1 cos x
-5
x
;
/L v
(b) lim
x-0
i
• tan x
X3
— sin a:
cos^2)
(c) lim
a:- 1
f
Solution, (a) lim
/vi-
*-»o
1 — cos
x5
—= a:
lim
*-»o
2 sin 2 (x/2)
x r^'-
l Hm
2 J™
/
^
sin(^/2)
x/2
y
J
==1
2
,
'
/ux ,. tan a: —
- sinA; = lim sin a: (1 —cos a:)
—
(b) lim '
x2
Let
(c) us put 1 x= z. Then x=l — z and z —O as x — 1.
Hence,
/ Jl TC \
lim
1— X
—
COS -y X
- — = lim
2-0
= lim
2-
sin -7- z
2 3T
(c
^ .fe)* ra
In (\+x)
;
(d) lim(l+£/A:) m *;
e4 *— 1 .
(e) lim t
(f) lim
x-+ o
3*— 1
1
*-* tan a:
*
In (a+x) — In a
(h) lim
t
(g)
x ->
... In x — 1
(0 llm -r=r •
§ 1.10. Calculation of Limits of Functions 65
lim 1 + = e'\
In +x)
(e) lim
u" 1
= lim (1
3* — 1
1
ln3'
x -* x -v
1.10.6. Find
•im (1+^
x
Solution, lim (l+l) =lim |^1 +1^*] = 1.
1
.
But at finite limits lim / (x) = A > 0, lim q) (.*:) =5 the following
lim [/
{x) = ex ~+ a
=e B\nA == j[B t
Hence,
°- r ""'"-"-(!)" !
lim =/!
{x)
/Vo/e. If in handling examples of the form lim [/
(x)]v it turns
x-+a
out that Vunf(x)=l and lim cp (jc) = oo, then the following
3— 31 43
66 Ch. I. Introduction to Mathematical Analysis
l«»n(p(x)L/U)-l]
= lim {
[ 1 + (/ (x) — 1 )]
<*> 1
>}* <*> U <*>
- n = e**a . (*)
(a)
(c)
lim(
lim (1
^r
2
+sin :rix)
+3
cot
;
™;
(b) li.n(4±^)
,/,m
*;
/U Q)
(d) lim (a¥=kn> with & an integer).
(f|^~y
^^StT' <P(*) = 8* + 3; a
lim/(x) = lim§^=l;
lim q) (jc) = lim (8a:
2
-f 3) = oo.
Use the formula (*):
8a: 2 +3 ,im V (*> ^ (•*>-!]
lim (ttt—)
/W— I=|S±4 — 1
:
[/(*)-!] = - lim = - 8.
2
gf +
3)
lira q> (x)
Therefore
1;
/2xH-3\8x« + 3
1.10.9. The function /(#) is given with the aid of the limit
/? -> cc
x r 1
f(x)=\-l if
{ if * = ±1
or, briefly, / (x) = sign (| x |
—
1) (see Problem 1.5.11 (n)).
The graph of this function is shown in Fig. 27.
Fig. 27
\ 50
have increased times. Taking into account that
lim —l
e.
J
Hence, after 100 years the population of the country will have
increased e'1 7.39 times. «
Of course, this estimation is very approximate, but it gives an
lim-
(a)
2 — x — 2x*
(b)
z /__ x _2 i
(c) lirn^E^;
2x- — 5x + 4 .
(d) lim
5a-
2 — 2x — 3 *
68 Ch. I . Introduction to Mathematical Analysis
(e) lini(j/>+ 1— — 0;
X-> QO
a
(c) lim .
1
s
~~
'".,
a "' n , ;
(d) lim tan 2x tan (n/4 — x);
gc-»ji *-*ji/4
— 3 tan x
(e)
.
™
..
3
tan 3 x
cos (x+n/6)
*
(c) lim^=^;
*
(d) lim (1
*->0
+ 3 tan 2
x) cot2 *;
x->0
22
(e) lim (sin2*)t- *; (f) lim lg-r\) ;
x-yn/4 x-y cd \
zx -r 1
/
X-+JI/2 X-yJt/2
(k) hm
x +0
— x
.
lim q-K- =
*-*a P to
f(x) = (x — l) sin
2 3
(b) as a: 1 are infinitesimals.
—
1
\J)(a:) - sin 3
j ; 1,
is bounded:
I
sin 3 — < 1.
(a) /(*)= as x -+ 4
^-yi >
(b) f (x) = as x — oo
are infinitesimal.
1.11.3. Find
lim x sin (l/x).
x - o
(a) /,
= tan x (b)
(x) 3
; / 2 (x)
= ;
x ->
im ifE^ =
* x
li
-
m
tan a:
3
x,1 =
2
l
J X
,.
lim
li
-
——
tan*
^
3
liin a:
2
= 0.
Hence, tanx 3 is an infinitesimal of a higher order relative to x.
(b) We have
* - * x -* X2 3/^
J/
(a) / (*)
= 3* + 2* + 5 2
and cp (x) = 2* 3
+ 2x — 1
f(x) = 2x + 3x (*) = (* + 2)
2 2
(b) and cp ;
*-> oo
2* 3 + 2x— 1 2 + 2/x — 1/x 2 3
it!), which means that these infinitesimal functions are not com-
parable.
1.11.9. Let x —
O. Determine the orders of the following infini-
tesimal functions with respect to x:
(c) Vl +x — 3
1; (d) sin 2* — 2 sin*;
(e) 1 — 2cos(x+£); (f) 2 |/sin*;
(g) TZTf; (
h ) tan* +* 2
;
a (x) ~y (
x )> P (*) ~ 6 (*), then
If
then
/ (x) a (x) ~ ka (x).
If
a.(x)~y(x) 9
P(*)~Y(*),
then
a (x) ~ P (x).
For two infinitesimal functions to be equivalent it is necessary
and sufficient that their difference be an infinitesimal of a higher
order as compared with each of the two.
Listed below are infinitesimal functions:
(a(x) is an infinitesimal as x—+0)
(1) sin a (x) ~ a (x); (2) tan a (x) ~ a (x);
(3) 1— cosa(x) ~ [a(Jc)] 2 /2;
~YTTi~'* TT7~* ;
(c) sin
Solution, (a) By formula (8) at P=l/2 we have
1 1
X
o/T+^-D^i. X.
y 2
+ Vx =X 3 3
'*V 1 +X 1/2 ~X 3
'\
/ v I-
llm
sin 5* /i\ i- 1 — cos a;
a) (b) l"n
(
.0 mna-.,^
1" 0+4*)' * '
_ ,_ cos |
•
/
(c)
\ r
hm 4
In cos
—
a:
;
,,v
(d) hm
1; V^" 1 -f-
\
a: -f- a:
^
2 — 1
,
.pv |. 3 sin* * * — + 2 3
sm x ~ tan ^ ~^ 2
1
~~ cos 2 x )*-r xb
(g) Hm ^
x ^ o
^°(arctan |/* ) (*
5
/ - *
1
j.
1 — cos *+2sin x— sin x— * + 3* 3 2 4
'
'
x ^ tan * — 6 sin- x
3
* — 5* -}-
3
J™lF(T+W , i o-4F = T- Il ,1
= 4 hm „ cos* — = — 4 hm
I A ,.
—x — =
2
/2
—2.
7i
a: -> * x -> o
x
V 1 +x + x — - 2
1 (x -\-
2
)/2 ~ x/2, sin \x ~ \x.
74 Ch. I. introduction to Mathematical Analysis
Therefore
vT+7+x — —
—=—
5
lllTl r—-j
l
r
lUTl —x/2 I
Hence,
,. sin 2x-\- arc sin 2 x — arc tan 2
x
= r 2x
= t2
hm Yx
Ilrn
35
•
lim
sin^ln(l+3,_)
m ^.3,_3
'
V 1—0.006 ~ — = 0.997. 1
(a) f/T+x—l~jx;
§ J. 13. One-Sided Limits 75
x '^ { x
(a) Vs\n 2 x + x*; (b) ^ 2* .
1 + /
1.12.7. For x—+2 determine the order of smallness, relative to
the infinitesimal $(x)=x — 2, of the following infinitesimals:
, , ,. sin3x /U v
,. In (1 + sin 4a:)
e 'i"3*_l
< C
>
^ In (1 + tan 2x)
— cos 2x)
'
<
d>
J
1
arc tan 3x
™ aTclI^r =
In (1+2.-3^ + 4*3) .
/T+^-l
g
"i o
In (1— — 7jc») '
y }
"J" 1-cosx
*
3,
1.12.9. Find an approximate value of the root j/ 1042.
>
.
{ if x I
(
b) / (*) = 77=777 as
76 Ch. I. Introduction to Mathematical Analysis
r{ s2x
(c) f(x)= -r as x 0;
as x —
+ 7l /(] _„
1;
i
V2 |
sin x|
but
= j
sin a:, if < < n/2 a:
f
|
sin x
-ji/2 < x < 0.
|
) —sin a:, if
Fig. 28 Hence,
_ = 0, f (1-0)^3.
* -> 1 - X 1 - 1 + '
y)
" ,+0 = »
i
! t1.(
3+
H^')" 3+, = 4 -
(e) Let us choose two sequences, {xn \ and {a:;}, with the general
terms
Hence, the function /(*) has no limit to the right at the point 0;
taking into account that f (x) is an even function, we conclude that
it has no limit to the left either (see Fig. 29).
^^^^
1-/I
ii
Fig. 29
'
W" j
3x +2 at 1 <x<3
has a limit to the left equal to 2 and a limit to the right equal to 5.
(c)
£ Continuity of a Function.
Points of Discontinuity and Their Classification
Let the function y =
f(x) be defined on the set X and let the
point x £X
be the limit point of this set. The function f (x) is said
to be continuous at the point x if lim f (x) f (x ). The latter con-
=
dition is equivalent to the condition lim
Ax - o
&y(x )=^ lim [/(x
a* -* o
+ Ax) —
-/(*.)] = <>.
78 Ch. I . Introduction to Mathematical Analysis
— +
is a non-removable discontinuity of the first kind, and the d-ifference
f(x 0)
{) + —
/(* —
0) is called a jump discontinuity of the function
f (x) at the point x, r
/ (*«)
=- 3a 4
+ 5x» + 2x1 + 3x + 4. {)
for — oo < x ^ 1,
/<l-0) = /(l+0)=/(l),
the function is continuous at the point x=l.
Consider the point x 3\ —
/(3 — 0)= lim (6— 5x) = — 9;
x-> 3-0
/(3 + 0)== lim (x — 3) = 0.
*-* 3 +
We see that the right-hand and the left-hand limits, though finite,
are not equal each other, therefore the function has a disconti-
to
nuity of the first kind at the point x = 3.
The |ump of the function at the point of discontinuity is
f(3 + 0)-f(3-0)=0-(-9) = 9.
The function is
(c) defined and continuous throughout the entire
number scale., except at the point #=3/2. Since 2x — 3>0 for
x > 3/2 and 2x- 3 < for x < 3/2,
i 1 at x > 3/2,
\ — 1 at *<3/2.
Hence,
/ (3/2 + 0) = / (3/2-0) = -1. 1 ,
[ 1 furx=-0;
80 Ch. J. Introduction to Mathematical Analysis
= (
4-3* for x < 0,
(d) (x)
2a + x
/
for 0;
(e) /(x) = arctan(l/*); (f) / (x) = (** + l)/(* + 1).
Solution, (a) The function is continuous at all points ,v=^Q. At
the point x we have =
sin x
/(0)=1; liin :
= liin
x
1.
Fig. 30
sided limits are finite and coincide. But at the point x =— 1 the
§ 1.14. Continuity of a Function. Points of Discontinuity 81
|
1 if a; is rational,
\ if x is irrational.
X(x) is called the Dirichlet function. For instance, X (0) = 1; h(— 1/2) = 1;
X (|/""2) = 0; *(jx) = f
etc.
Solution, (a) The function E (x) is defined throughout the entire
number scale and takes on only integral values. This function is
discontinuous at every integral value n of the independent va-
riable, since E (n — 0) = n— 1;
E (
n + 0) = n (see Fig. 31). 9>
(b)Let us choose an arbitrary 3 r**>
point x on the x-axis; two cases
2
are possible: (1) the number x i
! '
i
i
i i ——
i
|A*/|HM*o)-M*)Hi- Fig. 31
Thus, both cases the difference A*/ does not tend to zero as
in
Ax—>0. Therefore, x is a discontinuity. Since x is an arbitrary-
point, the Dirichlet function X(x) is discontinuous at each point.
The graph of this function consists of a set of points with irratio-
nal abscissas on the x-axis and of a set of points with rational
abscissas on the straight line y 1, that is why it is impos- =
sible to sketch it.
82 Ch. I. Introduction to Mathematical Analysis
J
f(x) — f (x ) |
= |
(ax + b) — (ax + b)\ = \ax + b— ax —b\ = \a\\ x—x |
the indicated limits are different and hence the function is discon-
tinuous at all points x=?^0.
On the other hand, let now x 0. Find the absolute = value of
the difference \f(x) f(0)\: —
\f(x)-f{0)\ = \±tf-0\=.x*.
It is obvious that x 2 <e at |x| <j/"e. If e>0 is given, then,
putting 6<j/e and \x 0| = |x]< 6, we obtain A/(0) = x < e.
2
— |
|
(a (x) = \
x+2 for x<2 f
—
<
f
1 x 2
1 for x>2; x = 2;
(b) / (x) = arc tan ^-5 x = 5; o (c) f (x) = j^H* x» = 0;
(d) /(A') = tanx; x = n/2' f
§ 1.14. Continuity of a Function. Points of Discontinuity 83
f(2 + 0) = X*lim+ (x 2 — 1) = 3.
2
Here the limits to the right and to the left exist, are finite but
do not coincide, therefore the function has a discontinuity of the
first kind at the point x = 2.
X
/ ex
\
I 3 for * = 0;
K/Mor^O,
| for x = 0;
(d) /(*)-- lim (smxr; (e) f(*) = ii|££l
(f) f(x) = E(x) + E(-x).
1.14.8. For each of the following functions find the points of
discontinuity and determine the jumps of the function at these
points:
(a) /W = F=^T- 1
;
(b) /W = * + r^!|;
—x for
(d) f(x)=\
(
(
_ 2 .
for X>L
1,
/(*)
, v
= _;
tan x
84 Ch. I. Introduction to Mathematical Analysis
+ 4x + 8x + 8x + 4 = (x + 2x + 2)
x* 3 2 2 2
,
4cos# —2 = or cosa:=1/2,
whence
x = x n = ±n/3 + 2nn (n = 0, ± 1 , +2, . . . ).
^n
+ nn (&» P» n = 0» ± U ± 2, ... ).
(a) y = cosx n y
where n is a natural number;
(b) y=^ cos log x\
(
n/4 + 2nn<^x<^3ji/4 + 2nn,
|cos*|<l/ 2/2, i.e.
^ 5n/4 + 2 nn^x<^7n/4 + 2nn.
1.15.3. For each of the following functions find the points of
discontinuity and determine their character:
(a) y = -TTT- -
— o » where u = ——7 {
( x— for1 0,
(b) y = u\ where u = x+l fof X<Q .
(c) y= j-j-^-^
l
where w = tan*.
Solution, (a) The function
, v 1
86 Ch. I. Introduction to Mathematical Analysis
Jim y = lim y = 0,
X -y 1 U -* cc
y = f{f[f(x)]\.
Solution. The point x = 1 is a discontinuity of the function
If x^=\, then
" =f [/(*)] = ,
-i/'d-v, "T 1 -
* = /</[/<*>]Htz^^ = *
iscontinuous everywhere.
Thus, the points of discontinuity of this composite function are
x =x=l, both of them being removable.
9
§ 1.16. Fund. Cont. on Closed Interval: Properties 87
(2) f (x) has the minimum and maximum values on [a, b]\
(3) If m= min f (x), M=
max f(x), then for any A satisfy-
ing the inequalities A^.M there exists a point x £[a, b] for
which f(x ) A. =
In particular,
if f(a)-f (b) 0, then we can find a point <
c (a <c<fr) such that f(c) 0. =
II. Continuity of an Inverse Function. If the function y = f(x) is
defined, continuous and strictly monotonic on the interval X, then
there exists a single-valued inverse function x = y(y) defined, con-
tinuous and also strictly monotonic in the range of the function
y=f(x).
1.16.1. Does the equation sin*— x+ 1 =0 have a root?
Solution. The function
/ (x) — sin x — * + 1
a * 2w + + a,* + 2
" . . .
+ a 2n x + a 2n = , 1 (*)
/ (*) =a x*"
* 1
+ a lX +...+ a *n
2n x -f a 2n+v
/ (x) = x /4 — sin nx + 3
3
/(-2)=1.; /(2)=5.
Since 1 < 2-^- < 5, then, by property (3), withi-n the interval
[—2, 2] there exists at least one point x such that f(x) = 2-y.
1.16.7. Show that the function
( 2x +\ for-l<x<0,
f(x)= |
2 X for x = 0,
[
2*— 1 for 0<*< 1,
§ 1.16. Funct. Cont. on Closed Interval: Properties 89
lim /(*) =
but f{x)=£\ for any
!,
/ / // / % x
x. It means that the function reaches -2-10 12 3
its minimum value but never p .
32
reaches its maximum. This is be-
cause there is a discontinuity at the point x =n (see Fig. 32),
2n +
Prove that the function y =
1.16.9. \/ x (n a natural number)
is continuous throughout the number scale, considering it as a function
inverse to y = x
2n+1
.
where a a iy a 2 ,
an are positive numbers, there exists
, ,
|
sin x 2 — sin x l
sin
2
cos—
-x
<2 sin
1
<2 •
\x 2 x t \
(x 2 x x ).
Since <e< 1,
— sin x < (x — x
e |
sin x2 x \
2 1) y
whence
(x 2 — x — e(s\nx — smx )=y(x — y(x >
l
) 2 l 2) l)
0.
(
x+ 1 at — <x<0,
1
f{x) = \
_x at 0<x< 1
2n ~
(U\
(D) — — — < y2n~TT
1 l
'
2 4
'
6 2n' ' ' '
(a) ||*|-2|<1;
(b) ||
2 — 3^|— J_|_> 2;
log 2 -,(*-3)>-5
has no solutions.
(a) ^^signx;
(b) x = |
x sign
|
x\
/ (x)
= ax + b
the values of the argument 1 2, x = x n (n =
) form an arithmetic ,
. .
.
1.17.15. Prove that the product of two even or two odd functions
is an even function, whereas the product of an even and an odd
function is an odd function.
1.17.21. Let us have two periodic functions f (x) and y(x) defined
on a common set. Prove that if the periods of these functions are
commensurate, then their sum and product are also periodic functions.
1.17.22. Prove that the Dirichlet function X(x) (see Problem
1.14.4(b)) is a periodic one but has no period.
f (x) -=s\nx-{-coscix
(x if — oo < x < 1,
2
x' if 1<x<4,
2X if 4 < x < oo.
1.17.28. Show that the equation x 2 + 2jc+ 1 == — + Yx1 has no
real roots.
y = f(x-i) + f(x+0,
94 Ch. I . Introduction to Mathematical Analysis
where
k(\—\x\ll) at
fW =
|
j
at \x\ > /.
1.17.34. Let the sequences x n and y n diverge. Can one assert that
the sequences x n +
y tn x n y n diverge too?
1.17.35. Let an be an interior angle of a regular n-gon (ai=3,
4, ...). Write the first several terms of the sequence a n Prove .
that \ima n = n.
1.17.36. Prove that from \imx n =a it follows that lim |a:
w |
=|a|.
n -> oo n -* oo
(a) V2 y j/ 2 [ 2 y V 2 V 2 |/T
§ 1.17. Additional Problems 95
(
£(*) + £ (2*) 1- ...+E (nx) \
Prove that
oo, if n > m,
lim P(x) = |
a /6 , if n =m y
0, if n < m.
1.17.46. Find the constants a and & from the condition:
(b)
X
lim
-> - oo
()/ > — x + — ax— b) = 1 Q.
' x)
(b) / {x) = 2^ l/ii
\
fl ! L l
§ 1.17. Additional Problems 97
1.17.62. Prove that if the function f (x): (1) is defined and mo-
notonic on the interval [a b]\ (2) traverses all intermediate values
y
between f (a) and f(b), then it is continuous on the interval [a, b],
1.17.65. Prove that the equation x 2*=1 has at least one posi-
tive root which is less than unity.
least one value the sign of which is opposite to that of the coeffi-
cient at the superior power of x of the polynomial, then the latter
has at least two real roots.
4-3148
Chapter
DIFFERENTIATION
OF FUNCTIONS
A* - A* Ax - ax
2.1.1. Find the increment Ay and the ratio ^ for the following
functions:
^
Ax
^L =10o
= 0.0001
2.1.2. Using the definition of the derivative, find the derivatives
of the following functions:
Aa: - lx
'
2 lim
A* -*
sm( ax + £.
V 2
^ Hm —
^Vax^O / Ax -
sin
A*
= — a sin ax.
In particular, if a^=l, then */ = cosx and y' = — sinx.
2.1.3. Show that the following functions have no finite derivati-
ves at the indicated points:
(a) y= ]/ x at 3
the point x=0;
(b) y=\/x — 1 at the point x=l\
(c) y = 3\x\+l at the point x = 0.
Solution, (a) Ay - j/(x+ Ax) 3 — j/x 3
.
=
F
=; hence, ,'(0)^^ = 00,
Ax - +
lim g
ax
= 3.
hence,
A*
lim
-> -0
Ay
aa:
= — 3.
Since the one-sided limits are different, there is no derivative at
the point x =
(see Fig. 34).
4*
100 Ch. II. Differentiation of Functions
Ar/=|ln(l+A^)H|^
ln(l
( + A*) at Ajk>0,
ln | + Ax) 1 at Ax<0.
Therefore
I n (1 + Ax) at A* > 0,
Ax
Ax ln(1
+ Ajc)
at Ax<0,
whence
lim ^=+1 and lim ^= 1.
(1) c'=0;
(2) (u±v)' = u' ±v'\
(3) (cu)'=cu'\
(4) = u'u + uv', the product rule;
(uv)'
(5)
^y = ii!E=ffHl(t,^0) the quotient rule. f
and y' (x
x ) = y'u (u )
u'x (x ), the function of a function, or chain, rule.
(4)/
V
(tanw)'=-^-;
V (5) (cot u)' = — -X-
/ V / \ / ;
»
cos w sin u 2 »
2
(a) = 5*
*/
2/3 — 3* 5/2 +2*" 3 ;
a
(b) y= 3
(a, b constants).
y xl x y x
lo
15 ,/-- b
'
2
n — cos x
(a) v = 3cos* + 2sin x\ (b) v = -
/ o
\ o /u\ .
• si at }—
smx — cos x
!
/y */ ;
v '
1 v '
(cos a:— sin (sin x — cos x) — (cos x-{- sin (sin #+cos x) jc) a:)
*
(sin a:— cos a:) 2
3
= (* *+ (arc sin *)' * = 3x
a:
(d) y' 3
)' arc sin 3 2
arcsin * + -^-== .
2.2.3. Find the derivative of the given function and then com-
pute the particular value of the derivative at the indicated value
of the argument:
(a) /(*) = !
— 1/7 + 16/* 2
at x = — 8;
102 Ch. II. Differentiation of Functions
(c) /'(/) ~~
(1 — sin t)
2
1 — sin /
*
Whence /'(ji/6) = 2.
2.2.4. Taking advantage of the differentiation formulas, find the
derivatives of the following functions:
(a) y = 2x + 3x— 5;
3
(b) y = V~7+ -^= + 0Ax 10
;
.
x 2x 2 + + a; 1 , x-{- V*
-cos (p
ex -j- sin x
(g) y = ex (cosx+smx)\ (h) #
(a) # = sin #; 3
(b) = lntanx; (c) y = 5 C0SX \
yx = (sin 3 = 3 sin 2
A:) S in x (sin x)'x x cos x\
(c) x )cos x a; In 5;
§ 2.2. Difjerentiadon of Explicit Functions 103
= sin C0S
^ + 3x2 = 3 *' C0t + ^ 1 >
(J* + '
— 1 '
1 1
(-2*)=- * . (x^O).
V\ — (\ —
,
x2 ) 2V~T^x* i^ii^T^j?
2.2.6. Find the derivatives of the following functions:
y = (\ + 3x + 5x y = (3 — sin x)
2 3
(a) (b) )*; ;
(d) + 2*+l + ln *;
j/2e* 5
= 4(l+3x + 5x 2
)
3
(3+10x:);
l 1 1
1 + X*
We
/e have
0'
l
1
2n-
2 (1 + s — 4x _2
)
a
2(1— a:
2
) 2(1— x 2 )
2x~y (i+* 2 2
y-(\-x*)*(\+x 2 |i-* 2 l(i+* 2 r
i. e.
v k
i
-i- x-
) )
frb at M<i.
f
,
= \ 2
"t|*|>i.
l-TT?
At |x;| = l the derivative is non-existent.
s\nh ax
e
(e) # ~ sinh 6a;— cosh 6a;
Solution.
— sinh x) /cosh x
|^(cosh 2
x 2 2
(a) y =
y 4/=-;
l—x
(b) » = U) (u(*)>0);
x 2
sin 3 * cos 2 x\
K~~
x+
(d) y^(VT^c) \
Solution, (a) Apply the method of logarithmic differentiation.
Consider, instead of y, the function
1.2*.
|
= 1 — 24a; +125x — 3 2
+ 75
14a:
Z
x
1
3(a: 2 +1) 1
xy'=(l— x 2 ) y.
Solution.
y> = e -x>/2 — X e -x*,2 = e -x>/2
2
(
1 _X 2).
o < / ,
Solution, (a) y
, . .
= — (cos
v
x)'
V 1— cos
—
2
*
— sin x
V sin 2
x
— sin*
sin x \
'
ferent iable.
(b) The domain of definition of this function is the interval
— 1.
y' ,r
{
-
7=^ee
•
l/^— 2
(-2x) at x=^0 and *=^± 1.
Since V 1— Ax 2 — 1 yAx 1
2
, then
X=- as Ax—> + 0,
106 Ch. II. Differentiation of Functions
Thus, yL (0) =/= y'+ (0), which means that the function under consi-
deration has no derivative at the point x = though it is conti- y
is continuous at the point x = 0> but does not have even one-sided
derivatives, since -ltl-
*
Ax
= sin-r—
Ax
(e) f (*)
= Kl+sinh 4.r, 2
(f) f(x) = e
ax {zoshbx+smhbx).
(c)
f/(x+2)* V(x+ 3)"
2.2.15.
COS 2 X
'
4- sin 2 a:
show that
f(ji/4)-3/'(*/4) = 3.
2.2.16. Show that the function
* 2* 2
+ ... +uv = %C
=
nu
- (n) k (n k)
v {k \
k
n(n ~ " {n ~ k +
wneie u {0)
where =uu, u {0)
v =vu and
diiu C =
k
o„
-2-3.
= -— £)!
1
1) '
- .As fc!(/t
are
y
m = J. \jn sin (7x
+ n^+ 3" sin ^3x + n^) ]
2x+l
y'<
(g) +a:— 2
x2
'
x +2 .
Whence
/=-l(*-l)- -l(* + 2)- 2 2
;
{x-\)» 1
(x + 2)»
2.3.2. #
v = cx
^4^;
«
find
-f
Solution. Transform the given expression in the following way:
ax -\-b be — ad be — ad
= ex-\- d c c(cx-\-d)
[CX + d)~K
Whence
— ad
y' = (-i) be c (cx + d)~
y" = (-l)(-2) b-^C*(CX + d)-\
y'" = (- 1) (-2) (-3) C3 (CJC + ^ 4>
/
<»)= = (_l)»n!*E_£^ C "( w + d)-«»+i) =
§ 2.3. Successive Differentiation. Leibniz Formula 109
_ x _ i r_j .
i_i
y x2 - 1 2 U+l" "^— lj 1 •
(x-
(a) y=x 2
smx\ find # (25) ;
(b) y = e x (x
—l); find 2
*/
(24)
;
(c) y = x
e° s'm$x\ find y
(n)
.
*/
(25)
=x l
sin (x + 25 y) + 50a: sin (x + 24 ~ ) + 600 sin + 23 y ) =
= (a:
2 — 600) cos # + 50* sin x.
2.3.5. Compute the value of the nih derivative of the function
3 2
u = .,
* '
c at the point x = 0.
Solution. By hypothesis we have y(x)(x 2 — 2* + 5) = 3x + 2. Let
us differentiate this identity n times using the Leibniz formula;
then (for n^ we obtain
2)
-
y
n
(x) (x
2
— 2x + 5) + ny< ~ n l)
(x) (2x—2) + n(n {)
y<»-» (*). 2 = 0.
Putting # = 0, we have
W n)
(0) — 2ny {n ~ 1)
(0y+ n(n—l) y
(n ~ 2)
(0) - 0.
Whence
y<»> (0)=±ny<»-» (0)-^^^~ 2
>
(0 ).
We
have obtained a recurrence relation for determining the nth
derivative at the point x = 0(n^ 2). The values y (0) and y'(0)
are found immediately: (0) = 2/5;
110 Ch. II. Differential ion of Fuw turns
W= -3x
2
>i -4x-f
\
19 ,
n 19
(0)==
y (*-2*+5)' ;
^ 25'
Z
y 5
" '
25 5 5 125
•
y
"'(C)\ —L ^ 56 3»2
"
19
~
234
625'
5 125 5 25
y = cfi 2x + c xe2x + ex 2
.
2.3.8. Using the Leibniz formula give the derivatives of the in-
dicated orders for the following functions:
(20)
(a) y=x*s'mx; find */ ;
y = e~ smx;
x
(b) find y"'\
(c) = ex (3x — 4);
y 2
find y
(n)
;
/ \ 1 /u\ l+#
y = x n [c t
cos (In x) +c 2
sin (In a-)]
1. The Derivative an
Inverse Function. If a differentiable
of
function y = f(x) t
a <x<b has a single- valued continuous inverse
function x = g(y) and y'x ¥=0 then there exists also
1
Xy , .
yx
Y" yxx
y'
xx = -^, y£ = ^>
{
and so on.
xt Xf
true:
- x't ytt—x'tt y't
(xt)*
We x = Sx + +
2
Solution, (a) have y' 15x 4 1, hence,
l 1
yx
6x 2 +15x 4 +l
112 Ch. II. Differentiation of Functions
, 1 1 1
yx = — 1
=^
Ve~iy~—i
- - x
(d) x = ec\ y = e~ ct
.
Whence
dy
~
dx
=—
a
a sin
— cos 7r = cot 172(1
i
/ , /
(/
v =7^
7 2fcri).
7
.
n , x
, . — 2 cosec 4
2
^
^ ~ cot! sin~27
*
dy
-f = sec
ctf
— cosec =
9
2 ,
/
9
2
,
/
4- cos 2t
.
sin 2 2/
20 .
•
d#
dx
4 cos 2/ sin 2/
4 sin 2 2t
^ V
, kn
2
(a) (x = acos 3
/, (b) jx=
fx=/ + 3/+l, /
3
;
y = bsm*t\ t* — 3t + U
\y = t*-3t l;
y = a(s\n — /cos/); t
I
= ^sin
t/ /.
= 3b sin
y'
t
2
/ cos t\ x\ — — 3a cos 2
1 sin t;
2
3b sin
^= - = -a
/ cos / b , . / . .
/rt « . . . ji \
3acos^sin/
ta "^ (' =*<2*+ 1) T ).
Then we shall find y"^ using the formula
u =^ xt
where
= ~acos 2
t'
Whence
„ b b
a cos 2 1 ( — 3a cos sin 2
/ /) 3a 2 cos 4 1 sin T
(d) = cos t
— e 1 sin / — e (cos — sin
x
/ t);
y
f
t
=e l
sin t + cos < = e' (cos + sin / /);
,
cos / + sin /
#
^* cos / — sin t
*
(a) y = t
3
;
(b) x = sec/; #=tan/.
Solution, (a) First find
= y't
= 3t 2
y
whence
yx = — Wle- = — 3eH\ %
6
xt
(d) x
M
+ tf
'»
,
3x 2
+ 2xy
2.4.7. Find yxx if:
2(l+y2 )
yXX yb
a;
3 — 2x + + */— 5-0 2
*/
2
5.x: and y\ x=x = 1.
3* 2
— Axy — Ax yy' + 5 + y' =
2 2
0.
6x — 4y — 8xyy' — 8xyy' — \x
2 2 y' 2
— \x yy" + y" = 2
0.
Putting x=l; y^=l and t/' = 4/3, find the value y" at #=-. 1:
a
6 — 4— y64 — T
A 64
— 3^/
„ n
=0, y
„
- — 827.
Q 22
y= /
find y'x ;
1 +6 cos t
'
1
-f b cos I
X = + 2, y=
3
(c) /* t /3-t; find yxx ;
y—yo = —77
y' (*o)
(x # ),
y' (x )¥>o.
The segments AT, AN are
called subtangent and the
the
X
subnormal, respectively; and the o
lengths and MT are the MN Fig. 36
so-called segment of the tangent
and the segment of the normal,
respectively (see Fig. 36). The lengths of the four indicated segments
are expressed by the following formulas:
AT = _y_
y'
AN: MT =
MN = \y\V\ + {y') 2
.
2.5.1. Write the equations of the tangent line and the normal:
(a) to the curve y=^x s 3a; — +2 at the point (2, 4);
116 Ch. II. Differentiation of Functions
y = x* + 3jk 2 — 16,
y = 3x
2
y
xt = — 2, x2 = 2, yx =y = 2
12.
3x 2 —3 = — 2,
3a:
2 — 3-9,
whence ^ = — 2, x2 = 2. The required points: M — 2,
x
( 3), M 2 (2, 7).
gent to these curves and drawn at this point. Find the points of
intersection of the curves by solving the system of equations
y=4 — x,
y = 4—x /2. 2
Whence
AMO, 4); Af 2 (2, 2).
0'(O)=O f
y'{2)=-2.
The slope of a straight line is constant for all its points; in our
case it equals —
1. Finally, determine
tanq^ — 1; ^ = 45°;
tan q) 2 = —1+2
+ 2 ~~
1 1
3
,
<P 2
arc tan « 18.5°.
2.5.4. Prove
that the segment of
the tangent the hyperbola y
to c/x =
which is contained between the coordinate axes is bisected at the
point of tangency.
Solution. We
have y' =—
c/x 2 hence, the value of the subtangent \
y_
y'
( x = + 3t — 8,
t
2
y = 2t — 2t — 5
2
\
at the point M
(2, —1).
Solution. First determine the value of t corresponding to the gi-
ven values of x and y. This value must simultaneously satisfy the
two equations
|
t* + 3t—8 = 2
\ 2t
2
— 2t — 5-— 1.
y =2
== == = T'
' '*
(il)/=2 (27 + 3)^2
And so, the slope of the tangent at the point M (2, —1) is equal
to 6/7.
cos 20 sin 0,
^cos26
y {)
= a sin 6 sin 26 -./•
^ cos 0,n
+ a V cos 20
.
V cos 26
y (Ji/b)
Thus, the slope k-=—Q = 0. Consequently, the line tangent to
*e (n/6)
the lemniscate at the point with o
= ji/6 and p = aV cos 20 =o
2.5.8. Find the equations of the tangent and the normal to the
following curves:
(a) 4a;
3
— 3xy + Sx — 5xy — 8y + 9x+ 14=0 at
2 2 2
the point (—2, 3);
(b) x b
+ y — 2xy = at the point
b
(1, 1).
1 2x 2 — 3y — 6xyy' +
2 — 5y — 5xy — 6yy' + 9 =
1 2x ' 1 0.
»-3=-y(* + 2)
and the equation of the normal
y-3-=±-(x + 2).
2.5.9. Through the point (2, 0), which does not belong to the
curve # = Jt\ draw tangents to the latter.
Solution. Let (x Qy x* ) be the point of tangency; then the equation
of the tangent will be of the form:
y—4=y' (x {x—x ) )
y—xi = 4x (x—x 3
).
— %t — 4xq (2 x )\ 3x1 — = 0,
whence x x = 0; =
8/3. Thus, there are two points of tangency:
AM0, 7W 2 (8/3, 4096/81).
0),
Accordingly, the equations of the tangent lines will be
A 4096 2048 / 8 \
120 Ch. II. Differentiation of Functions
/'(*)= 15a:
4
— 45a: + 5 = 15 [(x — 1/2) + 1/12].
2 2 2
\2y't = 3x 2
-x't
or
yt _x 2
(1) at —
2 < x < 2 the ratio y\\x\ is less than unity, i.e. the rate
of change of the ordinate is less than that of the abscissa;
(2) at x = ±2 the ratio y\\x\ is equal to unity, i.e. at these
points the rates of change of the coordinates are equal;
(3) at jc<-t-2 or x > 2 the ratio y\\x\ is greater than unity, i.e.
the rate of change of the ordinate exceeds that of the abscissa.
Therefore
v = s'i=ay s (a = const);
§ 2.5. Applications of the Derivative 121
whence
s"ti =v' t
=a jy=- s = a t
1
12.
is wound on a drum,
at a rate of 3 m/min. Determine the speed
of the raft at the moment when it is 25 distant from the bank m
if the drum is situated on the bank 4 above water level. m
Solution. Let s denote the length of the rope between the drum
and the raft and x the distance from the raft to the bank. By
hypothesis
s
2 2
42 =x + .
x t-T St '
we obtain
^25 a +4a
^;==
25
.3» 3.03 (m/min).
2.5.15. (a) Find the slope of the tangent to the cubic parabola
y=x* at the point x = ]/~3/3.
(b) Write the equations
of the tangents to the curve 1/(1 x2 ) y= +
at the points of its intersection with the hyperbola y=l/(x+l).
(c) Write the equation of the normal to the parabola y #2 -f-4x-f 1 =
perpendicular to the line joining the origin of coordinates with the
vertex of the parabola.
(d) At what angle does the curve y = ex intersect the y-ax\s?
r =a ^
1 — e cos M — y (cos 2M — 1)
where M = ^-(t — t
n)
= time parameter
t
1
The perigee of the satellite orbit is the shortest distance from the sate-
llite to the centre of the Earth.
§ 2.6. Differential of a Function 123
d 2 y=d(dy); d 3 y=d(d 2 y) y
dny = d(d"~^y).
If y = f(x) and x is an independent variable, then
d 2
y = y" {dx)
2
\
d3y = y" (dx)\ . .
. ,
dny = y<» {dx)\
But if where u = cp (x), then d 2 y = f" (u) du 2 2
-\-f (u) d u, and
so on.
y = In +e + arc tan e bx
10x
( 1 )
.
_ r (i-fgiox)^ e *xy -I
_ 5g 5x (2g5-y— ^
— [
i+«,io*
(
l+gio*J — 1+gio*
1)
y = 3x* + x—\
at the point x= \ at Ax = 0.1.
Find the absolute and relative errors allowed when replacing the
increment of the function with its differential.
Solution.
Ay = [3 (x + Ax) + (x + Ax) — —
3
1 ] (3a:
3
+ x— = 1 )
= 9xAx + 9x Ax + 3 Ax + Ax,
2 2 3
dy = (9x + 2
I) Ax.
Whence
Ay—dy = 9* A* + 3Ax 2 3
.
y =x s
— 7x + 8
2
y 5 V \2 — x) (2 + x) 2 '
cos31°;
(a) (b) log 10.21; (c) j/33; (d) cot45°10'.
Solution, (a) In solving this problem we shall use the formula (*)
of the preceding problem. Putting x jt/6, rc/180, we compute: = =
^(^)
/x V3
= cos-g-n = -g-;
y (
x) = _ S in-g= — — ;
2.6.6. All faces of a copper cube with 5-cm sides were uniformly
ground down. As the weight of the cube was reduced by
a result
0.96 g. Knowing the specific weight of copper (8) find the reduction
in the cube size, i.e. the amount by which its side was reduced.
Solution. The volume of the cube u==x 3 where x is the length ,
of the side. The volume is equal to the weight divided by the den-
sity: v =
p/d\ the change in cube's volume Av 0.96/8 0.12 (cm 3 ). = =
Since Av approximately equals dv and taking into consideration that
tiv = 3x 2
dx we shall have 0.12 = 3x5 2
x Ax, whence
= I y'x I
A*.
(f) A^lllogaan^^A^i^A,.
From (e) and (f) it follows that the absolute error in log tan x
is always more than that in log sin x (for the same x and A x ).
y = 4x 7a: 3,
b
— 2
+
assuming that:
(1) a: is an independent variable;
(2) x is a function of another independent variable.
Solution. By virtue of the invariance of its form the differential
of the first order dy is written identically in both cases:
dy = y' dx = (20x 4
— 1 4x) dx.
126 Ch. II. Differentiation of Functions
y = V~\n x — 4; d 2 y\
2
(b) find
(c) y = sm x\
2
find d y.
3
2.7.1. Given the functions: (a) f(x) \x\ and (b) = cp (a;) = |
x3 [.
f(x) = \x—a\y(x),
where q(x) is a continuous function and (p(a)=^=0, has no deriva-
tive at the point x =
a. Find the one-sided derivatives /1(a) and
/; (a).
§ 2.7. Additional Problems 127
j
x2 s\n(l/x) at x^=0,
f
\ at * = 0.
Use this example to show that the derivative of a continuous
function is not always a continuous function.
2.7.5. Let
^ ^ / -^
2
>
if a: ^x ,
\ ax +b y
if a: > a* .
2.7.7. From the formula for the sum of the geometric progression
(a) \ + 2x + 3x + +nx n ~ 2
. . .
1 '
(b) \ + 2 x+3 x +
2 2
+n xn -\2 2
. . .
2
(c) the function f (x) has a derivative at the point x , and the
function g(x) has no derivative at this point?
(d) neither function has a derivative at the point x ?
X X* xd
F(x) = 1 2x 3x2
2 6x
2.7.14. Find the derivative of the function y = x\x\. Sketch the
graphs of the given function and its derivative.
n~1 l/x
(b) f (x)=x e ; show that
(n=l, 2, ...).
2.7.19. Show
that the function y atcs\nx satisfies the relation =
(\—x 2 )y" = xy Find y in) (0) (n
f
. 2) by applying the Leibniz for- >
mula to both members of this identity.
(1 -x 2
)
r; (x)-xT'n (x) +n 2
T n (x) = 0.
2
2.7.21. The derivative of the nth order of the function e'* has
the form
(e-x
2
y>»= e - x2 H n (*),
is valid.
2.7.24. u = ^r\n^
2
r^-\
\—v
check the relation ^^
dvdu
= 1.
5—3148
130 Ch. II. Differentiation of Functions
„
_
~
d 2 ydx—dyd 2 x
»
dx*
<«-*> S- 4?+ y
be transformed (where y is a twice differentiable function of x) if
where / = current
k = factor of proportionality (depending on the instrument)
cp= angle of pointer deflection.
Determine the relative error of the result which depends on the
inaccuracy in reading the angle cp. At what position of the pointer
can one obtain the most reliable results?
Chapter
3
APPLICATION OF DIFFERENTIAL
CALCULUS TO INVESTIGATION
OF FUNCTIONS
f(b)-f(a) = (b-a)f®.
Test for the Constancy of a Function. If at all points of a cer-
tain interval /'(x) =
0, then the function f (x) preserves a constant
value within this interval.
Cauchy's Theorem. Let (p (x) and ip(x) be two functions continu-
ous in the interval [a, b] and have finite derivatives at all inte-
rior points of the interval. If these derivatives do not vanish si-
multaneously and (p (a) <p (b) then there exists £€(a, b) such that
9
-5-7 _ __ R
f> /tx
/ \±) — At _ /(Q)-/(-2)
_ ufc
0— (— 2) 2 ~" '
whence £ =— 1.
Solution. The given functions f (x) and g(x) are continuous eve-
rywhere, and hence, in the interval [1, 4] as well; their derivatives
= —
f (x) 2x 2 and g' (x) 3x
2
\4x = — +
20 are finite everywhere; in
addition, g' (x) does not vanish at any real value of x.
Consequently, the Cauchy formula is applicable to the given
functions:
/(4)-/(l)
= nE)
S(4)-s(l) g'iQ*
i. e.
27 —9 3g 2 — 14g + 20 y ^6^ ;
/(2)-/(-l) = /'(g)[2-(-l)] f
or
8 + = 36
1
2
• 3;
whence
Ex 1. I,= l.
yi = l'i = l; y2 =U= — l.
/ (x) = sin 2
x + y cos 2a;
defined throughout the number scale: — oo < < a; oo. The derivative
of this function is everywhere equal to zero:
or
.
9
s\n 2 x = — cos
^
1 2x
•
/ (x)
== arc cos l~* — 2 arc tan x,
—X 2
determined along the entire number scale, since
\
^2
< 1.
p / x
~ ~~~j/7
1 —4a;
~Y 2 4x 2
=^
1 ~ 3 ( 1 * a)a +7 * 2x (
i
~|- x~)
^ y
According to the test for the constancy of a function
l —x 2 \' 2
arc cos -
And so, we have proved that the group of functions for which
f'(x)^f(x) is covered by the formula f(x)^Ce x .
where x 2 > xr
Solution. To the function / (x)
= arc ianx on the interval [x l9 x2 ]
where x x <£<x 2
.
Since
then
arc tan x — arc tan x < x — x
2 l 2 1
.
/(n+l)-/(n) = /^-|/"n=^,
where n <| < n+ 1
If n>/V 2
, then 1>N\ hence 1/(2 < 1/(2jV), whence
3.1.13. Using the Rolle theorem prove that the derivative f (x)
of the function
( x sin —* at x > 0,
c
f(*)
, v
= i
( at a: =
vanishes on an infinite set of points of the interval (0, l).
2nn ! dx»
(X 1)" (Al = 0,
1, 2, ...).
f(x) = (x 2
— \)» = (x— l)
n
(*+l)".
This function and its n 1 successi- —
ve derivatives vanish at the points Fig. 38
lim / (x)
= lim g(x) = or lim / (x) = lim g(x) =- oo,
x -+ a x -* a x -> a x -y a
then
,. f
lim '-)-{= lim
(X) 1- f (X)'
a
g(x) x _> a g'W
f (x)
provided the limit lim exists (U Hospital's rule). The point a /
- a 8 x
,
w
may be either finite or improper -f oo or oo. —
II. Indeterminate forms of the type O oo or oo oo are reduced —
to forms of the type -jj- or ^ by algebraic transformations.
III. Indeterminate forms of the type l 30 oo° or 0° are reduced ,
e x_ e -x.__2x. , In (1 + a:
2
(c) lim lim
)
* _ o x—smx ; (d)
\ /
^ ^ Q C
Q
os3a: — e~ x y
sin 3a:
2 1 /* — 2
/ v „ e
1171 lim ~
Incns^-*! ' (f) 75 t
2
.
,.
Inn ^ttt
/' (x)
= ,.
lim —
ae ax +2ae- 2ax
T~n~\ —— i x
—= Q
oa.
lim
e ax_ e -2ax
— j
In
——
—(\+x) r: :
lim —
ae ax +2ae~ 2ax
r-Tf—
1/(1+*)
— — = 3a. -
(*)7
x> x
^
again represents an indeterminate form, the L'Hospital rule should
be applied for a second time, and so on until the indeterminacy is
removed or until it becomes clear that the required limits do not
exist. Therefore, henceforward we write only the necessary transfor-
mations, leaving to the reader the task of checking whether the
conditions of their applicability are fulfilled.
(b) lim
j/T
v '
l/2
+ 2^+1
+ x+x
= lim
1/(2
3/0+25)*)
2/(3 v
r
J^2 + x)+l
,
—-=ir 4
9
»'
. v
r sin 3x
2
,. —6a: cos 3a:
2
cos (2x 2 — x)
v/
K ^ In cos (2x 2 — x) x ^ (4a:— 1) sin (2a:
2
—x)
= —6n t
. cos 3a:
2
cos (2a:
2 — a:) i .
-
lim -
lim
x - o
4a:
A
— 1
x ^ o sin (2a:
2 — x) '
The limit of the first factor is computed directly, the limit of the
—6 ~ «.
lim
cos 3a:
2
cos (2a:
lim -r
2 — x) ,.
x - o 4a:
-=
—\ x ^
sin
1 — (2a:
2
a:)
= —6 —
— xlim —(4a:— n cos • j-
1
;
1)
^
,
(2a:
n 9
2
—\—r =
— x) = 6 — 1-1 • — 6.
_^
jloga*
Solution. 1. lim
l 0g x
-^f- = lim ,
k _l = \ogae lim —=
i
kx*
0;
2.
,.
lim —= x rn , .
lim
mx
—. —=
rn ~ 1
. . . = ,
lim
.
-7-71 —^ =
m\ r,
0-
140 Ch. III. Differential Calculus: Investigation of Fund's
<
a> (b) Hra (cotx-
o
1 1 1
lim
(c)
e* — \
,.
lim
^^jVlnA:
/
i
l l
x— \J :
\
= r
lim
x ^ j
x— 1— In* =
(x—
r—. —
1) In x
lim
x ^ 1
,
In
1 — 1/* =
,
— [/x,
1
,
= m IJ
_^ !
* In *
*T
+
1
—= r
l ^ ^
lim
,
In a: -|-2 2'
(a) lim x n
\nx(n > 0);
x ->
lim x n In
at ->
a: =. lim
x -+
—- =x
In x
x
lim
->
———- =——\\m1 fx
r
1
" * -+
x n = 0, since rc>0.
sin 2x
^ K^ Q
1+a:
3.2.5. Find the limits:
— —= — \/x
i-
lim \nu
x^ + o
,
= t
lim
.
x^ + oh'smx
-;
In
:
a:
x
,.
lim
^
—
—
;
(cos x)/sin 2 x
= hm
x _^
sin 2 x
xcosx
= A0.
Hence, lim y = e" = l.
x -> -I
tan
(a) lim (sin x) ^; (b) lim**.
X-7C/2 x-»0
3.2.7. Compute
cot *.
lim (tanx)
x -> + Jt/2-0
(tan co *
* = £ co * * ' n *
an x
but
Whence
lim (ianx) coix = e° = 1.
x-> +jt/2-0
(a) lim
* ->
sin x
(b)
v
v
lim
,.
—-—
*-*oo(2x+sin2;t)6>
24-2aH-
-
sin 2x
rj—
s,n *
tan a:
(c) lim
X -+ 31/
li m f__smJJ_/£) _ m jj
x
ii m^s j n _L ^ i .
o 0.
x-*0 smx x-+0 smx x-> x
But the limit of the ratio of the derivatives does not exist. Indeed,
i .
lim
2x sin
—— —=
(
K
1 fx)
1
cos ( 1 fx)
-
A
— lim cos —
, .
1
142 Ch. III. Differential Calculus: Investigation of Fund's
but lim cos (1 /'x) does not exist, hence the L'Hospital rule is not
x - o
applicable here.
(b) The limit of the ratio of the functions does not exist:
Hm
*->
2
oc(2*+sin2*)<?
+ 2,4-sin2,
sin
*
^
,.x\
,
{
'
2.v-|-sin
2
2xy
v
^ ,
but lim e~ sinx does not exist, since the function e~ Anx traverses the
X -> GC
r
lim
2 -f 2 cos 2a:
—
sm
x-> cc [2 + 2 cos 2a; + (2x -|- sin 2x) cos x| e
A
cos- x
= lim -
A
4 cos "
.,
/n
#+(2x+sin
,
4
. .
,
2a) cos
x ^ x a'
* C° S *
= lim .
g -s.nx = 0>
v - x + 4 cos A' -j- si n Q
2x *
lim
SeC x
a-
= ,.
lim
SCC X tan X
t = ,.
urn , = Inn = ...
* -* jt/2 *-*JT/2 x-+n/2 tan x SeC x
lim sin x
.
= t
1.
x->jt/2 x->n/2 Jt->.-r/2
3.2.9. Using the L'Hospital rule find the limits of the following
functions:
r In 2 — 3) a
Xnx —x
^
7 v
I™ ,3
(a:
+ 3*- 10
;
^
/Ux
l
l
™ =
<?H1=£. i-4s^(^/6)
(c)
v '
lim . v cin v '
(d) Hm - 1 X
X -* '
(e) lim arc sin —^— cot (x — a)\ (f) lim (n — 2 arc tan x) In x;
tdn
(g) Hm f-M *; (h) limfa 1
/*— 1)* (a > 0);
* + V * / * ->• GO
tan (jiA-/(2a))
(i) lim (cos mx)"'*
2
; (j) Mm (2— —
§ 3.3. Taylor's Formula: Approximate Calculations 143
X * 1
In a: In x J x -
l/sin a
2
(o) lim a; cosh -
(P) lim
-.0 \2+ V~9-{-x
J/x* 1
through order n — 1 on the interval [a, 6], and has a finite derivative
of the nth order at every interior point of the interval then at
[a, b] the following formula holds true:
+ f
(x-a)+
f(x)=f
^ f" (a)
^
(a) f (a)
+ r (a) + . .
.
+ -^^ + r ©
(
where
l = a + Q(x— a) and <9< 1.
+ f
ln}
&)%, where g=6*, 0<6< 1.
P(l)--0, P'(1) = 0,
P"(l)-0, P"'(l) = 18,
P (l)-72,
(4)
P (5)
(l)=120,
P {n) (x) = (n > 6)
at any x.
Substituting the values thus found into the Taylor formula, we get
P(x)^~(x-ir+^(x-iy + ^(x~\r;
P(x)=3(x— iy + 3(x— 1) 4
+ (jc— l)
5
.
f(x) = \n(\ + x) y
defined on the interval [0, 1], Estimate the error due to deleting
the remainder.
Solution.
/(0) = lnl=0.
The derivatives of any order of the given function (see § 2.3):
rw=(-ir i
{f^i.
^ (0 ) = (_i)"-i („_i)! (n= i f 2, 3, ...).
In (1 + *)=*— y + y— • • •+ y + ^io W,
where the remainder R 10 (x) in the Lagrange form will be written
as follows:
=— =— x U)
(10)
(*)
= / (£)
*
10
9!
<£< *)•
10! 10! (1 + g)
10
10 (1 H- £)
10 (°
f
(n)
(x) = ex .
§ 3.3. Taylor's Formula: Approximate Calculations 145
==e Bx m
Whence
/(*) = +TT+ir+
1
• • •
+(f^. + R ^ x ^
where
I I
< o.ooi
will be fulfilled apriori. To this end it is sufficient to take n 7 >
(7! = 5040). Hence, 7 terms in the Maclaurin formula will suffice.
cos x
than 0.00005?
« 1
— 2T + TT
have an error less
Solution. The right member of the approximate equation repre-
sents the first six terms in the Maclaurin formula for the function
cos a: (the second, fourth and sixth terms are equal to zero; check it!).
Let us estimate R Q (x). Since (cosa:)
(6)
cos x then =— 9
cos Qx
!*.(*)!
-
6!
•
R
x6 < 1*1
6!
For the error to be less than 0.00005, choose the values of x that
satisfy the inequality
-L£J1 < 0.00005.
Solving this inequality, we get |#|< 0.575.
4T~
And 50, within the required accuracy
cos 5° = cos i
00
= 1 —0.00381 = 0.99619.
3.3.6. Compute the approximate value of Y 83 accurate to six
decimal places.
(c) at 0<x<oo.
Solution, (a) According to the Maclaurin formula with the rema-
inder R2 (x) we have
In (1 +*) = *— 2(1 + 6)"
where < |< x.
According to the same formula with the remainder R s
(x) we have
3.3.8. Show that sin (a + ft) differs from sina + /icosa by not
more than h 2 /2.
Solution. By Taylor's formula
whence
+ h) — (sina + /icosa) = —
h 2
h2
'sin (a |
I
sinE |
-\-o(\x — a\ n
)
(1 + *)- = + ax + 1
yl
*+
v-3
- +° ~
{a l)
'
'
nl
^^
Yn
x»+o (*");
^+ o (x n ).
_
, . l
<b) lira
X - X
lim
cosx —e - X,L
x 2 /2
(c)
i- e x sin x —x (1 -\-x)
(d) hm ^ '»
x -» *
(e) hm —^- 2
.
x -> *
lim
- Y + 1 X 2 COS X 1
— (1 -f-#
2
)
1/2
COS X
tan4 * 4
x - *
* + o(* J
x2 x4 1
l+y* + 1/2 (—1/2)
. ,
2
1
)|
-
2 , 7 ; d
4 , ; 4 [,
1 --2" + ,
24
.
+0( *
. .
= 11 m [ J
x - *
V
4 8 24 (x4 )
, .
hm
'
;
' '
= hm , . [
1
1 .
3
*
x-+ L 3
x
(c)
eX _ up to the term containing x*.
x
<x < 2
. . . < xn < b) y where /' (x k ) = (& = 1 , 2, . .
. ,
n), then /' (x)
preserves its sign in each of the intervals (a, x x ), (jc
3 ,
x a ),
f'(x) = lx—\lx.
The function increases if 4x — l/x>0, i.e. * > 1/2.
The function decreases if Ax l/x < 0, i.e. x < 1/2. —
And so, the function decreases in the interval <x < 1/2 and
increases in the interval 1/2<a:<4°°-
(b) Evaluate the derivative
f (
X) = 6x 2
— 18a: — 24 = 6 (x — 3x — 2
4).
3.5.2. Find the intervals of decrease and increase for the follo-
wing functions:
150 Ch. III. Differential Calculus: Investigation of Fund's
(a) / (x)
= cos (n/x);
(b) f(x) = smx + cosx on [0, 2jx].
tj
,
= — sin — ji . ji
.
,2Jfe+l '
2k
26 +2 9
2k-\-\
f/ . v
= 2 cos # H
ri i 1 o
3 = (1 — COS *) (1 —+— ]
COS X — 2 COS 2
=
/ (a:)
v 7 5 1
2
'
1
COS X COS X
_ 4 sin 3
(jc/2) sin(3A:/2)
cos 2 x
are fulfilled.
Solution. We will prove only the right inequality (the left one
is proved analogously).
The derivative of the function
= arc tan x — x + X
3
f (x) -g-
is equal to
= *2(l 2 2
-i)
/ W
/'(*>=_!
1+JC 2
i
1
i
gl
2
(*
+x 2
)
§ 3.5. Testing a Function for Monotonicity 151
is fulfilled, whence
arc tanx < x— -g-.
or
where x = b
.
Let us show that the inequality (*) holds true at any positive x.
Introduce the function
f (x) = s'mx — bx + c
decrease along the entire number scale?
but if f" (x ) =
0, then the question of the existence of an extremum
at this point remains open.
§ 3.6. Maxima and Minima of a Function 153
f
{rn
(x ) > 0, a minimum.
If /2 is odd, then there is no extremum at the point x .
* = <p(0» y = ty(t),
where the functions <p(t) and ty(t) have derivatives both of the first
and second orders within a certain interval of change of the argu-
ment t, and cp' (0=7^0. Further, let, at t = t
y (/) = o.
Then:
(a) if (/ ) < 0, the function y = f(x) has a maximum at x =
= *o = <P('o);
(b) if i|/ (/ ) > 0, the function y = f(x) has a minimum at x =
= *o = <p
(c) if i|5
/r
3.6.1. Using the first derivative, find the extrema of the follo-
wing functions:
(a) f(x) — 9x +
= -jx'— 3 2
7;
(c) = 1)
3
(*-3) 2
;
(c) Just as in item (a), the critical points are the roots of the
derivative /'(#), since the function is defined and differentiate
throughout the number scale. Find f (x):
f' (x) = (x + l)
3
- 3) + 3x (x +
(x
2
l)
2
— 3) + 2x (x +
(x
2
1
)
3
x
X (x — 3) = 3 (x + (x — 3) (2x -3x —
2 2
1
) 1).
Intervals x < X l
< X < x2 x2 < X < x3 x :i
< x < x4 xA < X
3.6.2. Using the first derivative, find the extrema of the follo-
wing functions:
(a) / (x)
=3
(b) / (x)='t/(x-iy+V(x+iy.
Solution, (a) The function is defined and continuous throughout
the number scale.
Let us find the derivative:
x x
= jt/6, x 2 =ji/2, x3 = 5jx/6, x4 = 3ji/2.
Now find the sign of the second derivative at each critical point:
y" (xc/6) = — 3 < 0; hence, we have a maximum y (n/6) = 3/2 at
the point x = Jt/6;
l
point x = ji/2;
2
Fig. 41
^<Hr +3 ^o
Solution, (a) Though the derivative
)
)
/
_2(jr<0),
1 3(x>0)
exists at all points, except the point x= 0, and changes sign from
minus to plus when passing through the point x= 0, there is no
minimum here:
/ (0) = 5 > / (*) at — l<x<0.
155 Ch. III. Differential Calculus: Investigation of Functus
(a) / (x)
= 3 ^4
+ 8 ^3_ 18a: 2_|_ 6 o»
x) = 2x + 24x — 36jc= + 2x — 3)
3 2 2
f[ (
1 1 2x (x
fi(*) = 12(3* + 4* —
a
3),
x1 = 3, x2 = 0, x3 = 1
^" j/'(0) = 0;
;
= 2x e- x2 {6 — 9x -\-2x y 2 2 A
>
|
= <p(t) = — 5P — 20/ + 7,
x t
b
\
y== ty(t) = 4P — 3t —\8t \-3 2
(—2 <t <2).
Find the extrema of this function.
Solution. have We
<p'(0 = 5f — 4
15/
2
— 20.
In the interval (—2, 2) y'(t)=£0.
Find \|/ (/) and equate it to zero:
i|/(0= 12/
2
— 6/ — 18 = 0.
Ax
(a) f{x) = x*e-*\ (b) f(x) = '
xa -|- 4
(e) / (*)
= ^/2x + 3jc — 36jc;3 2
(f ) / (*)
= x In *; 2
(g) f(x) = x In 2
a:.
e v*, if x=£0,
<d > '<*>=.(>. if x=
§ 3.7. Finding the Greatest and the Least Values 0/ a Function 1 59*
this interval is not a closed one, then it can have neither the
greatest nor the least value.
3.7.1. Find the greatest and the least values of the following
functions on the indicated intervals:
'
/2 J
Hence, the greatest value /(— 1) = 8 and the least f(2) = — 19.
is
(b) Find the critical points: f (x) = x(l + 2\nx). The derivative
f (x) does not vanish inside the given interval [1, e]. Therefore
there are no critical points inside the indicated interval. It now
remains to compute the values of the function at the end-points of
the interval [1, e]
f(l) = 0; f(e) = e\
Thus, /(1) = is the least value of the function and f(e) =e 2
the
greatest.
3.7.2. Find the greatest and the least values of the following
functions on the indicated intervals:
—
a:
= cos x — cos 3x
y 2 «
r_ = 4-
3 vi
Hence, the least value of the function in the interval (— oo, oo)
is equal to —4/(3/3), and the greatest to 4/(3 1/3).
f'(x) = a-± =
at x = V'b/a (x>0). Since f" (x) = 2b/x> > for any x > 0, the func-
tion f (x) reaches a minimum at this critical point. This is the
only extremum (minimum) in the interval (0, oo). Hence, at
x = Vb/a the function f (x) attains the least value.
3.7.4. As a result of n measurements of an unknown quantity
x the numbers x lt x 2 , . . . ,
xn are obtained.
It is required to find at what value of x the sum of the squares
of the errors
f ( X) = (
X - Xl r + (x-x + 2 )*
. . .
+ (*-*B )
2
1' (
X) = 2 (x—x + 2(x—x )+...+2
t ) t
(x —x n ).
°n = *
/i
3
+200
3.7.6. Find the greatest and the least values of the following
functions on the indicated intervals:
6 -3148
162 Ch. III. Differential Calculus: Investigation of Fund's
(i)f{x)=
i
2^ 2 +|2
for -2<*<0; 0<*<2,
^ 1 for * = 0.
V = x 2 y = 32, (*)
S = x2 + \x —1
X }
=x + l
1
— X
.
2O = 2jc + 0,
whence
Fig. 42
= 2(10-*).
The area of the circular sector S = ^xy = x (10 x) (0<#<10).
The derivative S'(#) 10 = — 2x has a root x--=5.
Since the least value S = reached at the end-points of the
is
3.8.5. It is required
construct to
an open cylindrical reservoir of capa-
city V The thickness of the material
.
V= jx (x + df d + n [(x + df — x 2
] h = nd (x + df + nh (2xd + d a
). (
*)
f X) = x + m y a + (x—i)\ o<*</.
*
x = l-
j/~m 2 - 1
or
V 1
tri
tance in ohms.
Find the greatest effect which can be obtained at given E and R { .
(d) y=x+x l
'>;
y = ^r (*>°)
]J
(0 ;
§ 3.9. Convexity and Concavity. Points of Inflection 167
12* 2 + 6* — 36 = 12(V+-|- — 3) ,
whence y" = at x1 = —
2, x 2 = 3/2.
Hence, y" > on the intervals (—00, —2) and (3/2, 00); <
—
on the interval ( 2, 3/2). The sign of the second derivative deter-
mines the convexity or concavity of the curve in a given interval*
This enables us to compile the following table:
X x < — 2 ~2 x
3
< < 1
Sign of y" + +
Since the second derivative changes its sign when passing through
the points x 1 = —2
and * 2 3/2, the points ( 2, =
124) and — —
» are points of inflection.
=
1
l
i
+T x "<
2 /
f= 9-i77-
//
Xk = ^i/A + kn
9
& = 0, ±1, zh 2, ... .
it is convex.
(h) The given function can be written in the following way:
2— (x — b
1), x> 1,
y= 2+ — (a:
5
x< 1.
1),
Therefore
r _ \
— 5x\ *> 1,
y ~\ 5x\ x<l.
At the point x= 1 there is no derivative. Further,
/
—20a: 3 ,
x> 1,
20x\ x<l;
y" = at the point x = 0. Hence, we have to investigate three in-
tervals: (— oo, 0), (0, 1), (1, oo).
Compile a table of signs of y"\
Sign of y" +
e>ax 2
+ 3bx + c =
has different real roots, i.e. when the discriminant 9b 2 — 24ac > 0, or
3b 2
— 8ac > 0.
§ 3.9. Convexity and Concavity. Points of Inflection 169
y = x* + ax* + ^-x + 2
1
, _ — x — 2x+ * 1
y ' '
l)
2 '
r
2x :i
-\ 6x 2 — 6x—
y " (x*-f-l) 3
The second derivative becomes zero at three points, which are the
roots of the equation
x [i
+ 3x — 3x — = 0,2
1
whence
x x
-— 2 — [/ 3~, x2 - —2 + [/T, x, = 1
the relation
-2+/" 3 -1 (\+V 3)/4+l
3.9.5. Investigate the curves represented by the following equa-
tions for convexity (concavity) and locate the points of inflection:
(a) y= x- ^(x-3) 2
;
(b) y=e
x
(— jx/2 < x < jt/2).
s[n
§ 3.10. Asymptotes
A straight line is called an asymptote to the curve y f(x) if =
the distance from the variable point of the curve to the straight M
line approaches zero as the point recedes to infinity along some M
branch of the curve.
We will distinguish three kinds of asymptotes: vertical, horizon-
tal and inclined.
Vertical asymptotes. If at least one of the limits of the function
f (x) (at the point a on the right or on the left) is equal to infi-
nity, then the straight line x a \s a vertical asymptote. =
Horizontal asymptotes. If lim f(x) A, then the straight line =
X -> ± 00
X
lim
-* + cc
—=X
kv
*-» +
lim
oo
[f (x)
— k x]=b 1 A
Urn
— X
=k 2
and lim
—
[f (x)
— k x]=b 2 2
X -+ CO X -* 00
(a) #= -^3 ;
(b) y= ^r+3x; (c) y = -^;
#= 7 + 4,v 2
(d) ; (e) y-=xe^; (f) In
</=--f
§ 3. 10. Asymptotes 171
(i) y = 2Vtf + T.
Solution, (a) The curve has a vertical asymptote x = 3, since
5x
lim y = lim
,x — 3' oo
x^3T 0" x -+ 3 T- '
lim y = lim = 5.
* -* ± GO X -+ ±00 X 6
^A
a:
lim
-> I -
# =
a:
lim
-> 1 -
(-^13^==
\x ~~
— 00; 1
/
x-+
lim
1 +
# =
x-1+0
lim
\*~~
— -r
1
+ 3x = + /
I 00.
k= lim — == lim .
——.
r
3 =3;
b = lim (y—kx) =
X -*± GO
= lim
3a:
3* — 3x = 3 )
a; -> ± co
& =
X
lim 4"=
- ± CO * Xv
lim
-+. ~
± CO
,
^= g°=l;
b = lim
±
(a^ 1 /* — x) = lim
±V3
— l x
e l
.
—
:
lim
'
* -* CO X ->
l/x -
172 Ch. III. Differential Calculus: Investigation of Fund's
As x ^+ we have
lirn y = -
lim x\n[e
3x
K-*\/(3e) +
k = lira
X-*±cc X
b = lira [y — kx] = -
1r lirn x
X-+±cc
[n{l
-3x-e
lirn
x-*±<x> \ 3^ 2e
*,= [i m
>" l
+ ', + 2« - lim
Yl + 1 +2
= 3;
X
*^ + CO X-* + CO
Fig. 48 Fig. 49
k = lim —TT-t
X2
—= 1;
b= lim (
t4 x) =— 1.
x+cc V1+* J
-(*-!) =
l+JC
Hence, assuming
for all x> 100, we introduce an error of not more than 0.01.
^==
*2 -6*+ 3 = xarctanx;
(a) ;
(b) y
y=x+(smx)/x; y = In (4— x
2
(c) (d) )\
(a) y =x Q
— 3x* + 3x*— 5; (b) y--=\/~x— Vx+l;
(
c) y=-£zi*
-4 • (
d) y- -^ 1
0' = 6* 6
— I2x + = e>x(x* — 2x +
s
e>x
2
1) =§x{x 2 — l) 2 ;
*i =— 1, *2 = 0> x» = 1 •
y
" = 3(k 4
— 36* + 6 = 6 (5x — 6x +
2 4 2
1 ).
*, = 1/K5, *2 =1.
For convenience and pictorialness let us compile the following
table, where all the points of interest are arranged in an ascending
order:
X (I. <*>) 2
(f?0
1
(°-
7?)
y' + +
25 ^5
y" (5
-f -t-
y 23
3^/'(FH7 2
3
*i =— 1 ,
xb = 0.
-1
Fig. 50 Fig. 51
y
„_ J_/
~
___2 \ _J 1_ A) 1
3 V 3 J y# 3
1
X -l (--{) 2 K-°) (0, oo) 1
y' — 00 + 00 +
16
y" 00 00
1
9yj
1 ^ I
—0.26
y ~ (x 2— 4) 2 ~ (x — 2
4)
2
= 0, *=2]/~3«3.46
*
and becomes infinite at the point x = 2.
The second derivative
„_ 16a: (A:
3
-h 12)
y ~ (at
2 — 4) 3
r
(0, 2) 2 (2, 2/" 3 ) 2 |/"3 (2V 3", oo)
y' — oo +
3
y" +o 00 + 2
+
§ 3.11. Investigating Functions and Sketching Graphs 179
-A
V
4* l I
-1\
Fig. 52 Fig. 53
lim
- -
y = lim
- 1 -
[x + In (x — 2
= — oo;
1)]
1
^r
exists and is finite at all points of the domain of definition of the
function, only the zeros of the derivative
y _ — J/2) « — — J/2 +
( 1 (2 + 2^2) « — 0.84. 1 In
To plot the graph in the interval (l,oo), where there are no characte-
ristic points, we choose the following additional points:
*i
= ji
"g" » ^2 = ~Q~
5ji 3ji
*8 == ~2~
ji Jl 5ji 3ji
X 6~
T ~6~ AJ
2 (*• t) 2 (¥-) x4 (* 4 .2ji) 2ji
y' —2
y"
3 ]T3 3 ^ 3~
+
z 2
y l
lim y = lim jt
2
e /*
[
= Iirn ^- = oo ( t = —) ,
n
\t/2 B jc
2%
i*
-1
Fig. 54
From the information obtained we can sketch the graph as in Fig. 55.
To specify the graph in the intervals ( oo, 0) and (V 2 oo) the — ,
y = e^2J2.
(h) The function is defined and continuous
throughout the number scale, since at any x
1-f-x
2
< 1.
x
limy
-* + oo
= =
arc sin (-
D=Hf.
The first derivative
— 2x (\+x — 2x(\—x 2
)
2
)
4*
X (l+x 2 2
2 |*|
X (1+x 2
~ 2
(1-**)* ) )
(1+x 2 2
)
tice that
at the point x the =
right derivative is equal to 1,
—
and the left one to 1. +
The second derivative is posi-
tive:
2(l+x 2 )2x 8x
y x) —2 > for a
(
(l-fx 2 ) 4 (i +* 2 )3
Hence, in the interval (0, oo) the graph of the function is concave.
Also note that the curve intersects with the x-axls at the points
x =± 1.
§ 3.12. Algebr. and Transcendent. Equations
(e) y =y^-¥^=A-
(f) y = x*\n(x+2); (g) y =
\x arc tan— atx^O,
10 at x = 0.
*i (*) = *« (*)•
where the functions ^
(x) and i|; 2 (x) are simpler than the function
f(x). Sketch the graph of the functions y = ty 1 {x) and y = 2 (x) y\)
and find the desired roots as the abscissas of the points of inter-
section of these graphs.
The Methods of Approximating a Root.1. Method of Chords. If
the interval [a, b] contains the only real root £ of the equation
f(x)= and f (x) is continuous on the interval, then the first ap-
10 rw 2 1
/'w "' •
X"- Xn -*
r(*«-i)'
To estimate the absolute error in the nth approximation we can
apply the general formula
where
m = 1
min \f(x)\.
Under the above conditions the method of chords and the method
of tangents approximate the sought-for root from different sides.
Therefore, it is usual practice to take advantage of their combination,
i. e. to apply both methods simultaneously. In this case one can obtain
the most precise approximation of a root more rapidly and the cal-
culations can be checked. Generally speaking, the calculation of the
approximations x l9 x 2 , x n should be continued until the decimal
. . .
,
I \ Xn I I
X n-1 Xn |»
J
X X
— 00 1
~3 3 +
— 1
+ +- oo 4-
+
From this table we draw the conclusion that the equation has
three real roots lying in the intervals (—3, —1), (0, 1) and (1, 3).
f(x)==x + e* = 0.
the root £ lies in the interval (1.22, 1.23). The derivative f (x) =
= 4r* — 1 increases monotonically, therefore its least value in the
given interval is
m = 4xl.22
1
3 — 1 = 4x 1.816— 1 -6.264,
wherefrom we get an estimate of the error
I
x — < LZiZU =^^l « 0.00075 < 0.001.
1 1
x log X — = 1 0.
Here \f 1 (#)
= log a:, i|)
2 W=j- There are tables for the values of
/ (x) = x — 2x + 3x—5 =
s 2
^=1—-=^.
1
4
1=1.75.
Since
/(1.75) = — 0.5156 <0,
and / (2) = > 0,
1 then 1.75 <£< 2.
The second approximation:
Since / (1.835) =—
0.05059 0, then 1.835 g 2. < < <
The sequence of the approximations converges very slowly. Let
us try to narrow down the interval, taking into account that the
value of the function f (x) at the point x 2 1.835 is considerably =
less in absolute value than f (2). We have
^= 1.857-^-^=
QC7, / 7
1.857 , QC -
Q/1Qft
^= 1.8439-^^= 1.8438,
already gives the required accuracy. Here the sequence of the ap-
proximations converges much more rapidly than in the method of
chords, and in the third approximation we could obtain an accuracy
up to 10" 6 .
3.12.6. Find the least positive root of the equation tan x = x with
an accuracy up to 0.0001 applying Newton's method.
3.12.7. Find the real root of the equation 2 x logx=0 by — —
combining the method of chords with the method of tangents.
Solution. Rewrite the left member of the equation in the follo-
wing way:
f(x) = (2-x) + (- log*),
whence it is seen that the function f (x) is a sum of two monoto-
nically decreasing functions, and therefore it decreases itself. Con-
sequently, the given equation has a single root £.
Direct verification shows that this root lies in the interval (1, 2).
This interval can be narrowed still further:
1.6<£< 1.8,
188 Ch. 111. Differential Calculus: Investigation of Fund's
since
^ (1.6) = 0. 1959 > 0; / ( 1 .8) = — 0.0553 < 0.
Then
f W = _l_l lege; /*(*) = -! log*
and
/'(*)< 0; rW>0interval
over the whole interval
both the method
[1.6; 1 8].
Applying to this of chords and the
method of tangents with the initial point ,v =1.6 we obtain the
first approximations:
= 1 '
6 - (
V(i78
1 6)
l /(i
(1 )
= 1
6H 0. 1559= 1 .7559;
) 6)
x[=l.6—pffi = )
1.6 + 0.1540= 1.7540.
/(1 7540)
x'-l
X 2 ~
7540-
l./DW
-
(1.7540)
1.75557.
Since x 2 — = 0.00001,
x'2 the root £ is computed with an accuracy
up to 0.00001.
3.12.8. Using the combined method find all roots of the equation
f(x)==x 3 — ox + = 1 accurate to three decimal places.
3.12.9. Applying the iteration method find the real roots of the
equation x — sin x= 0.25accurate to three decimal places.
Solution. Represent the given equation in the form x 0.25 sin a:. — =
Using the graphical method, we find that the equation has one
real root which is approximately equal
to x = 1.2 (see Fig. 58).
Since
sin 1.1 >
=0.8912 1.1 — 0.25,
sin 1.3 = 0.9636 < 1.3-0.25,
the root £ lies in the interval (1.1, 13).
Let us rewrite the equation in the
form
x = cp (x) = sin x + 0.25.
Since the derivative q/ (x) =
cos x in the interval (1.1, 1.3) does
not exceed cos 1.1 0.46 < <
1 in absolute value, the iteration method
x= 1000 — a:
3
,
or in the form
1000 l_
X2 X 9
or in the form
(p (
X ) = Y 1000 — x,
<P'(*)= 3/
3 / (1000 -x) 2
|q>' (X) |
<C ~ 300
6W = 4-
3 y 990*
190 Ch. III. Differential Calculus: Investigation of Fund's
*o= 10,
x x
=V 1000— 10 = 9.96655,
x2 =V 1000— 9.96655 = 9.96666,
x3 = J/ 1000 — 9.96666 - 9.96667.
We may put £ =
9.9667 with an accuracy of 10~ 4 .
3.12.15. Applying the iteration method, find all roots of the equa-
tion \x —5 In x =
5 accurate to four decimal places.
f{x)=
\\/x if *>1
satisfy the conditions of the Lagrange theorem on the interval [0, 2]?
3.13.6. Prove that all roots of the derivative of the given poly-
nomial f(x) =
(x+ l)(x— \)(x— 2){x— 3) are real.
3.13.7. Find a mistake in the following reasoning.
The function
x 2 sin (l/x) for x =^=0,
f{x) = |
\ forx =
is differentiate for any x. By Lagrange's theorem
x 2 sin —x = x( 2| sin
V
—
I
cos \-
I
whence
cos 4- = 2£ sin \ — x sin— (0 < £ < x).
As x tends to zero E will also tend to zero. Passing to the limit,
we obtain limcos (1/g) = 0, whereas it is known that Tim cos (l/x) is
l -* x-+
non-existent.
(a) — —< In
a
J < —r-
-r- if < b < a,
192 Ch. III. Differential Calculus: Investigation of Fund's
(b) pyP~ l
(x—yX^xP — yP^pxP- {x—y) 1
if <y<x and p> 1.
are positive.
(*0» '0>
°(3) / (*o) = / (*l) =.-•=/ {*n) (X <X <...< *„),
1
lim — -p-:
e-*(cosx + sinx)
—— =
e-'2x (cos x -f 2 sin x)
x
,.
hm
_ ^
+
^~ x,.1- L 2 tan*
r -Manx
I
n
is non-existent, since the expression ^ 1 -j-
8"
tan x
is discontinuous at
the points x n = + =
nn n/2 (rc 0, 1, ...), but at the same time the
limit of the ratio of the derivatives does exist:
lim -— (cos^
——
\e~ 2x (cos x~\-2 sin x)\'
-
nr-=
+ sinx)]'
l;
lim — ——.— =
—5e~
-2e-*sinxs
2x s'mx 5
2 x
lim e -*
^ x
_„
= n0.
x^oo
Explain this seeming contradiction.
{ for x=
3.13.19. Prove the theorem if: (1) f (x) and (p (x) are continuous
in the interval [a, b] and differentiate inside it; (2) f(a) = y(a)\
and (3) f'(x)>y'(x) (a<x<b), then f(x)>y(x) (a<x<b).
3.13.20. Show that the function / (x) = ^+ has neither maxima,
at x= 0.
\
I a:
2
sin 2 (l/x) for x=£0,
f W
=
\0 for x =
194 Ch. III. Differential Calculus: Investigation of Fund's
f
I/* 2 (x>0),
\3* 2 (x<0)
has a minimum at the point #=^0, though its first derivative does
not change sign when passing through this point.
f(-i>=2,
3.13.32. For
1 if
r<— i)
what choice
— of the
i. /"(-i)=o, r'(*) >o.
parameter h does the "curve of
probabilities"
INDEFINITE INTEGRALS.
BASIC METHODS OF INTEGRATION
(1)
^
U »du = ^ +C
(2) ]^- = \n\u\ + C-
(3) aa dw-=^a* + C; J
e = e + C;
tt
du n
^
(4) ^ cos a da = sin u + C; sin^dw = — cosw + C;
J
J sin «
*
r
(9) f
V u1 ±a 2
=ln(tt + J/ i^±tf) + C;
J
(10) \
- — =— - n —— + C.
In all these formulas the variable u is either an independent
variable or a differentiable function of some variable. If
\f(u)du = F(u) +C 9
then
J/
(ax + b)dx = jF (ax + b) + C.
The method of expansion consists in expanding the integrand into
a linear combination of simpler functions and using the linearity
196 Ch. IV. Indefinite Integrals
J
\ 2=
1 1
a i fi (x)dx=^
i =
l
i
a i lf
J i
(x)dx f
\{=
S i
|fl/|>0).
/
dx.
4.1.1. /
J V x
Solution.
x2+
j = ^i~ 1
=j 3
(jc / 2 + 5*"', — x - v.) dx =
j
= }x z
i>dx + b^x i>dx—^x- l l
i*dx==
= 2^(^ + |-l) + C.
Note. There is no need to introduce an arbitrary constant after
calculating each integral (as is done in the above example). By com-
bining all arbitrary constants we get a single arbitrary constant,
denoted by letter C, which is added to the final answer.
a * o 1 P6*3 + * 2 — 2*+l ,
4.1.3. /= f
2 ,
2
.
J sin * cos *
Solution. Transform the integrand in the following way:
1 sin 2 *-)- cos 2 * 1 1
Hence,
/ = f-^r-
2 + J[ -£t- = tan x — cot x + C.
J cos * sin x
1
2,
4.1.4. I = ^tan 2
xdx.
Solution. Since tan 2 x = sec 2
x — 1, then
/ = f tan 2 xdx= [— ^|
cos *
I dx = tanx — + C. jc
t/ J J
4.1.5. / =J (x 2 + 5) 3
dx.
Solution. Expanding the integrand by the binomial formula,
we find
/= j(x 6 + 15jc
4
+75a; 2 + 125)dx = y- + -^ + ^+ 125x + C.
4.1.6. / = J (3x + 5) 17
dx.
§ 4.1. Direct Integration and the Method of Expansion 197
we get
_ l (3*+5)» .
r
i
is
rc-
4.1.7. /= r ^
4.1.8. / = J
cos(jtx +
Solution. Proceeding from the tabular integral (4)
J
cos w du = sin w + C,
we obtain
/ = J-sin (nx + 1) + C.
4.1.9. / = J
cos 4a; cos 7a: d*.
and therefore
cosmx cosnx =
Y [cos (m — n)x + cos (m + n) x].
4.1.10. / = J
cos a; cos 2x cos 5a: dA;.
Solution. We have
Thus,
4.1.11. / = $sin 2
3A:dA:.
/ =Y l c
\ (1
11
— cos6A:)dA; =-ja: — ^sin6^: + C
4.1.12. / = ^cosh 2
(8x+5)dx.
dx dx
Solution. I = ]C x + 4x+5 = ^C
*
= arc tan (* + 2 + C )
d*
4.1.16. / =j
V4— 9x 2 '
4-1.19. / =
|4
_^_.
Solution.
1 — { dx C ch
<ta
=TT7^1n
1
t
2 j/l + AT+2
J 4 — — 4x
2
J 8— t+2) 2
(a;
4^2 2 /*2 — (* + 2) + C.
§ 4.2. Integration by Substitution 199
, , P3 — 2 cot 2 * , C 2 + 3x 2 .
(
C
M cos** <
d>
JiM?^^
4.1.22. Integrate:
f — at^.h-
f cos 2*
^
(5)
^ ' \
y \ —x*
9
J cos*— sin* *
(c) \
^ dx; (d) \ (sin 5*— sin 5a) dx.
lf(x)dx=lf[<p(t))<p'(t)dt,
Whence
a:— 5=t\ x= t
2
+ 5, dx=2tdt.
Substituting into the integral we get
/=J(/»
+ 5)*.2* dt=2 j(/ + 5* 4 2
) df =2-£ + ^+ C.
2(*-5) 6/ »
3/
10(*-5) »
^
' = '
5
,
3
,
4.2.2. / = j- +d*e*
1
'
dt
J \+e x Jn(/-I)'
<
But
1 1 l_
t(t — \)~t—\ t
y
^ -^
therefore
I = r = ln\t-l\-\n\t\ + C.
Coming back to the variable x, we obtain
I^ln^ + C^x-lnil+e^ + C.
Note. This integral can be calculated in a simpler way by mul-
tiplying both the numerator and denominator by e~ x :
= -ln^-tl = x-ln(^+l) + C.
4.2.3. /= \
J
/2+3
V&x--5)3
dx.
(x2 - x)dx
4.2.4. /=r
4
+ 3x 2 +l)arctan^-t-J
(\ — \/x*)dx
+ l/x) 2
+l]arctan(.*:+l/;tr
(l-±)dx = dt.
Whence
d
/-f ± '
J (/
2
+l)arctarW
Make one more substitution: arc tan t = u. Then
dt
2 +\' du
t
and
/ = j^=ln|«| + C.
§ 4.2. Integration by Substitution 201
Returning first to t t
and then to x, we have
/ = In |
arc tan t\ + C = In arc tan (* + 7) + c# |
4.2.5. / = I
V a '~ x2 rlr
x*
x= T 1
\ dx =
,
— -^ dt
Hence,
^ 2 (3 / ~~ r "
2
J a sin
2
x+b 2 cos 2 a:
Solution.
dx
-
\ a 2 sin 2 A; +& 2
cos 2 a: b2 I a?2 ,
tan 2
+ .
, cos 2 a:
"
J J b 2
a: 1
dx
Make the substitution ^-tanx
&
= t; dt = -%-
6 cos 2
9
a:
. Then
4.2.8. / = j"
cos X
202 Ch. /V. Indefinite Integrals
4.2.9. f ^-yr .
b
J (arc cos x) y \ — x' 1
dx
Solution. Make the substitution: arccosx^/; r = dt. Then
1^ l - X*
/ =— [
J/ 5 J
Cr 5
d/-4-^"
4
4
+ ^
—
~ c = 4 arc cos rx + c.
.10. f ^^±L=dx.
sin 2x
4.2.11. / = j_5!^L<fr.
+ sin x^1
2
In x) x
-
In x
(c)
J
5
cos x V^sin .xrck; (d)
^
y* Ax -
4.3.1. / = J
arc tan x dx.
Solution. Let us put here
;
V==X)
4.3.3. / = J
xcosxdx.
Solution. Let us put
u = x\ dv = cos a; <£*;,
whence
du = dx\ v = s'mx i
1= ^ A;cosA:dA:
= Arsin a:
— J sinA:dA:== Arsin x-fcosx + C.
// ^ cos x] dv = x dx,
whence
du — — sin a: dx; a =y a;
2
.
In this case
/ ~ -1 cos x +yJX 2
sin a: <2a;.
4.3.4. /= J
a:
3
lnA:dA:.
204 Ch. IV. Indefinite Integrals
u = In x\ dv =x 3
dx,
whence
dx
du
i
=—X
;
'
v = — .,\ 1
/ = jc
4
In x — 4" J ^ ^ T — -j^x^dx = ^-x*\nx — -^x + C. nA: 4
4.3.5. 7 = 5
— 2* + 5)<T*dx.
(a;
2
w =x 2 — 2x + 5; dv = e~ x dx y
whence
= (2x — 2)d;c; u = — e~ x \
1 =^ (x 2 —2x+5)e- x dx = — e- x {x — 2x + 5) + 2 2
J
(x—\)e~ x dx.
x— = l u\ dv = e~ x dx>
whence
du = dx; v = — e~ x .
71 =2 J
(jc— \)e~ x dx=— 2e- x (x— l) + 2 J
e-*dx= — 2*g-* + C.
Finally we get
7 = — e-*(* 2
— + 5) — 2xe-* + C = — e~
2a;
x (x 2
+ 5) + C.
Note. a result of calculation of integrals of the form ^ P (x) e ax dx
As
we obtain a function of the form Q(x)e ax where Q(x) is a poly- ,
1= J
(3x 3 —\7)e 2x dx.
Solution.
J
(3a:
3
— 17) e 2x dx= (Ax 3 + Bx2 + Dx + E) e 2x + C.
Differentiating the right and the left sides, we obtain
(3a;
3 — 1 7) e
2x = 2 (Ax + Bx + Dx + E) e2x + e2x (3 Ax + 2Bx + D).
3 2 2
Cancelling e 2x , we have
3x — 17 = 2 Ax + (2B + 3 A) x + (2D + 2B) x+(2E + D).
3 3 2
§ 4.3. Integration by Parts 205
3= 2A\ = 2B + 3A;
0-2D + 2B; — 17=2£ + D.
Solving the system, we obtain
A— — -
R— ~- D—— '
F — —
Hence,
(3x 3 — 1 7)
a
*dx - (-| a; 3 — ~ + -J x — a;
2 2
) e * + C.
j £?
^
4.3.7. Integrate:
/ =J (*
3
+ \)cosxdx.
Solution. Let us put
a = a;
3
+ 1 ; do = cos x dx,
whence
da = 3jc 2
(iA;; o = sinA:.
/ = (^3 +1) sin a: —3 ^ a;
2
sin x dx= (x 3
+ 1) sin x — 3/ lf
where lx = ^
2
x sin x dx.
Integrating by parts again, we get
/x =— a;
2
cos x + 2/ 2,
4.3.8. /= J
(a:
2
+ 3a: + 5) cos 2a: g!a:.
jj
(a:
2
+ 3a:+ 5) cos 2a:g!a: =
= (A x 2
+ A x+ A x 2)
cos 2a: + (B a:
2
+B x+B x 2)
sin 2x + C.
206 Ch. IV. Indefinite Integrals
+ 3x + 5) cos 2x = —2 (A x + A x + A sin 2x +
(x2 2
x 2)
2B = 1; 2(B 1 + A ) = 3; A + 2B = 5; 2
—2A = 0; 2 (B — A,) = 0; B
1
x
— 2A = 2
0.
'
/I
i
__J_.
— 2 '
r
i
_
— A2 •
'
/i
2
_
— A4 •
'
R
2
—A
— 4
•
Thus,
J
(*
2
+ 3x + 5)cos2xdx= ( y + t) cos2jc+ (y * + jx + ~) sin 2 2
x+C.
4.3.9. /= J
(3a:
2
+ 6a: + 5) arc tan a: dA:.
whence
^w ^ I
"»
o = + 3a: + 5
a:
3 2
a:.
Hence,
/ = (*» + 3a: + 5x) arc tan x— ^"^ 3 5* dx.
2
j ff
Single out the integral part under the last integral by dividing
the numerator by the denominator:
=J + 3x + 2j^-3.J I ^ 1 =f + 3*+21n(x
2
+l)-3arctanA'+C.
/ = ^e bx cosAxdx.
Solution. Let us put
e bx = ^; cos 4.v =. do,
§ 4.3. integration by Parts 207
whence
5e bx dx = du\ v = sin 4a:.
Hence,
/ — —4 e 1
5*
sin \x — 5 P
\ e 5 *sin4A:dA:.
I 1 z=
r*
j
sin Axdx^ — j
5
-^-e *cos4.*; + ^-
5 P
\
5
e *cos4A:dA:.
Thus,
/ = -j e bx sin 4a:
1
— —5/1
—j ( e 5 * cos \x + 5 P
\ e bx cos 4xdx)
\
9
l. e.
£/ = cos(lnA:); dv = dx y
whence
= — sin (In x) ^ ; ^ = a:.
Hence,
/ = J
cos (In A:)rfx = xcos(lnA:) + ^ sin (In a:) dx.
Thus
/ = cos (In a:) dv = a: cos (In a;) + x sin (In x) — /.
Hence
/ =y [cos (In a:) -f- sin (In a:)] + C.
208 Ch. IV. Indefinite Integrals
In
(
1 + 1 ) = In i±l = in (x + - In x. 1)
Hence
/ = J
x In (x+ \)dx — \^x\nxdx = — Ii /2
Hence
j
X —
2
In X +l)-±§(x-\)dx = £ (x+l)-i*« + ±x + C.
1
2 (
T±ln
Analogously,
Finally we have
Then
2dx dx
<« = -V or
? ?
I j,
<ft.
Hence,
u = \nt; dv = \/"tdt.
Then
du = Y> v =-~t\ft
§ 4.3. Integration by Parts 209
Whence
-1 Jj/Tln*<tf=— -g-
\jtVT\nt— =
Returning to x, we obtain
'—
(^+i)
9x 3
1
[2—3 In
(1+33)] +C
4.3.14. / = sinx In tanxdx.
4.3.15. / = J \n{VT^x+VT+lc)dx.
Solution. Let us put
u = In + /l+x) ; dv = dx,
whence
~~
2 '
j/t^*+ i^tt* '
]/T=i2~~ 2 '
* ^r=^
= a;.
Hence,
/ = x in {VT=Tx + VTTi) - \
j
x^^^dx =
^u (x) v (x) dx = u (x) v (x) — u' (x) v (x) + u" (x) v (x) — ...
l 2 9
. .
+ (— l)"- ^"- (x) v n (x) — (— l)"" J u (x) vn (x) dx,
.
1 1
*
1 {n)
where
v1 (x)=^v (x) dx; v 2 (x) =^v x
(x) dx; . .
. ;
v n (x) = J vn _ x
(x) dx.
J
P n (x) dx = P n (xf£ - P'n (x) ~+
e
. . .
+
+ (-!)» PJ» {x)£t + C =
(b) (2x + 3x — 8x +
3
V2x + 6 dx.
2
1 )
J
Solution.
(a )
^( x
3
—2x + 3x—l) cos 2x dx = (x
2 3
— 2x + 3x — 2^ 2
1 )
— {3x — 4x + 3)
/o 9
2 /« i o\ f
^
cos
^
2a: \ /c
j+(€>x,
— 4)l^
/ sin 2x\
—J — c6—cosj^— + Cn =
2a: .
4.3.17.
J
In (x + Vl+x*)dx.
4.3.18.
J
Yx{\nxfdx.
arc s ' n ****
4 3 19
J
x cos # dx
4.3.20.
sin 3 a:
4.3.23. ^(l+x2 )
2
cosxdx.
(a) ^(3x 2 2
(3* + \)dx\ (b) dx.
^
I
(c) /„= J
(a
2 —x 2
)
n
dx.
dv = dx,
(*
a
+ a 2\na
)
whence
du
,
= — r(x^-
2n x dx
—
-^a n + 1 i
2 ir
,
9
v = x.
)
Hence,
/
(x 1 + a'
1
)
n
whence
n+ ~~ lna h
l
a1 n
1 '
{x 1 -f- a 1 )' 1
2n '
212 Ch. IV. Indefinite Integrals
/, = f 2 — 2
—
+ a = a arc tan a + C.
J x
'
+ a + 2a? ai C ¥+C
*
~~ 2a 2 ~~ 2a 2 x* T an
2
J (** +a 2 2
)
'
a:
2
+a 2
2a 2 1 2
putting m = 2, we get
/ —
~~
C
dx —
~~
1 a: 3 ,
7 2 ~~~
_
3
J (*
2
+a 2 3
)
4a 2 "
(x 2 +a 2 2
)
4a 2
x x 3 ^
~~
1
4a 2" (x 2 +a 2 2
)
+ 8a* '
x^fa~2 + 8^.
w=sin'«-i1 x;
•
z
ay
j
= cos —a
sin *
—
* OT
,
a:,'
whence
du
da = (n — 1) sin" -2 x cos
v 7
a: d^:; v = (m— ,
1X
1)
1
m -1 #
cos™
. (m
v ^ 1)
Hence,
/ _ sin" -1 * n — 1 C sin"
si
-2 x dx
~~
(m — 1 ) cos"2
-1
x ~ m— 1
J ~ccos 7™
- #
_ sin"- 1 * — 1
m ^=1
(m— 1) cos* 1
-1
* m— 1 "" 2 a-*' ( /-
j n=zX (a* — x + 2n $ x
2
)
— x )"- dx =
n 2
(tf
2 1
= x(a —x + 2n —a + a
2 2
)
n
J
(a:
2 2 2
) (a 2 — a;
2 )"" 1
dx =
= a: (a 2
—a; 2
)" — 2n/ + 2na I w
2
n_v
Hence,
j __ x(a
2 —
x2 ) n 2na 2 ,
I- 1/2
_
= P
\ ^
ax
dx
r -n » " arC sin ~x + Cn .
>
(c) /„ = J
xn e x dx = x? ex —nl n -
l
1 ;
J V x*-\-a
C h a p t e r
BASIC CLASSES
OF INTEGRABLE FUNCTIONS
Q {x) = {x—a) k {x — b) 1
. . . (x
2
+ ax+$) r
(x
2
+ yx + \i)
s
.
P(x) Ax A2 Ak ~
~ ~ (x—a)*
. . .
Q (x) x—a 1
x— b 1
(x—b)* 1
* " *
M*— b) 1
1
* '
#
x2 + ax + p (*
2
+ cu;+p) 2 (jca + ouc + P)' 1
*a + V^+M- (*
2
+ Y*+M0 2
(*
a
+ Y*+H) 5
where
i4 lf i4„ . .
. ,
B 19 B„ . .
. ,
M N lf 19 7W 2 ,
N 2y . . . ,
/? lf L lf /? 2 , L2 ,
. . .
5.1.1.
, f* \5x 2* — 4x — oi
81
<ix
+ 4) (*-l)
c
a+4 1
x— 1»
where A, B, D are the coefficients to be determined. Reducing the
fractions to a common denominator and then rejecting it, we obtain
the identity
= 31n|x — 3 + 5 In |
|
x + 4 + 7 In |*— + C =
|
1
|
= In |
(x—3) 3
(x +• 4)
fi
(x— I)
7
1 -f C.
we obtain
x* — 3x — 3*— 2
2
,
t
x +2
x — x — 2x
3 2 1
x(a: 2 — x — 2)"
Hence,
, rx* — 3x*—3x—2, p, .
1W p (x + 2)d*
x3 — 2x 2
+ x=x(x—l) 2
.
2x 2
— 3* + 3 == A (x— l)
2
+ Bx+Dx(x—l) =
^(A + D)x + (—2 A — D + B)x + A.
2
(*)
Thus,
5.1.6.
Hence,
x = A(x 2
— x + + (Bx + D) (x+l) =
l)
A=
-T> fi = T' D = T-
Thus,
j t * ^ x tdt ,
3 V d/
lln(/*+4) + K3 arctan^L + C.
Returning to x, we obtain
/1 = jln(x 2
— x+l) + K3" arctan^i + C.
218 Ch. V. Basic Classes of Integrable Functions
Thus,
= — y ln|* + 1
| + -g- ln(x 2 — *+l) + X^-arctan -y~ 2
+C.
hence
l=(Ax + B) {x 2
+ 4) + (Dx+E) (x + 2
1).
x + 4x +llx +12x-f-8
' 4 3 2
(* + 2x + 3) (x+l)
2 2
J
Solution, Here we already have multiple complex roots. Expand
the fraction into partial fractions:
^4 + 4a: 3_|_ 11 ^2 + 1 2^ +8_ Ax+B Dx + E f_
(x 2 + 2x + 3) 2
(x + 1 )
*~ (a;
2
+ 2x + 3) 2 '
x2 + 2* + 3 + * + 1
'
-4=1; B = —U D = 0; £ = 0; f=L
Hence,
^4 + 4a: +1U +12a; + 8-dx-
3 2
(jt
2
+ 2x + 3) (x+\) 2
§ 5.2. Integration of Certain Irrational Expressions 219
=
Calculate /,
j (
,2 / ^
= (x+
2 3)2
dx.
Since x2 + 2x + 3 l)
2
+ 2, let us make the substitution
x-\- 1 Then we obtain
I i=\ dt = dt ~2 I V = ~~ 2/ 2'
(/2 +2 )
2
^ 2
+ 2) 2
2 (/a
+ 2)
The integral
2 2 2
J(/ +2)
.
_ l / l r dt _ l t l , t
r
Thus
^r
*
/ } L_ ! o rn for,
i-
i
7
2
2(/ +2) 2(/>+2) 2}A2 f2
Returning to x, we obtain
/ £±J !_arctan^tl-J C
l
i~ + 2a;+3)
!
+ 2a;+3)
2(x 2 2(a: 2 2 V2 ^2
We finally obtain
2
= ln|x + 11— 0/ tan^y + C.
YU^2
o*,"^ arc
2 + + 3) 1 1
(a:
2
2a:
i ox
2 V^2
Find the following integrals:
5#1 10 . .
j
^"^^ — 22a; — 8 ^>
5x 3 -j-9x 2
3 r;;
-4a
—
dx
5.1.11.
(jc+1)(a:+2) 2 (jc + 3)»'
5.1.12.
4a:+4) (a:
2 — 4a: +5)
dx
5.1.13.
(1+*)(1+* > )(1+* 8 )'
x +3 3
5 ' U4 IcT+W+T)^'
'
x = t*, dx^Wdt,
whence
= 6p 3
d/ + 6 J^^7 = y/ + 6arctan/ + C. 4
Returning to x, we obtain
2
# 3
+6 arc tan 1/ x+C.
5.2.3. / = t j
(2x —
3
3) +1
Solution. The integrand is a rational function of £/2x — 3, the-
refore we put 3 t
Q
2x— = y
whence
—
dx = 3t b dt; (2x — 3) _L
T= t
3
\
(2x— 3) 3 =/ 2 .
Hence,
1+/ 2
«3-y~ 3-J- + 3-^ — 3^ + 3 arc tan + C. /
§ 5.2. Integration of Certain Irrational Expressions 221
Returning to x, we get
-6 — - -
y (2x— 3) —
11
/ =3 I
-J-
(2x— 3) -i-
l
(2x— 3) 6
l
2
5.2.4. /= f- 3
/•"
1
K 2 +x 2 +x '
whence
Hence
j
7 ~~
P 2(l+/ 3 )
2 />12/ 2 ,
ai ___3_
~~ " r
2j^""4/ 2 t U#
A " i
J 16/
6
(1 + '
3 2
)
Returning to x 9
we get
'=4K(W+ C -
5.2.6. /= (*__=£_.
Solution. Since
jZTj
-
; therefore let
x-\
~ l
K %-l 9
whence
_ tj—
a:—
/
4
+2 .
a: — —_ 1
1
, 4
3
,*
.
#-f-z
i _
— 3/4 .
, _
—
-12/ a ^
(/4 1)2
222 Ch. V. Basic Classes of Integrable Functions
Hence,
~ _ Jp
(/4-l)(/4— 1) \2t*dt 4 C dt__±_i_ r
l
3.3^(^ 4 -D 2 "~ 3 J /2-3/ + Lt
Returning to x> we obtain
dx
5.2.7. f
J (1- x) V 1— * 2
d*
5.2.8
J 7^=
t+1) 2
(x — l)
4
5.2.9.
j (,-2) /}±£xx dx.
ax 2
+ bx + c = a(x — a) (x— f$),
2
i.e. if a is a real root of the trinomial ax -\-bx-\-c.
5.3.1. /=f / x
Vx + 2x-\-2 = — x. 2
t
Squaring both sides of this equality and reducing the similar terms,
we get
2x + 2tx = — 2, t
2
whence
_ t
2 —2 ,
_ + 2/ + 2
/
2
*~~2(1 +/) ;
2(l + 2 '
1
+ ^+ 2*
+ 2 = + <-^=%T7F 1 *
J «
2
+ 4i + 4)2(l+0 2
J (l+0(<+2) 3
§ 5.3. Euler's Substitutions 223
Now let us expand the obtained proper rational fraction into par-
tial fractions:
/
2
+ 2/ + 2 _ A B D
(t + 1 ) + 2) ~~ + + + 2 + + 2)
(/
2
t I / (t
2 '
Hence,
p r2 + 2/ -f- Ai _ p d/ Q p d/ _ 1 i y i iii 2 , ^>
Returning to x, we get
2
/ = In (x + + j/> + 2x + 2) H x
1
r + C. _
+ 2+ y x* + 2x + 2
5.3.2. /= f *
c+ V * 2 — x-\- 1
Vx 2
— x+ 1 = tx— 1,
whence
(2/-l)x = (/
2
-l)x 2
; * = ^rr;
^=-2^^^; x + Kx'-x+^y^y.
Substituting into /, we obtain an integral of a rational fraction:
r — 2; + 2/— 2
J /(/ — !)(/ + I)
2 '
-2/ 2 + 2/ — 2 i4 . B . D . E
t(t—\)(t+\) 2 t
1
/ — 1
1
(t + \)
2 1
* +
By the method of undetermined coefficients we find
where / =
)/"x a — x+l + l
x
d*
5.3.3. /=f (1+x) ^l + x— a;
2
224 Ch. V. Basic Clastes of Integrable Functions
x dx
5.3.4. / = f-
(]ffx—\0 — x*) 3 '
Solution. In this case a and c therefore neither the first, < <
nor the second Euler substitution is applicable. But the quadratic
trinomial 7x 10 —
x 2 has real roots a 2, P 5, therefore we = =
use the third Euler substitution:
Whence
5— 2) t
%
\
_ +
X—
5 2/ 2
ax _
6/ dt
™
,
+ .
1
i v2
/
2 »
'
(1 + /
2 2
)
•
Hence
= + 2)*--»(-f + a
,
.«^--»fl| ) +C .
where /-
. — — 1/"7jc
^ 10 jc
2
.
Calculate the following integrals with the aid of one of the Euler
substitutions:
dx
5.3.5. f j±2
J x— Vx + 2jc + 4
dx
5.3.6.
J Y — — 1 AT
2
1
dx
5.3.7.
j ]/"(2x— 2 3
)
5.3.8.
f
^+.GI?"
yT+x
^ 2
J
R(x, Vax 1
+ bx + c) dx,
simpler methods are used.
I. Integrals of the form
*J Vat'+K *J Vat* + K
where M N K
l9 ly
are new coefficients.
The first reduced to the integral of a power function,
integral is
where
L
J
p
KaA: 2
P m _ 1 (x) is
:?Tl-
+ ^+c =v
a
1
polynomial
(x)^Hw/([-7
of degree m
J
—
^ V ax
1,
z
and
+r
K
=-,
bx + c
is
(i)
some
constant number.
The coefficients of the polynomial P m (x) and the constant ^.
1
x-a = T
1
.
x
5.4.1. 7 =f .
J V 4a:
2
+ 4*—
Solution. Make the substitution 2#+l = /, whence
/ — 1
— -^dt.
, dx
2
Hence,
1
4
CiLh5)dt_ 1 — + 14ln|/ + ^
^-4
./-tj 2 i 2
-4| + C.
J
Returning to x, we get
5X
5.4.2. /= f +- H*.
J ^x + 2x+5
2
/ = (
A x> + Bx + D) Vx 2 + 2x + 2 + K f -= dx
j y * 2 + 2* +
Differentiating this equality, we obtain
/'
— x— l
r
V x2 + +2 2x
= (2 Ax + B) Vr x* + 2x + 2 + (Ax + Bx + D) 2
4
y x' ~\~ 2x ~ j 2
*
+ Vx' #
1
+ 2x+2
Reduce to a common denominator and equate the numerators
2A + A=1, B + 4A + B + A=0;
2B + 4A + D + B = —l\ 2B + D + K = — 1.
Thus,
dx
*
+ 2x + 2
where
1
J Vx 2 + 2x+2 J 1)2+1
v
^ -r /-r
5.4.4. /= JjAx 2
— 4x + 3dA:.
Solution. Transform the integral to the form
V2 4f J V(2x-\\> + 2
= )
]/4x 2 — 4x+3 + y In (2x — 1 -|- j/4j^ — 4x+3) + C.
9' 3 -3' 2 +2 -
5.4.5. f rf
J K3x 2 — 2x+l
5.4.6. 1 dx.
(*+4)dx
5.4.7. =
/- r
1 (f±
J (a:— 1) (a:+2) 2 +
Solution. Represent the given integral as follows:
(x-{-4)dx C x+4 dx
(a:+2) 2
(x-l)(x + 2) 2 j7> + jc+1 J
(at— 1)
yx + x+{
2
x+4 A . B . D
(x— l)(* + 2) 2
x—\ 1
(x + 2) 2 1
x +2
Find the coefficients
Hence,
5 5 dx
1 = 2 1
dx 2 f dx
(x— 1) + 1
3
J (x+2) 2 |A^-f-x-fl
dx
_ "
5_ f
9
J + 2) /x + x-H 2
f*
3 - 6**+H*- 6 ^
5.4.8.
J W+4x+3
f 3^ + 5^-7,+ 9 Wt ,
5.4.9.
J |^2a; 2 + 5a: +7
228 Ch. V. Basic Classes of Integrable Functions
1
xdx
5.4.11
I (x 2 — 3x + 2) Vx 2 — 4* + 3
r dx
5.4.12. "
J (^+l) 3
/^ + 37+2
{ l)dx
x Vf-
5.4.13. r .
J l+3x + 2
;c
4
and n.
Case II. is an integer. We put a + bx n = t
a
, where a is
5.5.1. / = $
j/x(2+j/"x) 2 dx.
\2+x-V
= r -(
3 2
Here = 2, an
Solution. I
J
x y dx. /? i.e. integer; hence,
we have Case I.
C-/T+V*
5.5.3. / = \
*
, J_ dx.
J
Solution. I =
J
( 1 + xT ) T d.*:.
m+l = (-4+0 =
Here m = — j2 .
; n = j\ 1
p = -^ 1
; 1,
{
i.e. an
3"
integer.
§ 5.5. Integration of a Binomial Differential 229
l+x* =f« ;
±x 3
dx = 2tdt.
Hence,
/ =6§ t
2
dt = 2t + C = 2(1 +xT ) T + C.
3
= T ) Tdx.
J*~(2 +
5.5.4. / JK
5.5.5. [ = i
\
x b {\ +x 2 )~ dx.
j
5.5.6. / = jV 11
(\ + x*)~dx.
= — -j a fraction, = + = — also
Solution. Here p y is
tdt
x— dx — 1 ,
— 2(f — 2 4 2 4
(t l) I)
= _^ ^_i)M/=-^+|-4+c.
(
Returning to x, we get
3 r~
5.5.7. \ jj- dx.
dx.
5.5.8. j-
2
5.5.9. (!+**) dx.
J*»
230 Ch. V . Basic Classes of Inlegrable Functions
5.5.11. \Vx Y^ + V x * dx -
/=$/*(l— /*) 2
dt 9
t = smx
and are, therefore, integrated in elementary functions only in the
following three cases:
If m
an odd number, the substitution cosa;=/ is applied.
is
If the sum m + n is an even number, use the substitution
tan#=/ (or cot^=/).
In particular, this kind of substitution is convenient for integrals
of the form
^tan n xdx (or ^cot n xdx^>
=3 j/cos a:
(
y cos 2
jc— 1 ) +C .
c
5.6.2. / -^d*.
=fJ sin x 6
J J
The second of the obtained integrals is calculated by the substitu-
tion:
/, = gg Jsin 4
2a: cos 2x dx t* dt ==^ + C ==^sm* 2x + Q.
We again apply to the first integral the method of reducing the
power:
J_ x — y sin 4#j
128 +2§6 J + cos 8a:)c(a: =
:
/__L *^ —
256
^
256
sin
0111
™+
^ 2048 sin
4a: ^
^" + ^
" 320 sin 8a: 1
5
2a: + 0.
5.6.4. /
r
= \ —
2
r sin x
g-dA:.
6
J cos *
«
Solution. Here both m and n are even numbers, but one of them
is negative. Therefore, we put
tan a:= /; —^— = +
cos 2 a;
1
1
1
2
; T~ti==dt.
cos 2 a;
Hence,
232 Ch. V. Basic Classes of Integrable Functions
Q
5.6.5. /= [ ^dx.
2
J sin x
Solution. Here we can put cotx=t, but it is simpler to integrate
by expansion:
sin 2 * 2 1
J J Vsin x J
—r + -j\
3x\
sin 2x ~
+ C.
-
.
,
d*
5.6.6. / = cos 4
J a:'
5.6.7. /
-J jj/sin 11 x cos a:
fore we put
dx
tan x=t; ?
= dt.
4 11 11
J cos * j/tan * J j//
3 (1 + 4 tan 2
x)
•C.
2
8 tan a: ^/tan^Sc
Solution.
\ tanxdx= \ dx = — In |
cos x -)- C;
J COS
|
J AT
\ cot x dx — \
sin
dx = In |
sin x |
+ C.
J <J
at
/ /5
= -' , , =
=I TT7-. K' + '-TTT.)*
5.6.11. / = C£2^fdjc.
sin * 3
J
/
4
-dt.
Then
du = 3t 2 dt\ v — 2(1— 1
2
)
Hence,
/ = t* r t
2
dt _
2(1— 2
)
t* i + i
dt:
2(1— P) 1
2 J 1— t*
t
3
3,3.
+ ln I _±i l
"2(1 — 2
) 2
1
4 |
\—t
cos x 3
3 + cos * 1
-cosx + T ln — I
+c.
5.6.12. /= f
J COS*
—
dx.
2 COS X 1
sin x
2t
cos x-
l — t
2
IT/*' 1 + /
2 '
2dt
x = 2arc tanf; dx =
i+t 2
'
234 Ch. V. Basic Classes of Integrable Functions
or cot x= t.
5.6.13. /= —
/0
J sin x (2+ cos
.
dx
x—2sj^— r
sin x)
.
2dt
t /
A
=b B =ih D =- 1 -
Hence
Jt_ d£ f dt C dt
J ~ 3 J
f
t +
,
_5_
3 J t-3 J
T + |-ln|tan|— 3| — In
X
T ln |
tan tan-
2
+ C.
§ 5.6. Integration of Trigonom. and Hyperbolic Funcfs 235
dx
5.6.14. /:
5+ sin x + 3 cos x
dx
5.6.15. /:
sin a; (2 cos 2 £— 1) *
1
Solution. If in the expression
cos 2 x — we substitute
sin x (2 1)
—sin x for sinx, then the fraction will change its sign. Hence, we
take advantage of the substitution / cosa;; dt sinxdx. This = =—
gives
dl
J (
2/2_ 1}
'
Since
1 (2 — 2/ — (1— 2/
2
)
2
) 2 1
then
— +c=
dt dt
In In
1— 2/ 2 1-/1^2 2 l t
i+ VT COS JC ,
1 , II — COS X
1-^2 "•"T + COS X I
1
= -£=ln 1+^2
1
— cos #
+ ln tan
I
1 Y^2
5.6.16. /
_ C sin 2 # cos #
dx.
J sin #:+ cos #
Solution. Since the integrand does not change sign when sin*
and cos x do change their signs, we take advantage of the substi-
tution
t = tan*; dt = COSdx X
"
2
'
Hence,
7r
_ C tan 2 X cos 4 x m
dx _ C t
2
dt
/* A Bt +D Et +F
(/+l)(/a + 1)
2-
t+\ ^ t
2
+\ M/ 2 +l) 2 *
B= — 1
D-- =
1-
2 '
F=-± 2
Hence,
<— 1
/ »
4
In -J±L-4
^ + 1 ^2 4
•
^+
1 + /
2
C=
= In I
sin a; 4- cos x |
— -|- cos x(smx + cos + C.
2 tan ^ + 3
5.6.17. /= f sin
t
+2 cos si 2 2
^
J a: a:
(2 tan x + 3)
^_
.
r 2tan*+3 C cos 2 x
'
J sin
2
x+ 2 cos 2
a:
j tan 2 x+2
5.6.18. /= [j^-dx.
J + sin x 1
Whence
/ = C
S1 " *
9
2
J COS X
dx — J[ sec 2
x dx + Jf dx = —
1
COS
?
AT
tan a: + # + C.
1 1
5.6.19. /= f 4 2
1
. dx.
J cos x sin a:
2 4
C (sin cos 2 x) 2 2 2 4
C sin #-f- 2 sin x cos x-f cos x
r
— 4 ~~ cl.s 4
^ ~~
J cos x sin'2
x J * sin 2 x
= [
Sm
4
J cos x
/
dx-\-2 2
J cos *
1
C— — h C
1 f
2
J sin *
-
d
=
J
ftan 2 x ^ + 2 tan — cot x =
cos 2 a:
1
a:
= y tan 3
a: + 2 tanx — cot x-\-C.
§ 5.7. Integrating Irrational Functions by Substitutions 237
cosh 2
x — sinh x =2
1 ; sinh 2 x = y (cosh 2x — 1 );
t2
\
Solution.
5.6.21. /= J
cosh 3 ^d^.
/ = j cosh 2
* cosh xdx - j(l +/ 2
) d/ = +y + C =
*
= sinh x + y sinh 3
a: + C.
5.6.22. Find the integrals:
(a)
v '
f sinh
J
2
# cosh 2 x dx; »
(b)
v /
C-r-r
J sinh
—x+2^— — cosh
r
a;
III. J'/?(f, dt 9
238 Ch. V. Basic Classes of fntegrable Fund ions
where t = x + ^\ ax 2 + bx + c = ± p ± q 2 2
t
2
(singling out a perfect
square).
Integralsof the forms I to III can be reduced to integrals of
expressions rational with respect to sine or cosine (ordinary or hy-
perbolic) by means of the following substitutions:
Solution. 5 + 2x + x = 4 + (x+ 1)
2 2
. Let us put x+l=t. Then
/_ - j" (4+/
1 7(1+2^+^)3 2)3
cos zdz =
t
1 • ^ , ^ 1 tan z .
^ 1 2 .
^
+ x 1
-C.
4 y5 + 2* + jc
a
dx
5.7.2. /=f
J (*+l)» V*a+2*+2
Solution. x 2
+ 2x + 2 = (x+l) + L 2
Hence,
,
/ = C
\
cosh z dz
r —= C
\ U92
.
dz
= — C0th Z + C^ = ,
V l+sinh2 2 VT+Zi/- — ^+ 2
2jc +2 , ^>
5.7.3. I = ^x Vx^-ldx. 2
5.7.4. /-j^d*.
5.7.5. / = $K(* 2
— l)
3
d*.
x = cosh?; dx = sinh/d^.
Hence
/ = $j/"(cQsh*/ — l)
3
sinh t dt=\ sinh 4 / =
= -i j cosh 2
2/ df —i j cosh 2/ df + -J- J
d/ =
= y j(cosh«+ l)df — -J-
sinh 2/ + -J-
^ =
= ± sinh At — \ sinh 2t + 1- + C. 1
Let us return to x:
= Arcosh x = In (x + Vx* —
t 1 );
Hence
5.7.6. /=f * _
J (1+ V x)Vx-x*
Solution. We make the substitution:
x = sin 2
/; dx = 2sin*cos/df
240 Ch. V. Basic Classes of Integrable Functions
and get
j _ r
f 2 sin / cos t dt f 2 dt
=2 [tz™li
2
dt =2 tan
cos
t
COS
^- + C '
=
J ^ t
7 i =+ c-
2(l
CLi 1)
+c.
r ~> o / r
5 ,.s.
,=j 3
(x 2 — 2*+5) 2
5.8.1. / = Ji2£djc.
Solution. We integrate by parts, putting
w = In x\ *
dv = ^;
x 2 *
, dx 1
•
a: a:
,
7
_
-
In *
T + J"^-
,
C dx In
a:
x
—T + ^ 1
5.8.2. /=fi^.
j (i +<*)*
5.8.4. / = $e-*ln(e*+l)dx:.
§ 5.8. Integration of Other Transcendental Functions 241
/ = - e -ln(l+^) +
JT
^=- e -Mn(l+^) +
j
-^±i^^ =
e
~aarc tan *
5.8.5. / = l- — dx.
J (1 +* 2
)
2
xdx
u = arc tan x; dv =
V i +**
'
= T+x|/
2
arc tan x — In (x + |/* + l) + C.
a
242 Ch. V. Basic Classes of Integrable Functions
1
[ ^[(pMlfW dx Substitution (p (x) = t
2
J
/ W M dx <p' Integration by parts
J
/ W 9' W d*= / (x) (p (x)— 9 (x) /' (x) dx.
3
^ / W <f
(n)
W d* Reduced
{n)
to integration of the product
(x)q*(x) by the formula for multiple in-
f
tegration by parts
/ (
x) (p
(w)
(x) dx = f (x) q><»
- 1) (x) —
J
— f'(x) q>
( "- 2 > (x) + /" (x) (p
( "- 3 (*)—...
>
~ L a a2 1
a3 *
* '
~ ~
t
P"
+ ( »'< j? +c
C Mx + N
5
J x
2
+ px+q ^ X> Substitution
p
2 — 4q <
§ 5.9. Methods of Integration (List of Basic Integrals) 243
a.
r dx
o Reduction formula is used
x 2n —3
n
~(2n — 2)(x 2 + l)"- 1 1
2/1 —2
7 fPW
\ttH
, .
P(x)
where 7r^-J - Integrand is expressed in the form of a
sum of partial fractions
is a proper rational frac- P(x) A± A2 At
tion
Q(x) (x—x x )
1
(x— x t 2 1 ••• 1
X — Xl )l~T~
= (x — x
) (
Q (x) 1)
1
(x-
-x )" ... 2 (x 2 + px + 1
(x-x 2 )
1
(*-* 2 )
2 1 1
(*-* 2 )'« 1
+ ...
*a+p* + <7
1
(x* + px + q)* 1
{x * + px + q) k
~~n
'
T
where R is a rational
function of its arguments. cx-\~d
,
Mx+N *-mS m +
f t
V ax 2
-\~bx+c=t (x—x x )
(Aac —b 2
< 0).
V 2 - 4ac
sin /
2a
X
+2a=< V b2 — 4ac cos t (a < 0,
2a
\ac —b 2
< 0)
V b2 - 4ac
sec /
2a
X + Ta =
y 2 — Aac
2a
cosec / (a > 0,
Aac —b 2
< 0)
y 4ac--b 2
tan t
2a
V 4ac--b 2
2a
cot / (a > 0,
4ac- -6 2 > 0)
12
Pn W dx t
Write the equality
}/"ax 2 + +c toc
P„ (x) dx
where P„ (*) is a polyno-
j/~a# 2 + +c
w _
mial of degree n. 1 6#
dx
|/~ax 2 -j-kx;-j-c
+ |q b . 1 W(2(w + 6) + A i
dx
V ax' + bx + c
2
1st case
2nd case
*
if is an integer, then the substitu-
n
tion a-\-bx n applied, where— t
k is k is the
denominator of the fraction p;
3rd case
15
^ R (sin x t
cos x) dx Universal substitution tan -?r=t.
16
^ R (sinh x, cosh x) dx The substitution tanh-^- = t is used. In
this case
17
^ sin ax sin bxdx Transform the product of trigonometric
functions into a sum or difference, using one
bxdx of the following formulas:
^ sin ax cos
sin ax sin bx —
^ cos ax cos bx dx —
= -^- [cos (a b) a;— cos (a-\-b) x]
cos ax cos 6* =
— -i- [cos (a — fr) x-j-cos (a-\-b) x]
sin ax cos bx =
=-^- [sin (a b) x-j-sin (a+b) x)
sin 2
n
x—
1 — cos 2x ; cos 2 x =— —
1+cos2x
Let a function f (x) be defined in the closed interval [a, b]. The
following is called the integral sum:
i-
n- i
The sum S„ = 2= M,. Aa:,- is called the upper (integral) sum, and
/
A2 — 1
J
sin xdx
find the upper and lower integral sums corresponding to the division
of the closed interval [0, n] into 3 and 6 equal subintervals.
248 Ch. VI. The Definite Integral
%o
_
—n = n
-^2
_
— ~3~ 2ji
xs _
— *^ •
^ » »
[iT' T"j
* s m =s
i
i n »
an d the greatest value is M =
x
m = sin n = 0,
n M
AA 2ji
= sin -y V3
= -^y- .
2 2
k=0
=
.
rv
by the x x2 x3
points y *t=-g-, >
^^T* Xb
== —
5ji
f
x6 = jt, we find by analogy:
m = 0, 7W = sin-^- = 2
l
m = sm TT ~~
x
. ji 1
2
'
M 1
== sin = 2
m = sin T "~
2
Jl
2
»
M = sin -y =
2 1,
m = sm T3
2
9
7W 3 = sin-y = 1,
5ji
= sin -y = 2ji /3
m = sin-^-
4
6
"~
. 1
2
'
7W 4
.
m = sin 5
ji == o, M = sin — =
5
1
2
'
§ 6.1. The Lower and Upper Integral Sums 249
s3 ^ ^ s6 ^ sin x dx ^S ^S 6 3
n-i
J
sin xdx — 2 sm %k & x < 0.001 k
0<S„-s„< 0.001.
But
n-i n-i
S n -s n = 2 (Mi—m,) A*, < 5 2 (M,— /n f ),
i =o i = o
where M
and m are the greatest and the least values of the func-
f {
6.1.3. Show that the Dirichlet function [see Problem 1.14.4 (b)]
is not integrable in the interval [0, 1].
to /, i.e. v(t) =
kv. For a short time period A^ the velocity incre-
ment is approximately equal to the acceleration at the instant t
multiplied by A/. But in our case acceleration is constant, there-
fore Au = g"A/, and hence, v(t) gt since At t /. = y
= — =
Let us subdivide the time interval from t=a to t = b into n
equal parts; then the duration A/ of each subinterval will be equal
to A/^^1n^. We assume that during
to
each subinterval of time the
body moves uniformly with a velocity equal to its velocity at the
beginning of this interval, i.e.
v =ga,
( . , b —a
v n -i = g[<* + (n— J
)
Whence we find the distance covered by the body during the ith
Vi
subinterval: . The entire distance covered by the body is
approximately equal to
^^s n = ^(v + v + 1
. . . +v n _ = 1)
b—a
-7T8 na+ «
1
n.
\-2
n
\-
1
. . .
1 v
1)
'
n
— anin —
+—
b 1)1
= (b—a)g
z i \ T
[a
,
2
2~~"J
t =o
b b
s =^ vdt = ^gtdt = j- (b — a 2 2
)
.
a a
Solution. By definition,
1
lhn
«... 2n
~2-
252 Ch. VI. The Definite Integral
Hence,
i
x dx = y .
J
o
2. Using this example, we will show that for any other choice
of points l f the limit of the integral sum will be the same.
Take, for instance, the mid-points of the subintervals as £t
- =
f
+T '
= -}T (t ==0 *
1 n ~V-
---
i/o ixl
+(2n_1) ==
4^
= ^-l
J
i =
Hence
lim / n = i-*
n -+ cc
\x m dx (m=^— 1, 0<a<b).
a
_L JL A
x = a\ Xi=a(±-\
a
,
x^ai^A
a J
,...,*„ =a a
. J \ \ .
= aq n " 1
(q— l) =a — 1 and tends to zero with
n- 1 n- 1
i=0 = t
n (m + l)n 1 ,o 1
d* =
7h~T\
(
bm+1 — am+1 )-
it-
i
Ax, = <7'
+1
— f'^'fa-l),
and so t q (q— 1) >0 as
max Ax = n~
- oo, i. e. as g x
1. — n— —
Now us choose the right-hand end-points of the sub intervals
let
as the points \ h i.e., li x i+ ^=q i+1 = .
i=0 fe
' i=0 9
2
T
"( 2 "-0 =ln2>
lim /„= lim
n -* cc n -* <x>
—
2"
—
1
since 2 — ~ — In 2 1
J
as n—oo.
254 Ch. VI. The Definite Integral
And SO,
dx
= ln2.
*
/=$ J/25 —x 2
dx 9
is equal to
i j.
—25ji
Hence,
I = ^V25-x 2
dx = ^.
/ =$ (4x— l)dx.
^-arcsin-J (0<x<a).
4
2
J1 a: dx
H u
x
y \
1
la.
,27
expresses the area S 0AMx of the por-
tion of a circle of radius a lying in
the first quadrant (see Fig. 59).
V This area equals the sum of the
areas of the triangle and theOMx
sector OAM.
>OMx
Fig. 59
§ 6.1. The Lower and Upper Integral Sums 255
where sin/ = -
Hence,
a2 x
^OAM — ~Y arC Sin "J
c .
'
and consequently,
I = ~ya
2
2 — # + 4r arc sin—
2
2 a
.
(a)
J
s\n 3 xdx = 0; (b) [ e~*
2
dx^2 \e~*
2
dx.
^ s'm
3
xdx= 0.
o Fig. 60
2jx
§(2x+\)dx = 6; jK9=]?dx =
9jt
(c) (d)
2
256 Ch. VI. The Definite Integral
6.1.14. Passing to the limit from the integral sums, compute the
integral
\f{x)dx=F(x) \
b
= F(b)-F(a),
where F (x) is one of the antideri vatives of the function f(x), i.e.
F'(x) = f(x) (a
Solution. Since the function F (x) = arc tan x is one of the anti-
formula we get
/— f
dx —
~~
J l+* 2
ji
2 2
6
§ 6.2. Newton- Leibniz Formula 257
I
x2 for <x< 1
f (
x) =\ for 1 <x<2.
2
Evaluate J f
(x)dx.
6
2
Solution.
\f(x)dx=^f
12
By
(x)
the additivity property of the integral
1 o I
I 3 2
_
~
X*_ I
^
, 2^
X
~2
3 I
3
1
/ = Sl l—x\dx.
Solution. Since
,
| 1 —x for 0<*< 1,
1
^ 1
1 x— 1 for 1 <*<2,
we obtain, taking advantage of the additivity property of the integral,
2 1 2
1— x\dx= 5 (1— J
(x— l)d;c =
J |
-*) 2 (*-
(1
2
,
l)
2
=1+1=1
2^2 l#
a
where a < b.
Solution. If 0^a<6, then / (*)
= 1~- == 1 , therefore
a
b
b b
§l±\-dx = \b\-\a\.
a
V3
dx
T-i t
— — arc
= 2
1
rr tan
-77tanct
, 2x
\—x
Vs
o 2
[arctan( V 3) — arc tan 0] = ^ ,
V3
dx ,
arc tan x
V3
= arc tan ]/"3 — arc tan = -?-
o
o
jt
dx
J 1 + 2 sin H 2 2
J cos' x-j- 3 sin x
71 dx
+3 tan 2
1
arc tan (V 3tanx) = 0.
-f I * j/"3~
—F=-arc tan(+oo) = —— ,
x) =
n
1
C dx __ C 1 dx
2
J cos # + 3 sin 2 # J cot 2 # +3 sin 2 *
o _JT_
arc tan ([/ 3 tan a:) arc tan (|/~3 tan a;)
^"3
dx.
I I
cos x, O^Zx^-j ,
Therefore
cos 2x
1 -j-
dx = j
cosxdx + ( cosx)c(a: =
J
^ cos x dx = sin x^ = 0.
§ 6.2. Newton-Leibniz Formula 261
/ = J
Kl— cos 2xdx.
Solution. We have
Since |
sin a: |
has a period ji, then
IOOji IOOji
jj \
r
\—cos2xdx=\r 2 J |
sin a: |
rfA: =
o o
ji
= 100 J/T J
sinxdA;=200|/2 .
Ix—
-3
2 1
'
2
/= sin a ydx; (d) / =
J ^qry^;
(c)
j
-ji o
_2_
JT
-
I
sin
£ f
i_
n
1
3
xdx
JI
T
(j) 7=5 Kcos* — cos 3 Jed*;
jt
2
~
"3
262 Ch. VI. The Definite Integral
(x)dx<^ q>(x)dx.
J f J
a a
In particular,
b b
2. m (b—a) ^lf{x)dx<^M (b — a) 9
where m
is the least value, and the greatest value of the func- M
tion f(x) on the interval [a, b] (estimation of an integral).
3. If the function f (x) is continuous on the interval [a b], then y
\f(x)dx=f(&)(b—a), a<l<b
a
(mean-value theorem).
4. If the functions f(x) and y(x) are continuous on [a, b] t
and
<p(x), in addition, retains its sign on this interval, then
b b
2
+2
Solution, (a) Since the function / (x) = )/3 + x increases mono-
2
4 < ^3 + *
$
3
dx:<2|/"30^ 10.95.
'
jt
3"
L"4
since its derivative
m-
3 V3
2ji
3 /*3 / ji Jt
2ji
i. e.
J
~> dx.
10
[-\-x 9
< 10" 8 is fulfilled.
Therefore
19
C sin
TT X 8
<(19— 10)10" < 8 10"?
J
10
x 3 dx
is the greater?
2
(a) 0< \ 3
/l^_ < j\ (b) 1 < [e* dx<e.
b ^ 1+ * 8
b
then
i i
o < (-
s
*M=< [*dx=± 1
(b) Since for <x< 1 there exists the inequality 1 < ex < e,
*
then
i ) i
2
^dx < ^e* dx< ^ edx.
T
^e-R*tox dx< JL(i_e-«) (R>0).
o
^0,
jx \
[see
J
sin *
<^ g n
§ 6.3. Estimating an Integral. Integral as a Fund, of Its Limits 265
and
2 2
~2R
ji
r -—
2^
^
e~ R sin
*dx < ^
e n *
dx -
6.3.6. Prove that for any functions f (x) and g(x), integrable on
the interval (a, b), the Schwarz-Bunyakovsky inequality takes place:
^f(x)g(x)dx <:
or
& /; b
X2
J
g
2
(x) dx—Tk
J
/ (x) g (x)dx + J
/* (x) dx > 0.
a a a
J
/ (x) g (x) dx -Jf 2
(x) dx
J
g
2
(x) d* < 0.
j
Hence
r?
/~ b 1
y(x)g(x)dx 2
<j/ ^f*(x)dx ^g (x)dx,
, (' sin x i
sin x
I
dx = sin l\
b
—~—t2 = sin \ arc tan x = -J-sin6(0<6< 1).
J l+x' J l+*
266 Ch. VI. The Definite Integral
C sin x
< —r sin « 0.64.
A aA
, . «
-7-
\ T dx 1
P sin x
dx sin x dx = y^|2 (1
— cos < 1 cos 0.46.
i
+ Sj J
!
') 1
(b—a)f(a) <^f(x)dx<(b—a)-
f(a) + Hb) .
(b) if the function f (x) increases and has a convex graph in the
interval [a, b] 9
then
b
f(a)+f(b)
(b-a) < §f(x)dx<(b—a)f(b).
(b-a)f(a)< \f(x)dx
a
is obvious.
i
/ = J \fl+x i
dx = V'T+V, where 0<£< 1.
But
i < VT+T* < V%
whence
1< / < K2 » 1.414.
(b) The function f(x) = VY+ is concave on the interval [0, 1],
since
rw=f^±|>o, o<,o.
On the basis of the preceding problem we get
i ,
1 1
r
(d) \ +x*,g(x)
Put f(x) = \/ = 1 and take advantage of the Schwarz-
Bunyakovsky inequality
i i
r~\ i
where >
is an arbitrary constant.
c
Now us find the derivative F' (x) using the rule for differen-
let
-J
cos (t*)dt + J
cos (t*)dt;
F'(x) =- cos (t
2
) di
i
( x )*
+ cos (P)dt
J J
= — COS-o1/ 1
5 + COS*
, 1
Trrr == -.cos
1
-1,1^cosa;.
2
sin x
x
F" (nn)
v 7
=—
nn
cos (nn) = — —
nn v '
(
v 1
)
7
n
¥= 0.
Since second
derivative is non-zero at the points x
the nn =
(n = these points are points of extremum of the function,
1, 2, . .
.),
namely: maxima if n is odd, and minima if n is even.
x= ^ i/ z\n zdz\ y = J
z
2
\nzdz.
yt
Solution. As is known, =
xt
Find x and t y\\
. (
yt
\VT 'YT 2Yt
whence
= - 36/
31n
y'
x = f l
2
J/7 (f > 0).
-\Vt \nt
(arc tan x) 2 dx
^ sin I^a: ^
2
N
"°
(c) lim
+ oo
o
x2
Solution, (a) At x^-=0 the integral sin equals zero; it is
^
o
easy to check the fulfilment of the remaining conditions that ensure
270 Ch. VI. The Definite Integral
2
^ sin V".x dx sin V^xdx (* h
J
lim = lim 3x 2
= lim 2x 3xsin x2
2
3
'
x-+0
* \ 2
/
2 2 2
^e* dx 2 ^ <?* dx-?*
lim
o
= lim
X—*- + CD *-* + 00
2
2 e*
J
: lim
o
2
= lim 2-^— = 0.
X-> + CO e*
y *
(b) ^e 1
dt+ sin/d* = 0;
J
o o
(c)
^
|/"3 — 2sin 2
ede+ ^cos/d/=0.
y -x 2
r 1 dy ,
sin"
J
.0 _o
e
-' 2
^ + sin a
* 2 2x- = 0.
2
^^_2A;/? + ^ sin 2 A:
2
.
dx
§ 6.3. Estimating an Integral. Integral as a Fund, of Its Limits 271
Whence
/3 — 2 s ^ V3 — 2 sin 2
x
i
2
x + cos y ^ = 0; dx cos y
/ = l)(f — a
2) df;
o
x = aVn cos
( %- dt,
J
o
. — r»
t
y = ay sin
sir -77- dt
/; = (*-l)(*-2) 2
x't =ay
—n cos
. 1
Ttf^
= a yr n sin
— Tit
2
, y\ ,
272 Ch. VI. The Definite Integral
hence,
*7 */ ^ cos 3 -g-
=7 (*>0).
x = Cey.
then
1 =C^°-C,
which proves our assertion:
x=L' l
(y)=ey.
6.3.18. Given the graph of the function y=f(x) (Fig. 62), find
X
A 3a 2a.
2 2
Fig. 62 Fig. 63
= a y — 2* + llx — ) 2 1 -!- 6
( 1
The coefficient a is determined from the condition P(3)=2, whence
a = 3. Hence,
P(x) = x — 6x + 9jc-|
3 2
2.
274 Ch. VI. The Definite Integral
6.3.20. Find the polynomial P (x) of the least degree whose graph
has three points of inflection: (—1, 1), (1, 1) and a point with —
abscissa at which the curve is inclined to the axis of abscissas
at an angle of 60°.
Solution. Since the required function is a polynomial, the abscis-
sas of the points of inflection can only be among the roots of the
second derivative. The polynomial of the least degree with roots
— 1, 0, 1 has the form ax(x 2 — 1). Consequently,
P"(x)=a{x 3 — x).
=— = ^(T/a-i)
/>(_!) 1, whence a . H ence,
"
]/ 3 ~"
P(x)- 7
1
(3a:
5 — 1 Ojc
3
) + x J/3.
6.3.21. Taking advantage of the mean-value theorem for the
definite integral, prove that
i
2 ,
13 ^ J 10 — — 3 cos x
|
7
*
^ V +x 1
3
dx < -Xp- . Make sure that the application of the mean-
o
value theorem yields a rougher estimate.
§ 6.4. Changing the Variable in a Definite Integral 275
rically:
In /
(a) x = ^dz,
j
= j
e z dz\
2 5
sin / VT
(b) x= C
\ arc sin zdz, # = \
—
C sin- z2
dz.
(a) F(x) = )e 2
(l — t
2
)dt;
i
X2
(b) /7
W =f!!=g±id/.
+ J 2 e<
If a function ^=
the following conditions:
cp(/) satisfies
(1) q)(/) is a continuous single-valued function
defined in [a, p]
and has in this interval a continuous derivative q/(/);
(2) with / varying on [a, P] the values of the function x y(t) =
do not leave the limits of [a, b]\
(3) q)(a)=a and y($) = b,
then formula for changing the variable (or substitution) in the
the
definite integral is valid for any function f (x) which is continuous
on the interval [a f
b]:
lf(x)dx=\f[<p(t)] y'(t)dt.
a a
X t
a a
b p
r
6.4.1. Compute the integral
J \ \—x*dx.
[""IT' "t]
sa ^ s fi es a " ^e conditions of the theorem on changing
the variable in a definite integral, since it is continuously differen-
tiable, monotonic and
And so,
Tl Jl 1
since cos/>0 on the interval 3"
*
~3~J
Thus,
J
s
M— a
dx = 4 $
JT
cos 2 tdt=2 J
Jl
(l + cos20d/=»
" 3 "a
i
=2 [/ +
lsin2<]y = ^ + )/3.
iL
3
4 .
= 2 sec/; *l /
d* = 2^df; 2
2
cos /
ji
4
T
On the interval 0, the function 2 sec t is monotonic, there-
~J
fore the substitution is valid.
§ 6.4. Changing the Variable in a Definite Integral 277
Hence,
f ~4
_/ f J^sec 2 / —4 ^ sin/
16 sec 4 / ^cos 2 /^
H 3
Vz
=4i sin si
2
/ cos tdt =j2 s ' n3 ^
o
~ 32
*
a VI
dx
(a) §x*Vtf=^dx; (b)
j ^=
1
f
"8 dx
(a)7
v
J 6— 5 *f .
sinje = /; X /
cos x dx = dt\
ji
~2~ 1
The inverse function x = arc sin t[Q < -y for < < t 1) satisfies
cos xdx —3 i
_4_
/
6— 5 sin x-\- sin 2 x J 6- 5/-j-/ 2
" :ln-
—2 3
'
o o
l + t*'
ji
~2 1
interval —
1 2dt _ 9 f
df
J 2 + cos.v J
2+ \+t*
2
2
= -7=- arc tan -= . /
''arc tan —U — arc tan 0^ = 71
'
dx
2
(o>0, b>0).
cos 2 x-\-b 2 sin 2 a:
tan# = /,
dx
dt,
Hence,
<4
dx _— dt C dt _
~
+ bH ~ b*J
\
f
2
cos 2 x-\-b 2 sin x J cP a2 .
2
• -r arc tan —
= b —a arc tan—a = ab a \o
4
, , which exactly coin-
cides with the result of the substitution a = 6=1 into the initial
integral
n jt
4
dx [* ji
dx -
"
dx
dx; (b) f-
J X l^l + ln *
'
(0 dx.
j
§ 6.4. Changing the Variable in a Definite Integral 279
.1
C x sin x
/=\
6.4.7. Compute the integral
^ cos i x
dx -
ji
2 3T
~2~
To the integral
x sin x
'
2
dx
J 1 -j-cos 2 x
= n—
x /, X t
dx = — d/, ji n
T T
ji
Then
JI
2
— Q sin — (ji t) r (n — /) sin /
' 2 d/
J l+cos 2 (Jt-0
1 J 1-j-cos 2
1 /
C smt C t sin I
'
n df.
) 1-f-cos 2 /"*
J 1 + cos 2
/
Hence
/ =/i + / s
- J + cos x^
1
2
71
) 1 + cos 2
/ J 1 + cos 2
/
Since the first and the third integrals differ only in the notation
of the variable of integration, we have
n
) 1 + cos 2
*
280 Ch. VI. The Definite Integral
ji
T
2
du du
/ = 1+M a
Jl'
\ t-
1
x sin x
-j- COS X
— is not expressed in
\n(\+x)
2
dx.
1-f-x
dt
dx -=
cos 2 /
Jl
1
T
Hence,
Jl
4
In (1+ tan/) sec 2
sec 2 /
/
<tf = Jln(l +tan/)^.
Transform the sum 1 + tan/:
/2~sin (
/ -|
(I
T 4
JT Jt
T T
= -y ln2 + jlnsin^/ + dt— j In cos t dt - ln2 + /2
§ 6.4. Changing the Variable in a Definite Integral 281
= — dz
jt
dt y
JT
T
T
/2 = — ^ In cos ^— = ^ In sin
i
ji
2
= j" lnsin ^- + 2^ dz =I t.
Therefore
/=T ln2 -
6.4.9. Prove that for any given integral with finite limits a and b
one can always choose the linear substitution x = pt q (p, q con- +
stants) so as to transform this integral into a new one with limits
and 1.
Solution. We
notice that the substitution x pt q satisfies = +
explicitly the conditions of the theorem on changing the variable.
Since t must equal zero at x = a and / must equal unity at x =b
we have for p and q the following system of equations
= p-0 + q,
a
b = p.\+q,
whence p = b — a, q = a. Hence,
b l
$e<* +5 >
2
dx+3 .•(-*)' dx.
l i==
^
e ( x+ v 2
dx=-le<- t
+ ^ dt2
=- J
e«
- 1
>
2
dt
-4
dx = ^ and
_2_
o
3 / 2 \ 2 1
/2 =3$ e*\
x
~) dx = \ev-^dt.
Hence
+ =- je (
'- 1)2
dt+ J
= 0.
9
Note that neither of the integrals ^ e u+5)2 and $£ ^* 3
^ rfx is
p sin 2kx .
\
i
:
sin y
dX
= —
1
x tc /,
n
dx = — dt,
Jt
Since the definite integral does not depend on notation of the vari-
able of integration, we have
/ = — /, whence 7 = 0.
6.4.12. Compute the integral
2
dx
VT-
3
In both cases the variable Fig. 64
x= s\nt runs throughout the
entire interval'
1
2
^3 (see Fig. 64), the function sin/ being
»
2 J
2jt 5ji
monotonic both on and ~6~
|_<r> tJ 3
Let us show that the results of the two integrations will coincide.
Indeed,
V 3
2
r
dx — r
c° s * di
_ r _ n tan —
J a: 1^1 —x 2
J sin / cos / ,1 sin /
j
/-
12 V^3
V 3 2n 5jt
2 3 6
i
*
dx _ r cos t dt _ r dt
__
J xVl—x 2
J sin cos t) J sin/
5jt 2jx
6 3
, 5
tan ^"3
2-f
= In tan -In ln
^"3
tan^
the interval
|y , -yj , the values of the function x = sin t lie beyond
L^j=L(Xl )-L(x 2 ).
1 1 X,
t z
t=x z l y
1
dt = x dz x y
*•
Then
L(x,x 2 ) = §j +
i
^=
i
L(x l ) + L(x 2 ).
Putting here x 1 x 2 =x 3]
x2 = x—
x
3
, we obtain
1
m
It is also easy to obtain the other corollary L\x n
)= —
n
L(x) for
L{x n
J =^mL\x n \ L(x)=nL(x n ) ,
{x — 2f--^t.
Solution. A formal application of the substitution throughout the
interval [0, 3] would lead to the wrong result, since the inverse
function x = y(t) is double-valued: x = 2±Y~i> the function x
i.e.
not attain values x > 2, the latter values x < 2. To obtain a cor-
rect result we have to break up the given integral in the following
way:
3 2 3
J
(x — 2) 2
dx = J (x — 2) 2
dx + J (x — 2) 2
dx,
2
3 1 1
/2 =j(^-2)Mx=j/ -^ =i-Ji/rd/=i.
2 r
2
Hence, / = —8 -(- — = 3,
1
(x-2f
§{x-2)* dx -
= i+T= 3 -
Jt
Jt
4
, , _ C sin # + cos x i
dx
(e ) 7 =j 3 + sin2* >
/ = ^2ax-x*dx; (h)/ =
(g)
j fr ^ J
dx dx
.1 ^+1+^+1)3 ;
<b>J; + / —x 2 2 '
V(a* + b*)/2
(c (d)
).fmf^) ;
.f yV —a )(&
2 2 2 —* 2
)
V'(3a* + fc
2
)/2
4 +
— =
d#
a;
2-
2
1
-7T
2
arc tan
, #
2
2
-2~~ 2 [t-(-t)
Jl
T
-2
§ 6.4. Changing the Variable in a Definite Integral 287
X *
d*
,
= — dt
—2 1
2
1
2
2
[ dx r # r 1
arc tan 2t
2
_ _JX
2
+l
'
dx 2<ft
0.
j
5-2 cos* 5-2 1 — 2
=
(1+<1))
j
The result is obviously wrong, since the integrand is positive,
and, consequently, the integral of this function cannot be equal to
zero. Find the mistake.
_2
integral / = J
J/V+l dx?
o
i
for /?
288 Ch. VI. The Definite Integral
a a
J
f(x)dx=l[f(x) + f(-x)]dx
-a
stitution sin^ =
J
f(x)dx = 2^f(x)dx.
-a
2. If the function f (x) is odd on [
— a, a], then
a
J
f(x)dx--=0.
-a
\
j
f{x)dx^ J / (*)
a a+ nT
where /i is an integer.
i
-7
ji
(a) cosnxdx;
J /(*)
— jr
jt
*
sin 2a:
6.5.5. Compute the integral
J cos 4 x-\- sin 4 a:
JT
t = tan a:,
dt 9
ji
'
cos 2 x
T 1
JT
T 1
r tan x dx C 2t dt
2
JT
arc tan
J cos 2 #0 4" tan 4 x) J \+t*''
t
—
6.5.6. Prove the equality
J
cos xf (x 2 )dx =2 ^ cos xf (x 2 dx. )
C 2s 7
~~ ,
3* — >^ —
+— 6
— 10a: 5 7a;
3
1 + +
2a: 2---r^-ri
x 1 .
J +2 a;
2
-K~2
Solution.
V~2
C 2x7 + 3a;
6 — 1 0x b — 7x* — 1 2a;
2
+x+ 1 ,
dX ~~
J a; +22
-V~2
r 2xi—\0xb —7x*+X j ,
C 3a; 2 - 4) +
(a;
4
1
A
aX_t"
J *2 +2 J ^+2
-K~2 -K~2
V~2
= +2 [3(^-2x )+ 1?iT ]^ = 2
J _ 4^3 + 2
arc t an
|/"
x I v-2
= _ 16 "
K~2 2 |o 5 2 l/"2
cos* In
1 —x dx.
Solution. The function f(x) = cosx is even. Let us prove that the
function
+X
q>(x) = lny^
1
is odd:
cos* — X = 0.
ln\^dx 1
" 2
(a)
J
x s s'm d xdx -=0; (b) $ e cosx dx = 2 ^e cosx dx\
ji 1 o
~T "T
ji
(c)
J
sinrnxcostt^djc —O (m and n natural numbers);
— JT
Q
^f(x)dx^^f(a + b — x) dx.
a a
X t
x^=a + b — /, dx=^ — dt y a b
b a
Then we obtain
b
j f
a
(a
baa
a
the same interval, about the straight line x = ^-^-. Indeed, if the
point A lieson the x-axis and has the abscissa x, then the point A\
which is symmetrical to it about the indicated straight line, has
the abscissa x' =a + b — x. Therefore, (a + b— x') = [a + b — (a+
+b — x)] = f(x).
f f
But symmetrical figures have equal areas which are
expressed by definite integrals. And so, the proved equality is an
equality of areas of two symmetrical curvilinear trapezoids.
-lg(t-z)f(z)dz~lf(z)g(t- z) dz.
Jl Jl
2 T
m xdx=^ cos m xdx and apply
6.5.12. Prove the equality sin the
J
o o
T 2
2 , v 2
(y —
sin"
2
xdx = sin
m = cosmxdx.
J J J
Hence, in particular,
71 71
2 2
2 2
2/ = (sin 2 jc + cos 2
x) dx = dx = ~\
^ J
hence, / = -j
6.5.13. Prove the equality
ji 2
Solution. Since
^ /
(sin x) = jj
/ (sin x) dx +^ / (sinx) dx,
§ 6.5. Simplification of Integrals 293
ji T
^
f(s'mx)dx= / (sin x) dx.
J
_jx_
X t
x=n — /,
71 JT
dx= —dt, T
n
Then
JT
J
f(s\nx)dx = — ^ f [sin (n
— t)] dt
= J
/ (
sin *=J / ( sin x) dx -
o o
x=n — t,
dx= — dt, JT
ji
Then we obtain
J
a:/ (sin x)dx = — {[
(ji — [sin (ji — /)]d/ =
Jt
JT Jt
%\xf (s\x\x)dx= n J
f(sin x)dx,
s\n[n + j]x
= + cos^-|-cos2a;+
j
-x- . . . -j-cosnx,
2 im-
prove that
(a)
J
cp (jc) = Jta ;
(b)
J <P (x) cos kx dx = na k \
o o
2ji
(c)
J (p (x) sin kxdx = nb k (k=l, 2, tt).
J
u (#) u' (a:) dx = u (x) v (x) j
&
— (x) u' (x) dx
a a
\ xe x dx = xe x \
— \ ex dx = e — ex L = 1
§ 6.6. Integration by Parts. Reduction Formulas 295
ax sin
6.6.2. Compute the integral I= \ e bxdx.
o
= s'mbx,
u dv = e° x dx;
du = b cos bx dx, v = — e° x
/= —
a
e
ax smbx
a J
e
ax cos bx
dx -
= — —a [ e ax cos
J
bx dx = —
a
- L.
1
a
Then
/ = —a
-
\
—e
a
ax cos bx — ,
b r
e
ax s\n bxdx =
b f e~ \\ b* b G T +l) b*_
a \ a a J a2 ~ a2 a2
f
Hence
In particular, at a = b= \ we get
2
jt
Vx=t, X
x = t\
dx = 2t dt y ji
2
ji
Whence
.n
4 T
J
sin J/* dx: =2 J
/ sin / dt.
^
2
I
t sin tdt =2 171
~z
+) cos / dt =
2 sinH
lo
2 =2.
/„ = I
(a 2 — a;
2 )"" 1
(a
2 —x 2
) dx = a 2 l n _ x — \ x(a 2 — a:
2 )"" 1 xdx
= x;
u (a 2 —x 2
)
n~x
xdx = dv,
du = dx\ = —^-(a —x 1
2 2 n
v ' ) (n=£0).
2n
We obtain
| o
Y'
~i 7 «-
Whence
2n
/
l
»
=aU 2
I
2/1+1 '"-I*
Io = l dx = a>
o
we get
*~~ U
2n (2/1-2) (2/1-4) . . .
6-4-2 _ aw + 1 (2/z)H
/
(2/i+ 1) (2/i— 1) (2/1 — 3) . . . 5-3 (2/1+1)!!'
where
(2/i)!! =2-4-6 ... (2/i),
6.6.8. Using the result of the preceding problem obtain the fol-
lowing formula:
1
3 ^ 5 7 "I- • • • -r V U 2/1 + l (2/2+ 1)!!
'
In = \(\—x*) n dx =
o
i
= J(1— C} x + C t
2 2
*4 - C*x« + ...+(— 1 )
n
C nn x 2n ) dx =
x ~T
o ~T~ c 7 i • • •
2/7 + 1
^1 ^2 ^3 (-1)"
3^5 7
" "
t
2n+l »
m
// w ==5 sin^jcrfA;^^ cos xdx
(m a natural number).
cosjcd.x: = d/,
31
"2 1
Hm = \ (1— sin 2 *) 2
cosxdx=\{\—t*) 2
dt t
= J
cos#djc= 1,
therefore
_ (m-l)ll
* ml!
•
therefore
„ (m— 1) !! ji
ji
I = ^xsm m xdx
o
oo
ji
ji ji 2
/ = §xsm
m xdx =
j sm xdx = n
m s\n m xdx,
J J
o
ji rr
1 — u m is odd.
mil
i
[0, 1] for any m> and n > 0, by putting f (0) ^=0. Indeed,
du =— n(\nx) n - 1
#
— ,
ax, a = ——
xm +
m+
l
Hence,
i
we get
"
= (-!)" (m+l)" +
!
/„ 1 '
Then
is
The obtained formula
a positive integer, then,
is valid for all n
applying this
> 0,
formula
and — 1.
successively
If n
n
times, we get
n l)
/
— m+ i
/
1 m + i.»-i""( /0 +l)(m
+ 2)
7
+
==
• •
•
~~ m+n °'
* * '
(m+l)(m + 2)...(m + /i) *
§ 6.7. Approximating Definite Integrals 301
But
l
m+n dx: xm + n + l
u0 =\x
f» 1
'
m+n+ 1 o m+ /i +
o
Hence,
. /i (/i — l) (/i — 2) 3-2-
J
«. n
(
m+ J) ( m + 2). ..(m + /i) (m +n+ I)
'
(a)
J
arc tan V x dx\ (b) ^(x — \)e~ x dx\
o o
~3~
i
. . C Xdx r>
(e) J*ln(l +x 2
)dx\ (f)
J
ln(l + tanx)dx;
o o
ji
T 16
(g) ^ sin 2a: arc tan (sin x)dx\ (h) arc tan j/" V^x—ldx.
J
o 1
^(arccosArJ^dAr^Ai^y
1
— (/t — 1)
J
(arc cos a;)"
~2
dA; (n> 1).
o o
6.6.15. Prove that if f" (x) is continuous on [a, &], then the fol-
lowing formula is valid
b
j / (jc)dx * ^ [I / (* ) +/ + . . .
+ /(*„_,) + y f (*„)] .
302 Ch. VI. The Definite Integral
b
^f(x)dx^ -^.{f(x <) ) + f(x + 4[f(x + f(x
in ) l ) 3 )+...
iv
+f (*,„ - 1)] +2[f(x t ) + f (* +...+f (x
4) 2n _,)]}.
Assuming that f {x) exists and is bounded, the error in this formula
is estimated in the following way:
C dx
6.7.1. Approximate the integral I=\y^r£ using the trapezoidal
o
formula at n = 10.
1 +*i yi=
TTT„
Xi 1 +Xi M= TT7 t
r dx / 1.0000+0.5000 n qoqq
+ Aftnn1
, 1 .
?
.
/=3T+7~Tol 0.9091+0.8333 + ,
Estimate the error in the result obtained. We have f" (x) = ^_^ 3
.
exact numbers^ . Thus, the total error due to using the trapezoidal
formula and rounding the ordinates does not exceed 0.0018.
off
Note that when computing the given integral by the Newton-
Leibniz formula we obtain
dx
l+JC
ln(l+x) '
o
= In 2^0.69315.
Thus, the error in the result obtained does not exceed 0.0007, i. e.
we have obtained a result accurate to three decimal places.
1.5
C e°'
lx
6.7.2. Evaluate by Simpson's formula the integral \ —^-dx
0.5
accurate to four decimal places.
Solution. To give a value of 2n which ensures the required accu-
racy, we find /
iv
(a;). Successively differentiating f(x) — e-^-, we get
fiv (A;
)
= ^ (0.0001a;* — 0.004a; + 0.1 3
2a;
2 — 2.4x + 24) = ^g°- 1 *,
And so, |/
iv
(a;)| < 1.2x808 < 1000. Hence, the number 1000 may
be taken as Mv
304 Ch. VI. The Definite Integral
is satisfied.
Solving the inequality
5
l xl 000
180 (2/2)
4
<5xl0"
we obtain
2n > 19.
If we
calculate y t within five decimal places, i. e. with an error
not exceeding 10"
5
then the error of the final rounding off will
,
lues of x from 0.5 to 1.5 with the step h = 0.05. The calculations
are carried out within five decimal places.
IX,
i xi OAxi e o. yi
i %i 0. l*j e 0.lXi y\
i x\ at 1 = and
i = 20 at an odd t at an even i
0.50 2.10254
1 0.55 1.92098
2 0.60 1.76973
3 0.65 1.64178
4 0.70 1.53216
5 0.75 1.43717
6 0.80 1.35411
7 0.85 1.28085
8 0.90 1.21574
9 0.95 1.15754
10 1.00 1.10517
11 1.05 1.05782
12 1.10 1.01480
13 1.15 0.97554
14 1.20 0.93958
15 1.25 0.90652
16 1.30 0.87602
17 1.35 0.84781
18 1.40 0.82162
19 1.45 0.79727
20 1.50 0.77455
Sums 2.87709 12.02328 10.62893
j
^^^^(2.87709 + 4x 12.02328 +
0.5
6.7.3. The river is 26 m wide. The table below shows the succes-
sive depths of the river measured across its section at steps of 2 m:
X 2 4 6 8 10 12 14 16 18 20 22 24 26
y 0.3 0.9 1.7 2.1 2.8 3.4 3.3 3.0 3.5 2.9 1.7 1.2 0.8 0.6
Here x denotes the distance from one bank and y the correspond-
y
Hence,
Q = 55.5 x 1.3 « 72 (m 3 /sec).
impossible to estimate the error accurately in this case. Some
It is
error in Q is about 4 3
/sec. m
6.7.4. Compute the following integrals:
formula;
i
2
(b) ^e-* dx accurate to three decimal places, by the trapezoidal
o
formula.
1 .36
/ = ^ / (*) Ax*
1.05
§ 6.8. Additional Problems 307
F(x) = \f(t)dt
is continuous on the interval [0, 3] and that its derivative at each
interior point of this interval exists and is equal to f(x).
6.8.2. Show that the function
/ x In x , r\ . . ,
f(x) =\ at x =
^ — 1 at x=\
is integrable on the interval [0, 1].
ji
f* dx
6.8.6. Given the integral \
=- . Make sure that the fun-
te
J 1+ COS 2 X
o
ct ions
fc .
1
i
(xy
\
'
= —7r=-arc cos
V
1
2
}^2cosx
Vr + cos
l
= 2
x
z?
i
and r 29 m = -7=^
/
V
\
7
>^2
1
arc
j.tan ^
tan—^
1^2
6.8.7. For f (x) find such an antiderivative which attains the given
magnitude y = y at x =x (Cauchy's problem).
0.692 < 1
are valid.
r
1.096 < \ V xs\nxdx< 1.111.
are given on the interval [a, b], the function p ± (x) is non-negative,
§ 6.8. Additional Problems 309
Prove that
b b b
$ p3 (x) p x
(x) dx^\p 2
(x) px (x) dx^ I p,
(x) pi (x) dx.
a a a
6.8.14. Let the function f (x) be positive on the interval [a, b].
Prove that the expression
b h
a a
farctanx^l /
^
J x 2 Jf sin /
o o
$
UV {n) dx=[uV Kn - 1)
—U V , {n -2
>+. . . +(—l) n - 1 U in - l)
v] \a +
a
b
+ (— 1)" I
u {n >vdx.
Chapter
7
APPLICATIONS
OF THE DEFINITE INTEGRAL
, , . . . ,
—n as points of
7.1.1. Compute
— —
n
im
UI1J
-* Co
—n
jt I
\
sin
.
—
ji
n
.
h sin
. 2ji
n
,
h . . . + sin
,
1
(n
v
n
1)} Jil
.
ter will be the integral sum for the function f(x) = s\nx on the
interval [0, ji].
n
.
— .
\-
1
.
sin — 2ji
n
4-
1
. . .
'
sin
(n—l)n.
n — — hsin
1
. nn\
n )
=
7\
o
§ 7.1. Computing the Limits of Sums 311
lim ( r
1
+ *
+ . . . + -
1
V
Solution. Transform the sum in parentheses in the following way:
1
+ !
+ .
.+-=1
r
V~4n 2 -l '
V~4n 2 -2 2 \ 4n i -
1
+ ..-+•
'W-± V -(4)' 4
The obtained sum is the integral sum for the function f (x)
=
= on the interval [0, 1] subdivided into n equal parts.
Y==^
The limit of this sum as n— oo is equal to the definite integral
of this function from to 1:
lim ( r
1
+ r — .-
+ . . . + r
1
) =
1
- il
7.1.3. Compute
lim JL
AZ -* CD
1
+/^+/HT5+/5T5+--- + /»-+3fcr,]
Solution. Transform the given expression in the following way:
3 1
The obtained sum is the integral sum for the function / (x)
=
= Y 011 ^e interval therefore, by definition,
^^
[0, 3];
A™ H l+ 3
+
3
+ + /--+s5j=d) -
,
Y dx = 2VT+x = 4—2 = 2.
=
J V T^ c
dx = ^(l + x)~
312 Ch. VII. Applications of the Definite Integral
(C) lim
^V*+V}±:.+V'n.
In A = lim In J^JiL ^ li m — In —n + ln —n +1 1
. . . +1
In
= (A;ln x — x)
i
In #dA;
J {
n->-cc
li=~§f(x)dx.
i
i
% \
2
2
The square root
b __ a j
[/ (x)] of the average value of the
j
a
square ol the function is called the root mean square (rms) of the
function f (x) over [a, b].
7.2.1. Find the average value jli of the function f(x)=i/ x over
the interval [0, 1].
j/ xax = -
1
—A
o~~ 4
c
§7.2. Finding Average Values of a Function 313
(b) over 2 ]-
/(*)=?qrr [0.
hyperbola ^
a
— j^b =
2 2
l over the interval a^x^2a.
Solution. The problem consists in finding the average value of
2a
1 C b
'
2b
a2 K^^-f ln(* + Vx^a^ =b[2\^3-\n(2 + V3)].
7.2 .4. Find the average ordinate of the sinusoid y = sinx over
the interval [0, ji].
Solution:
—
ji jt
u
^
=— jt
[ sin xdx = - cos x
ji
- « 0.637.
ji
J
o
C
\i- n = —2 • ji = \
.
sin x dx.
7.2.6. Show
that the average value of the function f (x)> conti-
nuous on the interval [a, b], is the limit of the arithmetic mean
of the values of this function taken over equal intervals of the
argument x.
points x ( ^a-\-i —
b
- (/=0, 1, 2, n).
314 Ch. VII. Applications of the Definite Integral
„ /(*o) + /(*i)+-.. 1
Vf/r)
/ =
This mean may be represented in the following form:
n~ l
Vn=— ^f(X l
a i)
Ax i>
t =
where
b—
AX:-
n
The latter sum is the integral sum for the function f(x), the-
refore
n ~ l b
X
l
V>n = ~a Hm =
l
pv 2 = 160.
4 j/7oo
3
pm =
'
3 160u" 2
do =
4(j/l00-j/4) Jf
3
4 /4
3
320
— U
v
= —3—- « 4.32 atm.
4(^/100-^4) 4
J/ 4 ^20(^/10+^/2)
7.2.8. In hydraulics there is Bazin's formula expressing the velo-
city v of water flowing in a wide rectangular channel as a function
of the depth h at which the point under consideration is situated
below the open surface,
2
v = v -20V HL (A) ,
Solution. We have
H
£=£ sin-^-
Thus, the average value of the electromotive force over one pe-
riod equals zero.
lem 7.2.9).
Solution. Since
E =E sin ~y~
we have
2nt
™dt=±rEl
2 C
{
- Q0S —
j ^o J sin' \ Y^-dt^
T . 4nt ~ _ £o
t- -T- Sin
4jt
-TfT-
T o"
~ 2 '
\i= lim ~ \
f(x)dx 9
316 Ch. VII. Applications of the Definite Integral
/=/ O
cos (cot + a);
u=u cos (cot +a+ cp),
wm = lim ^ f
[cos (2a>/ + 2a + cp) + cos cp] dt =
= Iim
|^o. sin(2cor + 2a + cp)-sin (
2a + + cp)
^ cos
^|
= ^
Hence, it is clear why so much importance is attached to the
quantity coscp in electrical engineering.
7.2.13. Find the average value jli of the function f (x) over the
indicated intervals:
(c) f(x) = — 2x + 3
3* over [0, 2].
eccentricity, is equal to b.
cos2 *
f(1 X ')=
'
sin 2 # + 4 cos 2
x
over the interval ^0, yj. Check directly that this average, equal
^y 2 (
x) (a^x^b), then its area is computed by the formula
b
In certain cases the left boundary x=a (or the right boundary
x = b) can degenerate into a point of intersection of the curves
(a)
Fig. 65
y =y and y
l
(x) =
y 2 (x). Then a and b are found as the abscissas
of the points of intersection of the indicated curves (Fig. 65, a, b).
7.3.1. Compute the area of the figure bounded by the straight
lines x 0, x =
2 and the curves y= 2x , y 2x—x2 (Fig. 66). = =
318 Ch. VII. Applications of the Definite Integral
Fig. 66 Fig. 67
I x = 2y 2 ;
\ x =l—3y\
S= f [(l-m-(-W)]dy=2(y-^)\l = l
Find the area of the
7.3.3.
figure contained between the 2
parabola x 2 =\y and the witch x ^y
of Agnesi y = -p-^ g
(see Fig. \ 2
68). U = J-
Solution. Find the abscis- <Z£f i
y= —
x2
.
whence
a:
2
8
+4
- = 4
or x* + 4x — 32=0. 2
+4 dx = ^4 arc tan-|- — -V
\2J
\
= 2ax, y
2
we obtain x2 + 2ax — 3a = 0, 2
whence we Fig. 69
get the only positive root: x A a. Analo- =
gously, we find the abscissa of the point D of intersection of the
circle x2 y
2
+ =
3a2 and the parabola x2 = 2ay\ xD = a\r
2^.
aVl
Sr= [y*(x)—yi(x)]dx 9
J
V 3a 2
— 2
for a<x^aV2.
320 Ch. VII. Applications of the Definite Integral
V 2a • it
3
x 2
— 6a s- +1
2
V 3a 2 — x + — arc sin —-f——7t
2
26a Jo
J o L a |^3
^~2
2 ^~2
3
a2 6^2
3a 2
( arcsin —f=
drLMn }A 3
— arcsm —|A 1
\^ 3
1
arcsin -tt a\
3 2
7.3.6. Compute the area of the figure lying in the first quadrant
and bounded by the curves y 2 4x = y
x2 \y and x2 y
2
5. = + =
7.3.7. Compute the area of the
figure bounded by the lines y =
x+l, y = cosx and the A:-axis
(Fig. 70).
-7 1 Tt/2
Solution. The function
Fig. 70
(
x+ 1 if —1 <x<0,
y=f(x)-. cos a: if 0<x<-|
I
5= j
f(x)dx= ^ {x+ l)dx + ^cosxdx = + sinx 2 = -
the line x =
2 is the chord determining the segment.
§ 7.3. Computing Areas in Rectangular Coordinates 321
From the
Solution. equality y
2
x 2 (x- -1) it follows that =
2
* (;e— 1)>0, therefore either x or In other words, the =
domain of definition of the implicit function y
2
x3 2
consists = —
of the point x =
and the interval [1, oo). In computing the area
the isolated point (0, 0) does not play any role, therefore, the
interval of integration is [1, 2] (see Fig. 71).
Passing over to explicit representation
y = ±.xVx— 1, we see
that the segment is bounded abov e by the curve y = xVx— and 1
x— l-/ 2
,
dx=2t dt, 1
2 1
Then
1
y X) _V x \x- 1 - y-{x _
( 1
<j
l)f x > L
The common points of the symmetrical branches y l
(x) and y 2 {x) =
= — y (x) must lie on the x-axis. But y (x) = *
x A V \
x— 1 1
= only
at x l = and at x 2 = 1. _
Consequently, the loop is formed by the curves y = \f x(\— x)
and y = — Y x(l—x), 0<*<1 (see Fig. 72), the area enclosed
being
S=2jV*(l — x)dx = 2 ^x —x 2 2
)dx = ^.
2
y =x(x-1)
Fig. 72 Fig.73
2 2
S= j*[7 — 6x— (— x — 2x + l
3)] dx = J(x2 — 4x + 4)dx = -|
§ 7.3. Computing Areas in Rectangular Coordinates 323
£ + £-1 (a>b)
a point M(x,
y) lying in the first quadrant.
Show that the sector of the ellipse bounded by its semi-major
axis and the focal radius drawn to the point M has an area
So =
ab
-77-
2
arc cos —xa .
With the aid of this result deduce a formula for computing the
area of the entire ellipse.
y
1 M(x,y)
Fig. 74 Fig. 75
Since
2
— arc sin —a = arc cos—a ,
1
we obtain
>MABM — 2a -x Va — x + a
2 2 2
arc cos ~
j
Hence
—4 ^ellipse
9 "
ab
2
A
--arccos0 =—ab n
y=T
ab
•
x=±
By symmetry of the figure about the (/-axis the area sought is
2 ?
S = 2S 0AB0 = 2
j (3
V~y-\ V~y ) dy= 5 J Vy dy = ^p-
7.3.16. From an arbitrary point M(x, y) of the curve y x m =
(m > 0)
perpendiculars MN
and ML(x>0) are dropped onto the
coordinate axes. What part of the area of the rectangle ON does ML
the area ONMO
(Fig. 76) constitute?
Fig. 77
Xn — nil
^ i n — u, /-v «
l,
q
z, ... .
§ 7.3. Computing Areas in Rectangular Coordinates 325
The function y = e" ax sin fix is positive in the intervals (x 2ki x 2k+1 )
and negative in (x 2k+l x2k+2 ) i.e. the sign of the function in the
> y
= [^^(asinpx + pcosPx)
13
a
[g-a(«+l)«/pp (_l)»+i_ga«JV3p (—1)"] =
tP
*« 2
+P
Hence
-a(/z + I ) jt/3
j
(*-l)« + (» + 2)« = 16,_
\ y+ =
2 -(x-\) -2V3 +4, 2
whence x A =— 1, xD = 3.
Hence,
3
Sabdfa =
-
J
1
t(-^
2
+ 2x + - 2,/T + 2 + VT6-(*-l)*)]
1
> (
=
32 . ,
8
r/ = 1
6 x2 and the #-axis.
7.3.20. Compute the area enclosed between the parabolas
x = y2 \
x=^y 2
+l.
7.3.21. Compute the area of the portions cut off by the hyper-
bola x 1 3y
2
—
from the ellipse x 2=\
4y
2
+ = 8.
7.3.22. Compute the area enclosed by the curve y 2 =(\ —x 2
)
3
.
mulas:
S
Art
then the area
= -^y(t)x'(t)dt;
u
of the figure
S=§x(t)y'(t)dt;
a
is evaluated by one
S=
of the
S =yj i
x y' — yx')dt = -j^ abdt = nab.
o o
2 _2_
xy' —yx' =a 2
(cos 3 1 • 3 sin 2 / cos / + sin 3
/ • 3 cos 2 / sin t)=
= 3a 2
cos 2 /sin 2 /.
Hence,
2jt 2ji
=a 2
j 2cos/ + 4"0+ cos2 ')] dt = 3na 2
.
x=asint
Fig. 79
5=2
oo
ji
J
yx' dt =2
ji
^ b sin 2t x a cos / dt
ji
= \ab ^ cos
o
2
t smt dt =
7.4.5. Find the area of the region enclosed by the loop of the
curve
%=4(6-0; 0=4(6-0-
Solution. Locate the points of self-intersection of the curve. Both
functions x(t) and y(t) are defined throughout the entire number
scale oo — <
t <oo.
Fig. 80 Fig. 81
(6-0^Ul 27
'
24 5
y = t—J-
7.4.7. Compute the area enclosed by the cardioid: x =
= a cos t (1 +cos /); y = as\n t (1 +cos /).
S=
y §(xy' — yx') dt = a 2
J
( 1 + cos t)
2
dt = j na\
-JT
(b) x = 2t — t
2
\ y = 2t —P\ 2
(c) x=t 2
;
y=L (3 - n .
x = — cos 3
/; y~ — ^-sin a
/; c
2
=a*—b 2
.
§ 7.5. The Area in Polar Coordinates 331
7.5.1. Find the area of the region situated in the first quadrant
and bounded by the parabola y
2
= 4ax and the straight lines
y=x —
a and x = a.
Solution. Let us introduce a polar system of coordinates by
placing the pole at the focus F of the parabola and directing the
polar axis in the positive direction
along the #-axis. Then the equation of
the parabola will be p^
1
—— —-
coscp
, whe-
re p is the parameter of the parabola.
In this case p =
2a, and the focus F has
the coordinates (a, 0). Hence, the equa-
tion of the parabola will acquire the
form p =i — zzz^z ,
'
and those of the
l cos cp
4a 2
SpARP
FABF
"
2 (f— COS(p) 2
dcp = 2d 2 r dcp
\
.
4sin^"
JT
k/2 1
we obtain
cot (Jt/8)
"t^fj"/
sin (jx/4)
= +K 2
1 ,
>FABF = 2fl"(l+y^2
332 Ch. VII. Applications of the Definite Integral
the radius vector describes three equal loops of the curve as q) va-
ries between —
n and jx. Permissible values for q) are those at
which cos3(p^0, whence
_£ + ^ <(p< *
+ {k = 0i ±lf ±2 , ...)•
=| j 4a 2 cos 2 3(pdq) L
J
a2 dcp = a2 ^+ iLij .
-!t/9 -Jt/9
circles B is found from the equation 3|/2 acoscp = 3a sin q), whence
B (arc tan 1^2 ,
a|/6). As is seen from Fig. 84, the sought-for area
V3'
A
f
Fig. 84 Fig. 85
J
cos 2 cp dq> =4 a2 (t — arctan ^ 2 >
arc tan V 2
arc tan V 2
S 0CB0 -=Y a2
J
sin 2 q)dq) = -|-a 2
^arc tan|/2 —^p) •
o
Hence,
Soaho + Socbo = 2.25a 2
(ji— arc tan ]/Y—VT).
7.5.5. Find the area of the figure cut out by the circle p = ]f3 sin cp
changes from y to ji, and the arc OGB by the end-point of the
J J
= T (lt ]/T).
or
3a sin (p cos cp
cos 3 cp + sin 3
cp
3a sin 2cp
firstly, p = at cp = and at cp = ,
Fig. 86
and secondly, p — oo as cp
3ji
r and
.
1 C 9a 2 cos 2 cp sin 2 cp ,
>OAO
J (cos 3 (\ + sin 3
cp) 2 ^
Taking advantage of the curve's symmetry about the bisector
9 z
tan cp = 2,
dcp
cos 2 (p
T 1
which gives
4 1
_ qya
'
2
C cos 2 cp sin 2 cp ,
2 f 2 tfz
~* y " 2a(p ~""
^oao J (cos 3 cp+sin 3 cp) Jd + z 3 2
)
1+ = v, 2
3
3z dz = da,
2 1
1 2
dv 3
~ 2
a2 .
7.5.12. Compute the area of the portion flying inside the circle
V = \S{x)dx
a
where S(x) is the area of the section of the solid by a plane per-
pendicular to the x-axis at the point with abscissa x\ a and b are
the left and right boundaries of variation of x. The function S(x)
is supposed to be known and continuously changing as x varies
between a and b.
The volume V x of a solid generated by revolution about the
x-axis of the curvilinear trapezoid bounded by the curve y f(x) =
(f(x)^0) the x-axis and the straight lines x a and x b (a<b)
y
= =
is expressed by the integral
b
Vx = ji
\y
2
dx.
a
V x = n\{y\—y\)dx.
a
+ = i
with semi-axes b y 1
— ;
cy 1 — . Hence the area of the
r i—^
S(x)=TLbJ/ r xc ^ = nbc[\— (—a:
Therefore the volume V of the ellipsoid is
— ^ dx = nbc x — x
3 a 4
V = ^ nbc ^
1
3 a2 -a
= o
n u
abc.
Fig. 87 Fig. 88
side EF = Va — 2 2
, therefore 5 (x) =a —x 2 2
and V =8 (a
2
— **) dx =
Fig. 89 Fig.
a_ o_
2 2
Now find the volume of the solid. If the axes of coordinates are
arranged as indicated in Fig. 89, then in the section of the solid
by a plane perpendicular to the x-axis at the point with abscissa x
we obtain a parabolic segment of area S^~ah y
where a = 2y =
= 2/ R — x2 2
. Hence,
R R
S(x) = j\^W^ handV=[S(x)dx^jh^W^ dx = ^ 2 2
nhR\
-R
§ 7.6. Computing the Volume of a Solid 339
V =n 2
y dx =n \a
2
—x 2
dx =n ^a2 —4a 2
x
2
+ 6ax-
J J J j
— Aa 2
x 2
+x jdx= ^ na*.
2
1 1
I
1
1
1
>
1/2 2
Fie. 91 Fig. 92
V= 2
j\^ x dy = n^ (arc sin y) 2
dy.
340 Ch. VII. Applications of the Definite Integral
dy = cost dt,
1 n/2
V = jt n2 —&)
And so, \ t
2
cost dt. Integrating by parts, we get y=n (
4
i
y = ±-x + 2, 2
5x—8y+H== 0.
Whence x i4 =y; x B =2. In our case y x (x) =^ +2 2
and y 2 (x) =
= (5/8)jc + 7/4. Hence,
, 891
lra(T*+ 7 ) -(T + 2 )']^' JC,
1 280
Jl.
1/2
-x
a
-2a C\ B -x
0'
Fig. 93 Fig. 94
(y' — 2a) 2
= 4ax. B
Fig. 95
'
= 2n ^ y 2
dx.
342 Ch. VII. Applications of the Definite Integral
dx = —
3a cos 2 / sin t dt,
jx/2
y = a sin 3
1 9
a
Hence,
IT
= 6jxa 3
sin 7 /d/ - j sin 9 / dt
J
- o o
Using the formula from Problem 6.6.9 for computing the above
integrals, we get
t, c o / 6 4 2 8 6 4 2 \ 32 ,
At ^=0 we obtain xM = 2a t
at (p 2 = |n, xK =
§ 7.6. Computing the Volume of a Solid 343
X 9 X *
1
= a + cos
2 2
sin 2 2
r/
dx =
( 1
— a sin (1 +2 cos cp
(p) q),
cp) dcp,
— a/4 2jx/3 > —a/4 2n/3
2a ji
Thus,
o
V= ji
^ a2 (1 +cosqp) 2 sin 2 qp [ — a sin cp (1 + 2 coscp)] dcp —
2
— jt a2 (l +coscp) 2 sin 2 (p [
— a sin (p (1 + 2 cos (p)] d(p
==
J
2
= jia 3
^ sin 3 q) (1 +coscp) 2 (l + 2cosq))d(p =
o
i
= jia 3
+w) 2 (l +2u)du = ^ na 3
(w = cosqp).
-
(a) xy = 4, x= \, x = 4, */ =
about the x-axis;
(b) y = 2x — x'\ y = about the #-axis;
344 Ch. VII. Applications of the Definite Integral
y = —2 x.
7.6.21. Compute the volume of the solid generated by revolving
about the x-axis the curvilinear trapezoid bounded by the catenary
y = ^-(^e a
-\-e
a
) = acosh-^ and the straight lines xt =—c y
x2 =
=c (c> 0).
^j
about the #-axis.
l=lVl+y' 2
dx
a
where a and b are the abscissas of the end-points of the given arc.
adrant, y =x 2
. Hence,
3
y
f
= ~Vx and l/l+*/' 2 = j/ 1+-
Consequently,
Fig. 96
3 4
2
=A (10 KTo-i).
7.7.2. Compute the length of the arc cut off from the curve
4
y
2
=x 3
by the straight line * = -g.
7.7.3. Compute the arc length of the curve y=\ncos>x between
the points with the abscissas x=0, x=^.
Solution. Since y' = — tan x, then |/l + y' = \f\ -f tan x = sec x.
2 2
Hence,
n
4
3n
' sec xdx= In tan +\ = lntan-g
. .
|
from
x1 =a to x2 =b (b > a).
346 Ch. VII. Applications of the Definite Integral
7.7.5. Find the arc length of the curve x=-^y 2 — y \ny between
*'-U-w and ^ T+ 7 - 1
/(T^+i7=^+i-
Hence,
2 2
/ = J ^jqp^i dy = I (i y + 4) dy =T + T ln 2 -
1 1
~3 — ~\ T
In the first quadrant y =\a ( 3
x J and y = at x = a, y=x
at * =
Further,
2 '
y
'
=± ( a ^-jc T )
( -|) x~^~ = — x~~* [aJ-x^)
and
Consequently,
a
c
.L _J_
/ =8 ) a 3
x 3
Jx = 6a.
a
3/
/f
2
y
2
= 2x\
we find the point A (2, 4).
Find l fa. Here
2
J/2*, V\+y'*= )/ i+ J*.
Hence,
2
l
XB = ]/20arc tan 2.
Finally we have
I = A(ioi/TO— l)+4/5 arc tan 2.
y = Ux V~#-\ — In (x + KF=T)]
between
x = 1 and x = a+l.
348 Ch. VII. Applications of the Definite Integral
l = [Vx ,2
{t) + y' 2
{t)dt,
whence Vx' t
2
+ y' = at.
t
2
Hence,
2jt
2n
/ = at dt = =2an 2
.
J o
o
Hence
result, since
2ji
2ji
sin/cos/d/ = ySin 2
/ -0.
y = t-t*.
Solution. Let us find the limits of integration. Both functions
x(t) and y(t) are defined for all values of Since the function
x= V 3/ ^0, 2
the curve lies in the right half-plane. Since with a
change in sign of the parameter x(t) remains unchanged, while
y(t) changes sign, the curve is symmetrical about the #-axis. Furth-
ermore, the function x(t) takes
on one and the same value not
more than twice. Hence, it follows
that the points of self-intersection
of the curve lie on the x-axis, i.e.,
at y= (Fig. 98).
The direction in which the mo-
ving point M
(x, y) runs along the Fig. 98
curve as / changes from oo to oo —
is indicated by the arrows.
But y = at /
t
= 0, t
2 , 3
= ±1. Since x (t 2 ) =x (t 3 ) = J^3, the point
(^3, 0) is point of self-intersection of the curve. Conse-
the only
quently, we must integrate within the limits t 2 -1 and / =1.
8
=—
Differentiating the parametric equations of the curve with respect
to f, we get x't = 2V~3t, y't=\—3t\ whence
Consequently,
i(l+3t*)dt = 4.
= 2 — -j
Q
t t*
7.8.5. Compute the arc length of the curve #=-g- f y
between the points of intersection with the axes of coordinates.
whence
Vx + y' = Va
t
2
t
2 2
sin 2 / +b 2
cos 2 / = a V\—e 2
cos 2 /
6 = —a = -V a*a— b*
c
.
Thus
2ji T
l = a \V 1— 2
zos 2 tdt = Aa[\f\ — e 2
cos 2 / df.
The integral — 2
cos 2 tdt is not taken in elementary func-
o
tions; it is called the elliptic integral of the second kind. Putting
T 2
\ V\ — 2
e cos 2
/d/ = $ Kl— 2
sin 2 xdT=£(e),
where E (e) is the notation for the so-called complete elliptic inte-
gral of the second kind.
Consequently, for the arc length ot an ellipse the formula
/ =4a£(e) holds good.
It is usual practice to put e sina and to use the tables of va- =
lues for the function
E l
(a) =E 1
(arc sin e) =E (e).
* = </=y(' 2 -3)
x = 4 \^2as\nt; y = asm2t.
7.8.10. Find the arc length of the evolute of the ellipse
x = — cos 3
/, — ^-sin 3
/, c
2
=a 2 —b 2
.
between /, = and /
2
= ji.
*pi
where cp
t
and cp 2 are the values of the polar angle rp at the end-
points of the arc (cp, < cp 2 ).
7.9.1. Find the length of the first turn of the spiral ot Archi-
medes p = aq).
Solution. The first turn of the spiral is formed as the polai angle
cp changes from to 2n. Therefore
2ji 2ji
/ =\ \fa y 2 2
+a 2
dy = a\ Vy + 2
1 dcp
o o
=a [n |/"4ji
2
+ 1 -f 1 In (2n + + 1 ) .
<p
<Po
= a|/l + m 2
^
m
e '*dq) =a
IP — Po
V +m
1
2
.
Ap|,
o o
j^cos 2cp
s in
2
2cp \ _ a /"2
Y cos2(p 7
t V ]^ C os2(p
Hence,
o o
V — 2sin
1
2
cp
The latter integral is called the elliptic integral of the first kind.
It can be reduced to a form convenient for computing with the
did of special tables.
§ 7.9. Arc Length of a Curve in Polar Coordinates 353
p = asec ^q)
— y^ between (p = and cp =y
Solution. p^ = asec^q) — y^ tan ^(p
—y j ;
K*P
2
+ p; = «sec((p-y) ]/ l+tan ((p-f )=asec ((p-f
2 2
l =a j* sec 2 ^ q) —y dq)
4 V3 a.
^
p = asin y.
4
curve
a period 4jx, during half the period from to 2n the polar radius
increases from to a, and will describe half the curve by virtue
of its symmetry (Fig. 99).
Further, p^--=asin 3 (q)/4)cos(q)/4) and
Kp + P^ = Va? sin
2 2 8
(cp/4) +a 2
sin 6 (cp/4) cos 2 (cp/4) - a sin 3
(cp/4),
if 0<(p<2jT.
Hence,
'271 JT/2
/ = 2a ^
sin 3
(cp/4) d(p = 8a 3
j sin /d/ = ya ((p = 4f).
b o
2
d/ = Kp + p; dq) = Kp
a f
2
d(p» + dp = a
|/ P
2
(-^) ^ 1*.
354 Ch. VII. Applications of the Definite Integral
/ =
I
2
V P
2
4( -^) + ldP=j
1
2
2
V|(p -2+^ + 2
4)dp =
= 3 + ^.
4
;-.(/Kf*=i(f+'"' 2 ^
<Pi and •
<Pi = — 2 t0 ^^y-
L
where the differential of the arc length.
dl is
astroid x 3 y
3
+
a 3 about the #-axis. =
Solution. Differentiating the equation of the astroid we get
fx 3
+4*/ 3 y' = 0,
whence
y'=- y-
§ 7.10. Area of Surface of Revolution 355
X
a3 —x =t\
3
1
a3
—4" x 3
dx = 2tdt,
a
fl
l/3
y_
12
Then P=\2na 3
t*dt=^n<&
J
o
+ ±dx = 2n [ ^ *+ ^=
4
P = 2n§V~x }/~
!
1
1
4(4* + !)"
|(5K5-1).
P 2 = 2n I
x2 Vl + Ax 2
dx.
356 Ch. VII. Applications of the Definite Integral
Va* — x 2
'
Hence
-a
~b
where the quantity e= j/ a =~ is the eccentricity of the
ellipse.
When b —+a the eccentricity e tends to zero and
arc sin e
t
lim
.
8
= ,
I,
e - o
since the ellipse turns into a circle, in the limit we get the surface
area of the sphere:
P = 4jia a
.
§ 7 JO. Area of Surface of Revolution 357
hence, P = —a
.
1 8axx' = (3a — y) 2
— 2y(3a—y) = 3(3a — y)(a — y) y
whence = ^ ~y a~y
a ^
Using the formula computing the
xx'
^ . for
3a 3a
b o
— y )dy = 3na
2 2
.
Solution. Putting y = 0,
and, hence,
find ^ = and /
2t3
= ±K3,
^= and jc
2>3 = 3. Whence
the curve intersects it follows that
with the x-axis at two points: (0, 0) and (3, 0). When the para-
meter t changes sign, the sign of the function (x)t remains unchan-
ged, and the function y (t) changes
y, its sign, which means that the curve
1 m is symmetrical about the #-axis
(Fig. 101).
is sufficient to confine
ourselves to
the lower portion of the curve OnB
-1 n that corresponds to the variation
of the parameter between and
Fig. 101
+ 1^3. Differentiating with respect
to we find
ty
-.2t; y = t*-l
t
P = 2n\\y(t)\Vx' 2 + y' t t
2
dt =
VT vT
= 2n$ 3
(t*—3)(l
\
L "
°j + )dt = — ^-n
i" I 1 t
2
j (t* — 2t* — 3t)dt=3n.
7.10.10. Compute the surface area of the torus generated by re-
volving the circle x 2 +
(y— b) 2 r 2 (0 r b) about the *-axis. = < <
Solution. Let us represent the equation of the circle in parametric
form: x =
rzost\ y = b rs'mt. +
Hence
x't = — r sin t\ y't=r cos t.
|^cos2(p \
^l
V 2(p
Besides, # =
psincp=asin(p}/cos2q).
The sought-for surface area P is equal to double the area of the
surface generated by revolving the right-hand branch. Therefore
P = 2 x 2n f y dl
= 4«o« f
^^^P^ = 2«a« (2 - j/2 ).
J J K cos 2cp
dl = yi + y'2 dx= if + ( i
x
Ydx=-^=
Hence,
'
x+ Va* — x 2 -
a dx
P= 2n
V~2 V^ — x l
MN = p sin — (p^
=a|/cos2cpsin — qp^ ;
360 Ch. VII. Applications of the Definite Integral
then
a dw
j1
dl = r - .
V cos 2cp
Jl/4
-Jt/4
about the x-ax\s the arc of the curve # = xy between x =—2 and
x = 2.
7.10.15. Compute the area of the surface generated by revolving
one half-wave of the curve y =
s\nx about the x-axis.
7.10.16. Compute
the area of the surface generated by revolving
about the the arc of the parabola x 2
(/-axis 4ay between the points =
of intersection of the curve and the straight line y 3a. =
7.10.17. Find the area cf the surface formed by revolving about
the ;t-axis the arc of the curve x e'sin/; y^^cost between =
*=0 and *=y
7.10.18. Compute the area of the surface obtained by revolving
y =4 —^
t
1
between
the points of its intersection with the axes of coordinates.
Compute:
(a) the areas of the surfaces formed by revolving the arc OBA
about the x- and (/-axes;
(b) the volumes of the solids generated by revolving the figure OB AO
about the */-axis and the axis BC\
(c) the area of the surface generated by revolving the arc BA
about the axis BC\
§ 7.11. Geometrical Applic's of the Definite Integral 361
P x = 2n y dl = 2n j a ( 1
— cos t) 2a sin y dt =
J
64jta 3
8a sin 3
'ji
J y dt
When revolving about the (/-axis the arc OB A generates a sur-
face of area
TT
Py = 2n xdl = 4na 2
J
(t — sin t) sin y dt +
Jl
(b) When revolving about the y-axis the figure OBAO generates
a solid of volume
oo
2a 2a 2a
Vy = n^ (x\ — x\)dy = n J
jc| rf^ — Jt
J
x2 dj/,
o
Vy = n J
a2 (t — sin t)
2
asm tdt — n J
a2 (t — sin t)
2
asm tdt =
2jt
= na 3
J
(/ — sin t)
2
sin tdt =
2n
- -
= jia 3
J
/
2
sin/d/ — J
t{\— cos2t)dt+ 3
$ sin /d/ =6ji 3 a 3 .
fer the origin into the point C, which yields the following equa-
tions in the new system of coordinates
V=n ^ x'
2
dy' = na 3
^ (t
— — sin ji t)
2
sin t dt.
2ji
V = — na 3
\(z + sin z) 2
sin zdz — na 3
\ (z + sin z) 2
sin z dz =
n
=^ (9jx*— 16).
dl = 2a sin y |
dt \
= — 2a sin y dt.
Therefore
2a
P= ^2nxdl = — 4na 2 j*
(/ — — sin/) sin y dt =
jt
2n
ji
= 4na (z+
y dz = 4 (^2n — ^
2
sin z) cos
J*
b
(d) Transferring the origin into the point B and changing the
direction of the r/-axis, we get
x' = a(t — ji — sin /), y
f
=a (1 +cos /).
Putting / —n = z, we have
x' = a (z + sin 2), y' =a ( 1 — cos 2),
-ji
ji ji
J
-jt -ji
2
in the plane xOy.
V= 5 4 Vx (2 x) 8 (
2 x) dx = 4 \/"8
$ (2
— *) Vx dx =
256
= 4/8 (|2 J/* 3
--?-!/**) :
15
'
yt
where
h=x 2
{y)—x 1 (y) 9
x =x 1 (y)
being the equation of the Si
through integration:
V = 2n J
yhdy.
AV = n (x+ Ax) 2
y — nx y = 2n xy Ax + ny (Ax)
2 2
y
V= I
2n xydx = 2n \ x (2x2 + 3) dx = 4n.
o
as
^
directrix
—jY+y (Fig. 105
^ shows a quarter of the sought-
a for surface).
Subdivide the portion of the
circle shown in Fig. 105 into
small arcs AL The generatrices
y passing through the points of di-
Fig. 105 vision cut the cylinder surface
into strips. If infinitesimals of
higher order are neglected, the area of the strip ABCD is equal to
CD- AL
If p and cp are the polar coordinates of the point D, then
p = acoscp and CD = ]/ a2 — p = asinq),
2
and A/ = a.A(p, whence we
§ 7.11. Geometrical Applies of the Definite Integral 365
dP =a 2
sin cpdcp.
Hence,
P=4 J
a 2 sin cp dcp == 4a 2 .
7.11.6. Find the area of the surface cut off from a right circular
cylinder by a plane passing through the diameter of the base and
inclined at an angle of 45° to
the base.
Solution. Let the cylinder axis
be the e-axis, and the given dia-
meter the x-axis. Then the equa- C/
tion of the cylindrical surface q
will be x
2
+
y =a\ and that of
2
P = a2 J
sin qp dy =a 2
[
— cos cp]* = 2a 2
.
7.11.7. The axes of two circular cylinders with equal bases inter-
sect at right angles. Compute the surface area of the solid constitu-
ting the part common to both cylinders.
/? Vt
C_
6
7.11.10. Find the areas of the figures bounded by the curves rep-
resented parametrically:
(a) x=2t — t 2
; y = 2t 2
— t
3
;
7.11.11. Find the areas of the figures bounded by the curves given
in polar coordinates:
(a) p = asin3(p (a three-leaved rose);
(b) p =
1 — cos cp
x = V^2sin/; y = cost
is equal to the wavelength of the sinusoid y= s\nx.
7.11.16. Prove that the arc of the parabola y = ^ x2 correspon-
ding to the interval O^x^a
has the same length as the arc of
the spiral p =
Pcp corresponding to the interval O^p^a.
7.11.17. Find the ratio of the area enclosed by the loop of the
7.11.18. Find the volume of the segment cut off from the ellipti-
cal paraboloid
^+^ =x by the plane x = a.
7.11.20. Find the volume of the right elliptical cone whose base
is an ellipse with semi-axes a and fr, its altitude being equal to h.
(c) x + y — 2rx =
2 2
about the x-axis between and h.
A= ^f(x) dx.
where e x and e 2 are the values of the charges, and r is the distance
between them.
Note. When solving practical problems we assume that all the
data are expressed in one and the same system of units and omit
the dimensions of the corresponding quantities.
7.12.1. Compute
the force of pressure experienced by a vertical
triangle with base b and altitude h submerged base downwards in
water so that its vertex touches the surface of the water.
Solution. Introduce a system of coordinates as indicated in Fig. 107
and consider a horizontal strip of thickness dx located at an arbi-
trary depth x.
Assuming this strip to be a rectangle, find the differential of area
dS = MNdx. From the similarity of the triangles and ABC BMN
we have -^
b
= 4-,
h
whence MN = ^-h and dS = ^dx>
h
P = xdS = -^ |
x*dx = -jbh*.
J
n
7.12.3. A dam
has the form of a trapezoid whose upper
vertical
base is 70 m lower one 50 m, and the altitude 20 m.
long, the
Find the force of water pressure experienced by the dam (Fig. 108).
Solution. The differential (dS) of area of the hatched figure is
approximately equal to dS =
MNdx. Taking into consideration the
§ 7.12. Computing Physical Quantities 369
equal to
dP = xdS = x (70 — x) dx.
Integrating with respect to x from to 20, we get
P= \
(70x — x )dx=
2
11 333-
jl
2
dP.=
h
2
Xy+U-I
f
h
ldx=[ x ~)idx.
Consequently, the force of pressure of the mixture on the lower half
of the wall is
370 Ch. VII. Applications of the Definite Integral
r-^
P—P 4-
+ fi - — 4-
P — ———
+i6 - m 4 i6
lh 2 '
If the vessel were filled only with oil, the force of pressure P on
the same wall would be
h
IK1
=4.f xl dx = -r .
Hence,
P — P = l:lh*
16
= i-P.
5
eE
As b—* oo the work A tends to —a
7.12.7. Calculate the work performed in launching a rocket of
weight P from the ground vertically upwards to a height h.
Solution. Let us denote the force of attraction of the rocket by
the Earth by F, the mass of the rocket by Ri and the mass of m
the Earth by mE .
According to Newton's law
F =k X'
where x is the distance between the rocket and the centre of the
dA = F(x) dx = ^-dx.
§ 7.12. Computing Physical Quantities 371
Integrating, we obtain
R+h R+fi
PRh
VR — x
2
AnyR* Mco 2 /? 2
K = 2nya dx =
co /?
2 3 2 2
jx 5
o
nusoidal current
2ji
I = /„ sin
,
Q=0.24/? [l*dt.
In this case
T_
-0.12 RI 2 '
0.12/? 77*.
2ji
372 Ch. VII. Applications of the Definite Integral
M x = J y dl, M y = J x dl.
L L
/o=S(^ + »
f
)d/.
L
b b
a a
a* ^ b*
about the #-axis.
Solution. For the ellipse
ydi dx = Vy + (yy'ydx;
2
since y 2 =b 2
— ^-x 2
and yy
f
= — ~i x * we have
r
ydl = ]/ b2 —^x + ^x 2 2
dx =^Va — e x 2 2 2
dx y
Mx = — [ Vet 1 —£ * 2 2
dx = —[ V — e 2 2
x 2 dx =
-a
Z==:
~a{K
a ^r *
— ~T
ar ° s n 8 ^
)
=^ ^+ arc sln p )«
e -
Ix = i
\ by 2 dy = -^-.
j
dS = 2\y\dx = 2\/4axdx.
374 Ch. VII . Applications of the Definite Integral
Hence,
a a 5
- — -z-^x
*~x
lx = l\
-h
y
2
VW^ 2
dy = \\y 2
VW^ 2
dy.
lx =4
Jy
2
VR — y 2 2
dy =4 [ R 2 sin 2
/ . R cos tR cos t dt =
arc sin (h/R) arc sin (h/R)
7.13.6. Find the moment of inertia about the *-axis of the figure
bounded by two parabolas with dimensions indicated in Fig. 112.
Solution. Arrange the system of coordinates as shown in Fig. 112
and write the equations of the para-
bolas.
The equation *
of the left parabola is:
2
b2 ( ,
a
y ^Ya[ X +2 the equation of the
dI x = y*dS = y*\MN\dy,
where
\MN = x \
2
—x 1
.
2 f
— —y = f-
2
Hence,
6/2 b/ 2
ab*
y*(a-%tf)dy = 2
.
/,= j> u
j i,2
-b/2
7.13.7. Find the static moments about the x- and (/-axis of the
arc of the parabola y 2 2x between x and x 2 (y>0).= = =
7.13.8. Find the static moments about the axes of coordinates
7.13.10. Find the static moment about the x-axis of the figure
r
bounded by the lines y =x 2
\ y = \/ x.
b>0).
7.13.12. Find the moment of inertia of the trapezoid ABCD about
its base AD
if AD = a, BC=b and the altitude of the trapezoid
is equal to h.
7.13.13. Find the centre of gravity of the semicircle x 2 -\-y 2 --=a 2
situated above the x-ax\s.
Solution. Since the arc of the semicircle is symmetrical about the
(/-axis, the centre of gravity of the arc lies on the */-axis, i. e. x c 0. =
376 Ch. VI I. Applications of the Definite Integral
7.13.1: M x —2a 2
; therefore y c = —= Thus, xc = 0, yc = -.
7.13.14. Find the coordinates of the centre of gravity of the ca-
dl=\/ 1 + */'
2
dx= V\ + sinh 2
xrfx = cosh xdx
whence we find
/ = ^ d/
= ^ cosh xdx = sinh a.
l o
Then
a a
Q
=
L
= a sinh a — cosh a + 1.
Hence,
a sinh a — (cosh a — = a- 1) cosh a-
- = a — tanh-£-
sinh a sinh a
Analogously,
a a
M x = ^ydl= ^
cosh 2
x dx =
y J
( 1 + cosh 2*) djc =
I / . sinh 2x sinh 2a
=t[ x +-t-
a sinh 2a
TH 4 cosh a
sinh a 2 sinh a
7.13.15. Find the centre of gravity of the first arc of the cycloid:
x =a(t —
sin /), y a(1 =
cos /) (0 t —
2n). < <
Solution. The first arc of the cycloid is symmetrical about the
straight line x =
na, therefore the centre of gravity of the arc of
the cycloid lies on this straight line and x c na. Since the length =
of the first arc of the cycloid l Sa we have = 9
2ji 2ji
ye =-j §ydl=--±2a J 2
(1— cos t)sm±dt=-j§ sin 3 ~2 dt =Ya '
+y =a* 3
situated in the first
quadrant.
§ 7.13. Computing Static Moments and Moments of Inertia 377
y = p sin = a (1 +cos
q) cp) sin (p.
dl =V + (y'^f
9/
d(p - 2a cos ^ d(p (see Problem 7.9.3), we have
JT
xc = \ \y ^ ^Va [
asin W + cos <P) ^ a cos-|- dcp =
:
2a ^ cos 4 y sin -|- dcp = — 4- a cos y 5
i 4
5
Analogously,
JT
yc=z
Ta \
xdl =^«cosq)(l + cos(p) 2acos^d(p =
L
ji n
_JT_
T
yc = 2a \ (2 cos b t — cos* t)dt =4ftg^ — 2ay = -^-a.
=y x V~9 —x 2
dx = 3\
I
b
3 3
5 — x2 ) dx =5.
x
and equals y , therefore the area
Thus,
My 4 Mx _ 20
~S"~ 3n'
ction coscp it is symmetrical about the polar axis and passes through
the origin of coordinates at (p = ±y.
Compute the area S of the figure obtained:
iX3x5
S = 2xy j =a ji
2 2
p dy 6 2
Jcos (pd(p= a 2x4x6 X '32
2
§ 7.13. Computing Static Moments and Moments of Inertia 379
x — p cos (p = a cos 4
cp;
_
2
r*
\
,
xy ax —jt
-y
ji
Solution. The straight line y = —2 x and the sine line y = s\nx inter-
5=
J
^sin jc ——x dx-
380 Ch. VII. Applications of the Definite Integral
Hence,
4 — jt
sin 2x 4
J_ Xr
4 — ji 2 4 3jx2- "6(4 — ji)
•
^x(^sinx — ^-x^dx —
12 — ji
2
dx — —
Jt (4- 3(4 ji) 12 3ji"
P=2n\ ydl }
L
with that for the ordinate of the centre of gravity of this curve
MX
y c = -f=T)ydi.
f Ai I
L
Hence we conclude that
P = 2n lyc = l-2ny ct
where / the length of the revolving arc, and 2ny c is the length
is
-yl) dx
a
with that for the ordinate of the centre of gravity of this figure
b
M x
yc = -j
Hence we conclude that
V= ji • 2S y c =S • 2 ny c
where S is the area of the revolving figure, and 2ny c is the length
arc length l =
na. Therefore, accor-
ding to the first Guldin theorem,
Ana 2 = na • 2ny c
Fig- 116
-a
Y a
X
a }A3
intersection of the medians and at a distance of b- from the
Q ^"3
axis of revolution in the first cas. j
, and b+ in the second.
Fig. 117
V J
7.13.26. Find the centre of gravity of the arc of a circle of radius
R subtending a central angle 2a.
7.13.27. Find the centre of gravity of the figure bounded by
the arc of the cosine line y = cosx between x= — y and x = ^ and
(b) Prove that an analogous area for a parabola of the fifth order
y = f(x) = Ax + Bx" + Cx + Dx + Ex + F
b 3 2
S=
^[ ,(£±»-/F?) + 8f(^) +
5
7.14.3. Show that the area of a figure bounded by any two ra-
dius vectors of the logarithmic spiral p ae m and its arc = '+
is pro-
portional to the difference of the squares of these radii.
volume
Using
of this solid is equal toV = ^ S (0) + 4S
deduce formulas for computing the volume of
this formula,
^ +S (h) .
V= 2ji
J
xy (x) dx.
384 Ch. VII. Applications of the Definite Integral
V == —
2ji r
l
p
3
((p) sin q)d(p.
a
7.14.8. Prove that the arc length of the curve given by the pa-
rametric equations
jc = nOcos/ + nOsin*,
y = — f"(t)sint + f'(t)cost
is equal to [f (t) + f (t)]l\.
7.14.9. Find the arc length of the curve represented parametri-
cally
x
cos
= C—
\
z
^— dz,
,
y = \
—
C sin z ,
dz
between the origin and the nearest point from the vertical tan-
gent line.
7.14.10. Deduce the formula for the arc length in polar coor-
dinates proceeding from the definition without passing over from
Cartesian coordinates to polar ones.
7.14.11. Prove that the arc length l(x) of the catenary y —
= cosh x measured from the point (0, 1) is expressed by the for-
mula / (#) =sinh a; and find parametric equations of this line,
using the arc length as the parameter.
7.14.12. A flexible thread is suspended at the points A and B
located at one and the same height. The distance between the
points is AB =2fr, the deflection of the thread is /. Assuming the sus-
pended thread to be a parabola, show that the length of the thread
at a sufficiently small y.
7.14.13. Find the ratio of the area enclosed by the loop of the
curve y =±
.
3
— x Vx to the area of the circle, whose circum-
ference is equal in length to the contour of the curve.
7.14.14. Compute the length of the arc formed by the intersection
of the parabolic cylinder
§ 7.14. Additional Problems 385
Ax 1
+ 2Bxy + Cy* + 2Dx + 2Ey + F = (AC — B > 0)
2
is equal to
A B D
S = where A -= B C E
(AC—B- D E F
7.14.16. Find: (a) the area S of the figure bounded by the hy-
perbola x 1 y'1 — U the positive part of the x-axis and the radius
vector connecting the origin of coordinates and the point M(x, y)
lying on this hyperbola.
(b) The area of the circular sector Q bounded by the x-axls and
the radius drawn from the centre to the point (x, y) lying on N
the circle x 2 +
y*— 1. Prove that the coordinates of the points M
and N are expressed respectively through the areas S and Q by
the formulas
\
(ax + b) f (x) dx - (a% + b) \ f
(x)dx
a a
(Vereshchagin's rule).
y 3
p cos (p dcp p sin cp dcp
^
X = 2
3"
cp
N-' 2 H 2
2
^ P *P
386 Ch. VI /. Applications of the Definite Integral
<p 2
^
pcoscp |/"p 2 -|- p"2 rfqp
^ p
sin cp ^~p 2 + p' 2
cfcp
= 2i ; « =JBi-
Chapter
8
IMPROPER INTEGRALS
any interval [a, A], Then lim \ f (x) dx is called the improper
integral of the function f (x) in the interval [a, +00] and is de-
+ 00
^ f
(x)dx and ^ f (x)dx.
— GO — 00
Thus,
+ GO A
f / (x) = lim ^ f
(x) dx;
a A-> + v a
B B
< ^ g (x) dx; from divergence of the integral ^f(x)dx it follows that
a a
OO
J
f(x)dx converges for X > 1 and diverges for X^l.
a
Absolute and conditional convergence. Let the function f(x) be
00
\f{x)dx <^\\f(x)\dx.
a a
If the integral \
j
f(x)dx converges, and [
)
\f(x)\dx diverges, then
a a
x
jxln 3 *
e2
^ Jx
e2
In 3 x aI™ x { 2 In 2 *
= lim
(b) By definition,
oo o A
2
+ 2x + 5
(instead of the point x any other finite point of the #-axis= may
be taken as an intermediate limit of integration).
Compute each of the limits standing in the right side of the
above equality:
o
s
= y arctan T + T
1 , 1 .
Jl
,
= * + 1 ^ ji l , l
1™ f o ,
1*
i c 'i m 4- arc tan
2 =T— arctan -2'
o
Hence,
—X
+ 2* + 5 2
"
(c) By definition,
qo A
^xs\nxdx= lim [xs'mxdx.
o ^ + 00
o
A / A x
14
lim ^xsinxdx^ lim f a;cosa;|^ + J cosArdA; j==
o
But the last limit does not exist. Consequently, the integral
QO
dx
x+x* (O
j V" (4x
2
+l) 3 '
QO
dx dx
(! + *)'
(e)
] x2 — 6x+10 ;
(f)
J
/4 —>• + GO 2 -\I2
=— lim
\
-3
— = 1 1.
y4 -> + GO V <4
2
dx
f
J l+2x 2 + 3x 4
for convergence.
Solution. The integrand
l + 2x + 3x
2 4
C dx
J x+sin 2
for convergence.
integral diverges.
§ 8.1. Improper Integrals with Infinite Limits 391
1 r
1 1
, x i 2 , ,n , .
x V ,
J J i j/ x
i
(*+ + 1) dx
for convergence.
Solution. The integrand is continuous and positive for x^\.
Determine its order of smallness X with respect to as x — oo;
since
x+ Vx+\ 1
1
+ 1^ * +
*2 +2 ^+l *
1+2 l/_L.±
the order of smallness According to the special comparison
00
r dx
J Vx (x-\)(x-2)
3
for convergence.
Solution. Since the function
f(x)= r _r_-±—- /
—=~ x
is an infinitesimal of order ^ = y3 with respect to —
1
as # — +oo,
according to the special comparison test the integral converges.
8.1.9. Test the integral
for convergence.
392 Ch. VIII. Improper Integrals
= -TT X
V
v-35
1 — cos—x dx
for convergence.
Solution. The integrand
f(x)=
'
v
'
1 — cos- = 2sin -
X
2
y ~2 — = -p-,
1
is positive and continuous for x^l. Since 2 sin 2 f
J
"
e
In dx. n>0
for convergence.
Solution. Transform the integrand:
1 + -
/(*) l
1 — 4 sin 2x ,
dx
for convergence.
§ 8.1. Improper Integrals with Infinite Limiis 393
1 — 4 sin 2x|
|
dx
'[/ x
— 4 sin
<^ —
1 2a: |
5
for convergence. Since 3
, and the integral
J x
converges conditionally.
Solution. Let us represent the given integral as the sum of two
integrals:
= \—
C slnx
dx=
. C sinx
\
— dx+ \— . P sin* ,
dx -
A C cos x dx P cos*
= lim
x*
A -> co IL J
2 Jl
T J
2
co
^ ^ ^
c ° sx
[
X
dx also converges. Now let us prove that the integral
n
2
sin x |
^ sin 2 x 1 — cos 2x
x x 2x
but the integral
/I -»• CO
31 Jl
T 2 T 2
=4 lim In 4 - 1 In £-1 j
It ji 1
00
^
C cos 2x
X
<te
f* cos 2x
diverges, since lim In A =oo, and the integral \ converges.
JX
T
8.1.14. Prove that the following integrals converge
Ot 00 00
2 2 4
(a) ^ s\n (x ) dx; cos (a; ) dx; (b)
J
2x cos (jc ) dx.
J
Let us represent the integral on the right side as the sum of two
integrals:
V' J V i J V i
COS / I
The
I
QO
T
integral cos(x 2 )dx is convergent. The integrals considered are cal-
J
2
integral Jcos(/ )d/. The latter integral converges as has just been
proved.
Note. Fresnel's integrals show that an improper integral can con-
verge even when the integrand does not vanish as x oo. The last —
convergent integral considered in item (b) shows that an improper
integral can converge even if the integrand is not bounded. Indeed,
at x = l/nn (n = 9 1, 2, ...) the integrand attains the values
=h \/ tin, i.e. it is unbounded.
8.1.15. Evaluate the improper integral
QO
dx
? n natural number.
J (l+* 2 )"'
Conse-
quently, by the theorem on changing a variable in an improper
integral
jt ji
~2~
oo 2
"
I(TW«I« xsec2M/= I cos2,, 2/d/ -
Therefore,
n/2 n = l >
? dx
aX
= f
>
1.3.5. ..(2/i — 3)
) ji
^
\
J<
l +* a
>"
\ 2.4.6. ..(2.-2) 'T- *> L
,
X
8.1.16. Compute the integral / = j-j-p^d*.
Solution. Apply the substitution
*=l/f; dx = — (l/t 2
)dt\ ^ = 00, f2 = 0;
If another integral
/
/
?
J
is
*2
l+* 4
added
//y _? (^
00
J
to
l
the
+
4
)^
l// 4
right
_?
J
and
/
4
+ l
•
2
dt.
r2 -f \/t
= t— l/t, = dz.
— + 00, z — ++ oo.
Make the substitution z (I l/t
2
)dt Then, as
* —H-O, — — 00 z and as t »- Hence,
00 r a
C dz dz dZ
lb
..
. 1 1
f ,
f
-00
1
2
(a) l~Y^dx\ (b) §e-* x 2m+1 dx.
x5 + * +l 2
1 7v
§ 8.2. Improper Integrals of Unbounded Functions 397
then
0.0031.
5^0.2155,
and for a step -^- = 0.5
S . 5
=0.2079.
Since the difference between the values is 0.0076, the integral I t
gives a more accurate value S 5
= 0.2079 with an error of the order
°^L6 - 0.0005.
Consequently, the sought-for integral is approximately equal to
/ & 0.208
with an error not exceeding 0.004, or / = 0.21 with all true deci-
mal places.
\ f (x)dx= lim \
f (x)dx.
J f {x)dx = $ f (x)dx + J f
(x) dx.
a a r
]f(x)dx=F(b) — F(a)
a
holds good.
Sometimes the function F (x) is called a generalized ant [derivative
for the function on the interval [a, b}.
f (x)
For the functions defined and positive on the interval a^x<b
convergence tests (comparison tests) analogous to the comparison
tests for improper integrals with infinite limits are valid.
Comparison test. Let the functions f (x) and g(x) be defined on
the interval a^x<band integrable on each interval [a, b e], —
< e <b —a. If ^f(x)^g(x), then from the convergence of the
b b
the integral ^f(x)dx converges for X< 1 and diverges for X^\.
a
In particular, the integral
b
C dx
(b — xf
converges for X 1 <
and diverges for % 1. ^
Absolute and conditional convergence. Let the function f (x) be
defined on the interval a?Cx<b and integrable on each interval
§ 8.2. Improper Integrals of Unbounded Functions 399
2
dx
(a)
l4^T : (b)
I cos,-
3 2
(c )
J
1
f4 x-x>-3
;
^ J FTT^f
P* 3 + ?/*— 2 f dx
V
Solution, (a) The integrand / (x)
=— .. \ is unbounded in the
x y \wx
neighbourhood of the point jt = l. It is integrable on any interval
[1+e, e] 9
since it is a continuous function.
Therefore
f^==
J
\
xr/\nx
V
lim
S-++Q J
1+8
\-J*
xW\nx
V
= lim
e^+o [i
3
/lH^|
e
|l
1
+-ej
=
8^+0
lim !* /ln»(l+e)
2 2
3 3_
2
'
-t
dx
= lim
J cos* e ^+ Jf cos x
= lim
e-> + o
In tan f
W4 + ^1 4 /
T
o
e
= Hm
b-> +o
In tan (£
\ 1
— 1
c dx r dx r dx
J V~4x— x'
2 — 3 ~~ J V~4x— x2 —3 J VT? -a;
2 —3
f ^ ^ = lim ( r
dx
== lim arc sin — 2) 2
1 + tr
1 1 + <?
3 3-t
f
J
-==
^-^-3
d*
6-.
lim
+
(
J
-7=^==
\
r \-(x—2)*
lim arc sin (x
8-. +
— 2)
2
' =
2 2
r
dx ~ r
dx i r — 2
JC
|
§8.2. Improper Integrals of Unbounded Functions 401
dx
J V^l 1 —x 2
\ J ^2_! p _+ J
j/- x 2 —\
1 1 1 +e
lim lnU + K* 2
— 1)
8-> +
8
lim [ln(2
-> +
+ K3) — ln(l + e + +e) 2 - l)] = In '2 + j/"3 ).
Hence,
= | + ln(2 + |/3).
dx
"
„3/5
17' lo
17'
The second and third summands are unbounded to the right of the
point x = 0. Therefore,
i i
dx
lim ^ x 11 / 15
1
_ !^
* 4/15 8-> +
analogously,
Hence,
i
625
dx = T7 + T- 2 'Y = "187*
1 1 x+2
/(*) =
1
— a:
3
(1— x)(l +x+x 2
)
TU
Then
P
j
_ _
djc
= - jC 1 dx
+ - Jf
,
1
o
x +2 ,
c .
Since
1-8
dx l-e
: lim lim In (1 x)
J I-* E _
8-> +
-3 b
i-, ^9=^) + C.
TCT =
J"
(9 arc sin
J
The function r
(x)= y^9 arc siny — xK9 x 2
^j is a generalized anti-
-3
(b) Transform the integrand
\/~*±x== 2
+* = 2 X
f (x)=s +
The indefinite integral is equal to
]/ |^dx^2arcsin|-l/4^7 + C.
2
j
The function f (x) =2 arc sin -|- — ]/~^ —x 2
is a generalized antideri-
dx
-1
x /x
3
for convergence.
Solution. At the point x = the integrand goes to infinity. Both
i^
ly, the given integral diverges. If this were ignored, and the New-
ton-Leibniz formula formally applied to this integral, we would
obtain the wrong result:
1
dx 3
f
= —6.
1: i
- i
/
(a)
\
\—F==dx\
P j e* u\
b)
Psin#-{-cos#
^
w -
,
dx.
v
r .
/(*)=
, sin #-|-cos x
5
~ 1
,
(c)
x i
J v/ x
cos
—
—
a: d#
sin
n (l 4-
In i/x?)
Solution, (a) The integrand f (x)= J,; ^
is positive in the
we have
lim
*-+ o
— .
v
1
= hni
x ->
- —=
*
lim .
*- o ^/* 2
= oo.
At the same time we have shown that /(x)~y-L= as x > 0, i.e.
r '
x
— is unbounded in the neigh-
as +
jc—> the function / (*) is an infinitely large quantity of order
ex dx ,i x f x l dx
(b)
(a)
.1 Tct ;
j K y u=ji? *
b o
1 2 _
^ /n r in +
fe)
V
'
J
r
x-sinx ' W J
(
gtanx_ 1
406 Ch. VIII. Improper Integrals
1
f sin•
7 dx
1 Vx
converges.
Solution. For <x< 1
sin —
x
Vx
C dx
But the integral y= converges, therefore, by the comparison
j
b
i
sin(i/*)
test, the integral
[
^x a j sQ conver g eS) an(j consequently
/ =^ In sin xdx
lnsin xdx=x\ns\n x —C \
J
COS X j
x - —
dx=
sin x
—
J
C
\ t
X
tan x
dx.
,
Since lim
tan x
1, lim
i^^ — Q' ^e ' as * integral is a proper
5 In sin xdA: = 2
J
lnsin2^=2 J
(In 2 + In sin + In cos f)df =
t
JT/4 JT/4
4
= 2Hn2f + 2
J
lnsin/<« + 2
J
lncos/df =
o o
JT/4 jt/4
Thus,
JT./2 JT/4 JT./2
^=
J
\ns\nxdx=- — In 2.
dx n
fx
yr==z(n a natural number).
00
1 Jl/ 2 JT/ 2
s\n n tdt.
„ ,
J 1^ l
-* 2
J
cos/
J
yy, 1
ft even,
— ——
n
-
n
^
2
. . .
~3 »
1
n odd.
f
dx r dx f 3a:
2
+2
(a)
J
Tun; (b) (c) y^rdx.
J k
1
408 Ch. VIII. Improper Integrals
l =^xm dx= m+ :
-
1 o m+ 1
"
u = In" x\ do = xm dx\
X m+l
J
du=n\n n-i
n 1
x
x —
<lX
u = -
x
;
m +
We get
^
1
" m+ 1 o
XT Jf ^In^^dx-
m+ 1 m-f- 1
/„-,.
n 1
=— n
= n ^n — x
)
= __ (— 1 )" /z!
/
I I i
And finally,
"
w (m+l)» + 1
j _ C e~ x dx
~ J ^/ 2-\-x — x 2
0.3
accurate to 0.03.
Solution. The integral has a singularity at the point x=2, since
2 + x — = (22
x)(l+x). Let us represent it as the sum of two in-
tegrals:
2 -e 2
2 -e
Now compute the first integral to the required accuracy, and estimate
the second one. For e^O.l we have
V*-
by Simpson's formula with a step h = 0.8 gives
S . 8
= 0.519,
and with a step ft/2 = 0.4,
S . 4
= 0.513.
And so, integral /j gives the more accurate value, 0.513, with an
error not exceeding 0.001. Taking into consideration that integral 2
/ ^0.52
with an error not exceeding 0.03.
Note. By putting e = 0.01, we get the estimate / 2 < 0.005, but the
computation of the integral
1.99
f e-*dx
4
0.3
F=;
Ccos 2 xdx f tanxdx sin#dx
.
< c)
x
Jo^gr: (d)
j
.
(e)
,
j
f
_1_
2
^
$ 8.3. Geometric and Physical Applications of Improper
Integrals
l
8.3.1. Find the area of the figure bounded by the curve y--
l +x*
(the witch of Agnesi) and its asymptote.
Solution. The function y = is continuous throughout the en-
} _^ 2
tire number scale, and lim y = 0. Consequently, the x-axis is the asymp-
X-+ CO
dx
ral S= JC \ -t-t—
\ -\-
r
X"
. By virtue of the symmetry of the figure about the
— oo
y-ax\s we have
^=
CO GO
A
S=f -4^- = 2[ T 2 lim arctanx = 9Z'-tt^ = „
31
Jl.
i i
-7 / X
Fig. 118 Making the substitution e~ x = /,
—
= e~* dx, we get x^=0 at
t = 1 , x = oo at £ = 0; hence
i
= ji[J/'l+ln(l+j/~2)].
8.3.3. Compute the area enclosed by the loop of the folium of Des-
cartes
x 3 -f if — 3axy = 0.
x^pcoscp; y = psincp.
Then p 3 cos 3 cp -f-
3
sin 3 cp — 3a p 2
cos cp sin = 0, whence,
cp cancelling p 2 ^
we get
3a cos cp sin cp
jt
i r* o j ^
9a 2
n
r
2
sin 2
sin'cpcos'cp
cp cos
2
cp ,
§ 8.3. Geometric and Physical Applic's of Improp. Integr's 411
tan cp = t\
CO gf
= dt\ cp = at t = 0, cp =y at t = oo. Thus we get
c 9a 2 f t
2
dt 9a 2 ,. C i
2
df 3a 2 ,. 1 1
<4
"
2
8.3.4. Find the volume of the solid generated by revolving the cis-
soid y 1
= 2a— a k° u * ^s as Y m ptote x=2a.
Solution. The cissoid is shown
in Fig. 119. Transfer the origin of
coordinates to the point 0' (2a, 0) without changing the direction of
the axes. In the new system of coordinates
X= — x 2a, Y =
y the equation of the cissoid
has the following form:
Y2
(X + 2a)*
-X
V = Ji$ X 2
dF = 2jcJX 2
dY.
X2 X 1
Hence,
(1 /
2 2
)
dt\ X + 2a- + 1 /
2
X-a = 3o + fl/
2
(
1 + /
2 5
412 Ch. VIII. Improper Integrals
whence
v
V ~ 9
f + at
2at 2 (3a 2
) 4at dt _
~
) t(\ + t*)(\ + t'*)(\ + t*)*
= 48 ™ 3
J (trw dt + 16 ™ 3
j (ttf¥ dt -
n
= 48jia 3
^ cos 4 zdz — 48na 3
J
cos 6 zdz +
JT/2 JT/2
JT/2
V = 64 jca 3 — • —— 64 ra 3 — .
1x3x5 -
2
8.3.5. Prove that the area of the region bounded by the curve
t/=-7=L=, the axis of abscissas, the axis of ordinates and the
V 1 —* 2
8.3.6. Prove that the area of the region bounded by the curve
y = y^=t » ^e axis of abscissas and the straight lines x=±:l is
finite and equals 6, and the area of the region contained between
the curve # = ~2> the axis of abscissas and the straight lines x=±:l
is infinite.
law). Find the work performed by the force F as the point moves M
along the x-axls from x=r to infinity.
Solution. The work will be negative, since the direction of the
force is opposite to the direction of motion, hence
oo N
A=\ — ^xT dx=
2
lim \ ^rdx
2
= —— r
J n- oo J *
r t
00
2
taneous value of the current intensity / or to its square / . Here t
(b) / =I
e~ kt sin cot (simple oscillating process); coefficients k and a>
are constant.
Solution.
(a) g= \
l e~ k1 dt= lim f V"*' dt = / lin
oo
s J /;*-•*«<«= -'J-
= ;
oo A
(b) g= \ I ()
e~ kt s\n cot dt = lim ^ V"*' sin cot dt =
o
A - 00
n
= -^f7T2
+o)
2 2
!™ [(cocosorf
LV + ftsinwOe-"]^-^^;
or-f-&
7 J o 2 *
/2
^ _ n '
oo ^4
~ co * 2<ot
5- f sin 2
(o/d/= lim I
li e
-* kt l
^
dt =
J A - co J
=—A lim
co
2
I
+6 ;
2
(A cos 2cot + (o& sin 2a)t) 9 -2kt
46 /l - 00
4k (k* + 2
co )
y = ^ e~ a 1x1
(cos ax + sin a x \
|),
oc
4
y"
Pol
= -£-e~ ax [(cos ax
+ sin ax) — 2 (— sin ax+cosax) +
-j-( — sin ax — cos ax)] = —-e~ ax (sin ax— cos ax).
§ 8.4. Additional Problems 415
Hence,
P 2 a 6 Ee
w k2 J
f
e~ 2ax (1 — 2 sin ax cos ocx) dx =
o
P 2 a Q Ee f 1 2a P 2 a*Ee
2a 4a + 4a 2
2
4fc
2
dx
1 xP In* x
i
8.4.3. Prove that the Euler integral of the first kind (beta func-
tion)
i
lim
7
\ sinax-sinP^dx^O,
7 -> a> J
if |a|=HP|.
8.4.5. Prove that
oo
I = ^e- xi
-x* n + 1
dx=Y (n natural).
416 Ch. VI II. Improper Integrals
8.4.6. Prove that if the integral J-^— converges for any posi-
a
tive a and if f(x) tends to A as x—> 0, then the integral
00
r /(ax)-/(N
*
<fa .
(a>0( p>0)
J
g" ax — cosfk
__ f cos
X—
f ^
J *
^x
J x
dx ~ \nn "cT
'
JT/2
——
J cos X
dx con-
verge?
ji
P dx
8.4.9. Prove that the integral
J
j^^k converges if k < 1, and
diverges if k^\.
00
lf(x)dx (1)
a
lf(x)<p(x)dx (2)
necessarily converge?
What can be said about the convergence of integral (2), if integ-
ral (1) converges absolutely?
8.4.12. Prove the validity of the relation
JT
In =^\ncosx-cos2nxdx (n natural)
Chapter I
=—
2
p
1.1.5. (b) Hint. Prove by the rule of contraries, putting 2 , where/? and q
2s
*
^
^ 0; (b) 2^x<:3. //m/. The equality holds true for those
\a — b\ > — 16| holds good when a and 6 are opposite in sign or when
\a\<\b\.
'• 2 - 3
' °=
[aV + ta +
a
W 3
<«•+«>«"-»>
^
1.2.4. + a&+a
&'- + 2
;
(
/t
)
_L 1-2 .6. 4 V'l+l;
4" '
; 2 ^10-5.
g
1.2.11. /(*) = 10 + 5x2*.
,,,,
1.2.16. x=±2; ±3. 1.2.17. / (x) = x — 5x+6. 1.2.18. (*) = 23; (x) = 527. 1
/ <p
1.2.21. (b) (2, 3); (c) (— oo, —1) and (2, oo); (d) x = ^ + 2kn(k = 0, ±1,
±2, ...). Hint. Since sin#^l, the function is defined only when sinx=l;
(g) (-^, 2) and (3, oo); (h) [1, 4); (i) (-2, 0) and (0, 1); (j)
-iL + 2tot<*<
1.2.22. (d) The function is defined over the entire number scale, except the
points x= ±2.
1.2.24. (a) (~oo, oo); (b) (3 — 2jt, 3 — ji) and (3, 4); (c) [—1, 3];
(d) (—1, 0) and (0, oo). 1.2.25. (b) 5<x<6.
1.2.26. (a) 2£ji<a:<(2£+1) ji (* = 0, ±1, ±2, ...); (b) — y, —1 .
*2
1.3.3. (b) Hint. Consider the difference B
^— -
\+x\ \+x\
1.3.4. (b) It increases for — —.-ffcri < x < ^ + kn (k = 9 ±1, ±2, ...)
14
1.3.10. (a) Even; (b) odd; (c) odd; (d) neither even, nor odd; (e) even.
T= —
2ji
; (c) |/1 1 = 5, = —
= arc tan y r = //m/. 3 sin y + 4cos y =
(o ,
cp , 4ji.
a: a:
= 5sin ^y+cp^, where cos q^^, sincp = y. 1.3.13. (b) T = 2n\ (c) T= 1.
1.3.16. The greatest value /(1) = 2. The function reaches the greatest ////if.
value at the point where the quadratic trinomial 2x 2 — 4#+3 reaches the least
value.
1.3.17. (a) Even; (b) even; (c) odd; (d) even.
1.3.18. (a) T = n; (b) r = 6ji.
1.3.19. Hint, (a) Assume the contrary. Then
since the left side is not constant; (b) suppose the contrary. Then
cos \^x+T = cos V x, whence either Vx+ T4- V~lc=2nk, or— 7= = = >Y
1+arcslnj/
1.4.6. (a) x= ; (b)x = 3sin*/; (c) x = y lo 8 5
(y > 0); (d)* =
3
log 2 ^ lQ gj/
1 ° g2 ^ 1
==
.og|
(0 < y < 2 or 2 < y < 00).
1.6.3. (a) — — , ,
0, — ,
(b) —y ,
T>
- y, ,
I -)- <x n
1.7.4. (b) 1; (f) 0. Hint. Multiply and divide by imperfect of a sum, square
± —— 1
1X2
1
J_
2' 2x3
111 2 3 '
'
1
/i(/i+l)~
11
n n -f
1'
1.7.5. (a)
y ;
(b) 1; (c) 0, (d) --i. Hint. The quantity — is an in-
4
finitesimal, and cos n 3 is a bounded quantity; (e) 0; (f)
y .
1.8.6. (b) Hint. The sequence is bounded due to the fact that n\=^\x2x
w-
X3x...X/?^2 and1
therefore
^H4)'+-.+(^n=3-(±)-'<,
1.8.7. (b) 0. Hint. Take advantage of the fact that 2«±!
xn
= n— o
< i.
-f-
1.8.9. //t/i/. For all /z, beginning with a certain value, the inequalities
—
i
< a < n are fulfilled; therefore —n J=- < ,"/ a < {!/ n,
*
and lim iV /i =
iy
i
= lim -
+
1.8.10. Hint. The sequence {y n \ decreases, since y n ±i =a 2
'l 1
= a 2flX2 =
= J^G/« > 0-
The boundedness of the sequence from below follows from a > 1. Denote
lim y n by & and from the relation y n + i= Vy n b=\.
fl -> oo
1.8.11. //m/. Ascertain that the sequence increases. Establish the bounded-
ness from the inequalities
n n(n—\) n — i n v 7
*<'+('-±)+(T-T)+-+(.-±r-7)-»-f
A n skits and /lints to Ch. 1 421
2n
1.8.12. Hint. Transform x n into the form xn = and take advan-
yn + 2
1 -j- n
tage of the inequalities
2n 2n
< < 1.
2i + 1
Y& + + n I
and ascertain that the sequences of appropriate values of the function have
different limits:
i
—=
Xn
x'
n
lim 2 + oo, lim 2 =0.
1.9.3. (e) Hint. Take advantage of the inequality
71
— arc tan x < tan ( — arc tan = (x > 0).
2 V 2
sin x —— = 1
sin x — sin . jx
-g-
— = log a (*-3)(^* + 6 + 3)
Hint, lim log a lim = log a 6;(i)
+ 6 — 3J x—3 *
at-v3L V x L*-3 3
<j)^.
1.10.3. (b) 32. (c) -5-. Hint. Put x=z 15 ; (f) oo. Hint. Put x =z;
1.10.5. (b) e
J
(c) *" (d) (f) 4; (g) (h) 2.
2
1.10.11. (a)
_1_
'
(c)
1
y ; (
-
422 Answers and Hints
2
1.10.13. (a) e 4 ; (b) -1; (c) 2 In a; (d) e 3 ; (e) e ; (f)e-J; (g) 1;
(h) 1; (i) 9; (j) 1; (k) a — p. Hint.
$X
lim
X-*
oiX
X
—= X
lim e?*i
~* Q
rj e (at -fax
-
X
1
L=a— 0.
1.10.14. (a) j/lT. Hint. Replace arc cos (1— x) by arc sin }/~2a;— x 2 (b) l;(c)a. \
|.
m tan a— sin a 1
a- n a3 ~~T'
1.11.6. (b) They are of the same order; (c) they are equivalent.
1.11.8. (a) 100* is an infinitesimal of the same order as x\ (b) x 2 is an
infinitesimal ot an order higher than x\ (c) 6 sin a: is an infinitesimal of the
same order as x\ (d) sin 3 x is an infinitesimal of an order higher than x\
(i) of the second order oi smallness. Hint. Multiply and divide the difference
cos x—
jj/cos x by imperfect trinomial square; (j) of the first order of smallness.
The diagonal d is of the first order of smallness; the area S is of
1.11.10.
the second order of smallness; the volume V is of the third order of smallness.
/(5_0)= —y JI
+ =,
— /(5
(c) 0)
JX
;
at the point a: = 0, a discontinuity of the
JT
first kind: /(— 0)=1, / ~H 0) = 0; ( (d) at the point ^ =y , an infinite dis-
f
if I sin x | < 1,
flx)=UmJs\nx)*»=\ { jf | s in^ |
= 1;
(e) at the points x = kn (k~ 0. ±1, ±2, ...), discontinuities of the 1st kind,
since
- . .
_ |
sin x |
j 1 if sin x > 0,
(
. .
I
— 1 if x=n t
to —4); (d) at the point x—\, an infinite discontinuity of the second kind.
(c) at the points x = + (n = 0, ±1, ±2, ...) there are removable dis-
on the interval [
— 1, 1].
1.17.1. (a) Hint. Multiply the obvious inequalities:
n-
n < '
2
n -4- 1
V2(n-\) <
/(S=T^<=±1;
2n —
rr*
T34 X T56 X...X —>2n
1 1
.f. t i a
b) Hint.
j
Let ^-o-X
2
.
D 2 4 6 2n
'
3 5 7 '
' 2n f- 1
-
r .
424 Answers and Hints
1.17.2. (a) Hint. Extract the 101st root from both sides of the inequality
2
and reduce both sides by 1 1 .
or x >
5
y ; (c) the
(e) [-4, -2] or [2, 4]; (f) *=(2/i+l)y (/i = 0, ±1, ±2, ...).
1.17.17. (a) f (x )
9
7l
y
= (b) a*=
T
± fiX
"
+ T±1?
(see Pro- ; + /
blem 1.17.16).
1.17.18, An even extension defines the function
, , / f(x) =x 2
-{-x for 0< a:<3,
x for —3 < x < 0.
i / * * , ^ , i for rational x,
>=m*)={
.
'
n
; for irrational x.
1.17.23. Hint. II wc denote the period of the function f (x) by T, then from
f(T) = f{0) = f(-T) we get
sin T-\~ cos aT = 1 = sin — 7)-]- cos — aT),
( (
1.17.26. Example:
a: if a: is rational,
-{- x if x is irrational.
^
y for — oo < y < 1,
yn
--
L
+ (-I)»]. lim (x n y n ) = 0,
£ n -* oo
xn = —n yn =n z
lim (x n y n ) = oo.
n -+• oo
that
1
1
xn |
— |
a 1
1 ^ |
xn —a |. The converse is incorrect. Example: = 1)" + 1
.
1.17.38. Hint. The sequence a„ only the following values: may attain
0,1, 9. If this sequence turnedout to be monotonic, then the irrational
number would be represented by a periodic decimal fraction.
1.17.39. Hint. If the sequence ~
On
increases, then
i. e 6„ +l a (
-
< an + l
bj (/= 1, 2, «)>
426 Answers and Hints
and hence
i 1 i )
n
^ x.
2 ix 2 £ ^2
^=1 /z=i fe=i
it follows that
/i+l 1
2/i n nl 2n
1_ 1
qualities a
rt
— 1 < ah — 1 < a
n — 1 take place.
1.17.45. Hint. Divide the numerator and denominator by x m .
1.17.46. (a) a=l; b = —l; (b) a=l; b=-^ . Hint. To find the coefficient
for x > 1.
1,
(b)
for x ?= — +/2JI,
JX
(1 — x)(\+x) (1+x 2
)...(1+a: 2 ") = 1— x* n .
lim
ln(l + *) + ln(l-*) = ]ijn
ln_0- f 2)
=_ 1
X -> X2 X -* X2
— x) —x
~ X =Q. +
and if we replace In ( 1 a:) by x and In (1 by we will get the wrong
X
result: lim
Answers and Hints to Ch. I 427
sideration, then the chord is equal to 2R sin ~ Ra, and the sagitta to
a2
R(\— cosa) ~ R-.
1.17.51. 2. Hint. The difference of the perimeters of a circumscribed and
inscribed regular /z-gons is equal to
on
2tf/z
/.
tan
jt
n
sin
.
—n = 2nR..tana —
jr \
a
sin a n
~ tiRcc .,
2
,
\ J
where a — —,
n
and the side of an inscribed rc-gon is
1.17.55. (a) No. Example: / (x) =x is continuous everywhere, and g(x)~ sin-^-
for Xf:0, g(0) = being discontinuous at the point # = 0. The product of these
^ *
both functions are discontinuous at the
for x < 0;
point x = 0, their product
J
'
=< — I
irrational.
1 if
if
.
is
. We may 3
.,
write
\ 1 a:
f (x) 2\ (x) = 1, where X (x) is — 1.14.4 (b)). the Dirichlet function (see Problem
1.17.57. (a) # is a discontinuity =
of the second kind, \ is a disconti- x—
nuity of the first kind; (b) is a discontinuity of the first kind: f(\ 0) 0, x=\ — =
/ ( 1 H— 0) = 1 ;
(c) cp (x) is discontinuous at all points except x = 0.
1.17.58. (a) x = n = 0, ±1, ±2, ... are discontinuities of the first kind:
lim y=\, lim y=y \
x==n = 0. The function has a period of 1; (b) x= ± n
x-+ n-0 x-> n +
(n—±\ t ±2, ...) are points of discontinuity of the first kind:
lim y = 2n — 1; lim y = y\ X ~_ y— = 2n
x -* Vn-0 x Vn-rO
The function is even; (c) (/i=±1, ±2, ...) are the points of x=±V'n
discontinuity of the first kind; at these points the function passes over from the
value to 1 —
and returns to 1. The function is even;
1
428 Answers and Hints
(d)
x if |
sin x |
< y , i.e. —^-\-nn < x < -^-f-ji/z,
y=] y
-75- if |
1
sin
s a:
I
1
= ~2 ,
•
i.e. x =± 111 ^--f-jx/z,
6~
if
I
sin x I
> — , i.e. —+ J1/1 < x < — |-jw.
x=± — -\-nn JI
are discontinuities of the first kind.
1.17.59. The function f[g{x)\ has discontinuities of the first kind at the
points x =—
1; 0; +1. The function g[f(x)] is continuous everywhere. Hint.
The function f (u) is discontinuous at u 0, and the function g (x) changes sign =
at the points # =
0, ±1. The function g[/(#)|==0, since f (x) attains only the
values 0, ±1.
1.17.61. Hint. Write the function in the form
f(x) = { ° for
2
* = '
x
(*+l)2 tor < x<2.
Make sure that the function increases from — 1 to 1 on the interval f — 2, 0)
3
and from to — on the interval [0, 2]. Apply the intermediate value theorem
to the intervals — — [ 2, 1] and [0, 2]. The function is discontinuous at the point
* = 0:
/(-0)=1, /(+0) = 0.
1.17.62. Hint. Suppose e > is given and the point x £[a, b\ is chosen. We
may consider that
e<min[/(x )-/(a), /(&)-/ (*„)].
min [/ to), . . . , / (x n )] <1 [/ to) + / to) +...+/ to)] < max [/ to), • . , / (x„)].
1.17.65. Hint. Apply the intermediate value theorem to the function g (x) =
= 2X —~ on the interval lj .
is continuous in the intervals (—00, —1) and (1, 00) and has one isolated point
y = 0.
Answers and Hints to Ch. II 429
Chapter II
2.1.1. (b) -—
20
. 2.1.2. (b) 10a; — 2. 2.1.5. v av = 25 m/sec.
2
2.1.6. (a) y' = 3x 2
\ (b) y' =— . 2.1.7. The function is non-differentiable at
2
= — -j
—3
-—
3 =
the indicated points. 2.2.1. (b) y' ax • 2.2.2. (c) y'
l^x '
~
e'»'
a;
= T5r
(cos a:— sin a;)— sin
;
a:—?*
(0 *' = 2e*+y; (g) *' = 2e*cos*; (h) j,'
x2e x
2
2(1— a:)
„ ~ „
2.2.6. (b) */'
/
= -3(3-sinA:) o /o x9
2
cosA:; (c)
/
— 3^==+—5—; 2 cos a: . 2 sin
cos
a:
y
3 sin x sin x 2 *
u'———2e* + 5 In 4
2* In 2 *
(d)
K)
. a;
;
.
(e)
.
,
*/' = 3 cos 3a;
1
=- sin
.
+
5^ ,
3 3/(2e*-2* + l)
a * 5
-j —
2M
l
sec'2 VI; (f) ^' = (2a: — 5)cos(a: — 5A:-H)-Asec — 2
a:
2
a:
; ( h) =
= 1
H 7
h-ri 5;
tanx^\+x* — (i) (/' = 21narc tan-^-3 •
•
9+^
tti
—
x/l + ln** arc
arc tan
3
(x+\)
2.2.13. (a) /' (*) =y (coshy + sinh-0 ; (b) /' (x) = tanhx; (c) /' (x) =
j/cosh *+l; = -Jj— (d) /'(x) ;
(e) /' (a;) = 4 sinh 4a;; (i) f
f
(x) = (a+b)e ax X
X (cosh fo + sinh ^) = (a + e + b)x fr)
(a .
—
4
3a: (1 3a:)
5a:
2 -}-
x — 24
(c) r-
j_ jl
2 3 2
3(x-l) U+2) (*-j-3)
430 Answers and Hints
2.2.17. (a) y
/
=—r.
^81*—
In 3
1
tan ^arc sin3" 2 *
J^arcsin3- 2 *
— ;
...
(b) y
,
=
5x j/cos 4
c /
In 3
— x
. r ,
sin In 3
cos 2 In 3 *)
#-ln x
v
2
o r
— 5/ cos
•
+ COS p*
2* 2
+ 3' 1 +^» are sinx
2.3.6. (a)
(1+a: 2 ) )^1 +a: 2
(b)
'
A
3 +7r \,;
'(1— 2
)
2
(c)2r-"x
2
(1— a:
2
)
X (2x 2 -l).
2.3.8. (a) * 3 sin x— 60 x 2
cos x— 1140 X sin x-j-8640 cos x\ (b) 2e~* x
X (sin x + cos x)\ (c) [3* 2 + 6/™: + 3rt (n— 1) — 4]; (d) (— 1)" [(4/z
2
+ 2/z +
+ 1— a:
2
) cos a: — 4n^sinA:].
lx3x5x...X l97x(399 — x)
2.3.9. (a) .00! [_^__^_] :
b
201
2
2ioo (l—^)
1_ _1_
Hint. y = 2 (1— x) 2
— — x) (1
2
.
~ s -
2.4.1. (b) Xyy
/, "
= —7^-+—sin 4 cos
i
:
a:
rr-
3
(6 a:)
2 4 7
(b)
(b)
y'
x
tr
(e x -ey )V-* x+y )
(c) ifi=^=|;
.
(c)
„~
(d)
_ ^ _
4 sin —
;
(
f
)
- Yt=F •
~ ^l^- fe)
~ 4 ~ ln *
2.6.9. (a) d*i/ = 4-* 21n4(2A: ln4— l)dA:
2 2 2
;
(b) d2 ^ 4 ln *
a:
2
y (In — 4)
2
a:
*
(c) d 3y = —4 s'm2xdx :i
.
oaift (a)d.2
2.6.10. y
M = 4(1+3a: 4 ),
dx 2*'>
.
.
2
—(TZ^r ^ - f="p ^
4(1+3a: 4 ), 2
4a: ,
2
^^4)2 1
4
in particular at # = tan /, d2 y= ~ CQS 2 2/
4
2.6.11. AV = 4nr 2
Ar-\-4nr Ar 2
+—o jxAr 3
is the volume contained between
two spheres of radii r and r Ar; dV 4nr 2 Ar is the volume of a thin layer+ =
with a base area equal to the sphere's surface area 4:xr 2 and a height Ar.
2.6.12. = g/A/-|—i- g At
As the distance covered by a body within the 2
is
time At; ds=gt At = vdt the distance covered by a body which would move at
is
a velocity v = gt during the entire interval of time.
2.7.1. (a) It does not exist; (b) it exists and equals zero.
2.7.2. 90°. Hint. Since
(e*, x^0
y ~ \ e-* x < % 0,
/1(0) = -I, /;(0)=1.
2.7.3. r_(A)=-<p(fl); /;
2.7.4. Hint. For x ^ the derivative
n*) = -cos(l)+2xsin(l
At a: = the derivative equals zero:
a
Aa:- sin
9
— 1
/'(0)= lim
XT-^^ '
Thus, the derivative /' (a:) exists for all x, but has a discontinuity of the
second kind at the point x = 0.
2.7.5. a 2x b = x\. ,
=—
2.7.7. Hint. The formula for the sum of a geo-
metric progression represents an identity with respect to x. Equating the deri-
vatives of both sides of the identity, we get
"'" +1
1 2r 3* + + 1
1
+
; . . . +^- = ^+;>'" +
432 Answers and Hints
//m/. To prove the identity multiply its left side by 2 sin* and apply the
formula 2 sin a sin P cos (a P)= cos(a p). — —
To deduce the desired formula +
differentiate both sides of the identity and equate the derivatives.
2
2.7.9. (a) sin 2x [/'(sin *) 2
/' (cos *)]; (b) <*> [e x —
f (e x ) /'(*)/ (e x )\; +
ln
(C ) ^11?} .
1
<P' (*) . ^ (*)
In 2
"
* ' cp (*)
\|) (#) ln cp (a:) (p (x)
induction.
2.7.18. Hint. Apply the Leibniz formula for the /ith derivative of the pro-
X
I at n = 2k
2.7.19. yM(0)={ [1x3x...X(2/j— l)]
2
at n = 2k+\
I (k=\, 2, ...).
Hint. Differentiate the identity n —2 times and, putting # = 0, obtain
y(n)
(0) = — 2) (/i
2
\fn -2 >
(0) (/i ^ 2)
e-*
2
Hn + 1
(x) = (e-x 2 )<n
+ l
) = (— 2xe-**Y n >
and the Leibniz formula for the nth derivative of the product u~e~ x2 and
V =-2 X yx
. 2.7.22.
=j^jy
2.7.23. x 12 = ±V + Y\- y \ (_oo <{/<l),
*3,4=±Kl-l/7=^ (0<y<l),
(t=l, 2, 3, 4) for Xi ^ 0, ±1.
4x/(l-jtf)
Answers and Hints to Ch. Ill 433
= f 2/ 2 / > [ 2x
y=< „
2
< ^
< ^ '
through x: y ' '
This function is differentiate eve-
|
0, t * j 0, x < 0.
rywhere. 2.7.27. a =c= -!- ; 2.7.28. Hint. The curves intersect at the
4 2
points where sinaA:=l. Since at these points cosa#=0,
y'
2
= f' (x) sin ax-\-f (x)acosax = f (x) = y'v
i.e. the curves are tangent.
2.7.30. Hint. For / y= nn the equations of the tangent and the normal are
reduced to the form:
Chapter III
3.1.2. (b) Yes; (c) No, since the derivative is non-existent at the point 0.
3.1.5. l = e—\. 3.1.7. No, since g(— 3) =g(3). 3.1.9. (d) Hint. Consider
Ihe functions
2x
f(x) = arc sin ^
-j-2 arc tan x for |
a: |
> 1,
2x
g(x) = arc sin y-j— — 2 arc tan x for |
x |
< 1
3.1.17. Hint. Apply the Lagrange formula to the function f (x) = \n x on the
interval [1, l+*]> x > 0, and estimate the right-hand side in the obtained re-
lation ln(l+x) =y (1 < £ < 3.2.1. (c) 2; (d) 0; (f) —1
3.2.3. (b) 0. Hint. Represent cot x -—= — JI!iLf
; ( c) JL 3.2.5. (b) gi = e.
434 Answers and Hints
3.2.6. (a) 1; (b) 1. 3.2.9. (a) -i I (b) lna-1; (c) 2; (d) ; (e) - ;
/ b a
rr.2
n
(n)i-; (o)
^; (p) e~™; (q) 1; (r) -1 . 3.3.5. (b) 0.34201.
(0, oo); (e) the function increases on the intervals ^— oo, y^j
and (3,-j-oo)
and decreases on (^-^ , 3 (f) the function increases over the entire number
scale.
5t
3.5.2. (b) The function increases on the intervals
(
( 0, — JT \
and
(
(
^- , 2jt
J
and decreases on ,
^
3.5.8. (a) increases throughout the number scale;
The function (b) the func-
tion increases on the interval (—1, 0) and decreases on (0, 1); (c) the function
decreases throughout the number scale; (d) the function increases on both
intervals ( —
oo, 0) and (0, oo) where it is defined; (e) the function decreases
on the intervals (0, 1) and (1, e) and increases on (e, -j-oc); (f) the function
(b) the minimum is /(— 2) = — 1, the maximum / (2)= 1; (c) the maximum is
Answers and Hints to Ch. Ill 435
/ 25
—5 =— —
\ f s
= 0, y — —
1
greatest value is /
^
± = ^L_ - , the least value f(±\) — 0.
value is f(— —
) = -£-(-0.25
l
*
'
3
^
greatest value
j
=^ is / if" » the least value / ^-^j=— 2; (e) the great-
4 4Wi
3u~
- — R
r=—
2tl
. 3.8.10. The radius of the cylinder
j base is
2
, where R
is the radius of the cone base. 3.8.11. The equation of the desired straight
. . x y .
T+T —
,
e 15
3.8.14. cp —— TL
. Hint. At the board width a the cross-sectional area of the
u
trough is equal to a 1
(1 +cos cp) sin cp. where cp is the angle of inclination of the
walls to the bottom.
xity , 1^; the points of inflection are , , (1, 13); (c) the in-
3 -V~5 3 + VI
(0, x x ) and (x 2) oo), of convexity (x lt x2) t
where x x =e 2
, x2 =e 2
; the
points of inflection are (x lt y x ), (x 2 , y2) t
where
3.9.5. (a) The point of inflection is (3, 3); the curve is convex for x < 3 and
l/~~5 — i
concave for x > 3; (b) the abscissa of the point of inflection #=arc sin - ;
in f
I
—— n
, arc sin
• V"5-A
- I ,
,
and convex
•
in
{
( arc sin -
• V^—\
—- ,
2~
metrical about the (/-axis and has no asymptotes. The minimum is y(0)-=l t
3 / V"3 23 \
maxima y (1) ==y (— =y 1) . The points of inflection are ( ± —g— ,
(b) the
J;
function is defined in ( —
and ( oo,
1, —
+oo). The graph has a vertical
1) —
asymptote x= — 1 and an inclined asymptote y=x 3. The minimum is y (0)^0, —
maximum y ( — 4) = — 256
The points of inflection are
.
/
6,
3296
and f — ——J
2, —6 \
1
J;
(c) the function is defined in (— oo, 0) and (0, +oo). The graph
tion is
( -- V~* \
(d) the function is defined in the intervals (— oo, —
( ,
)
; 1),
g
(—1, 1) and (1, oo); it is odd. The graph is symmetrical about the origin, has
two vertical asymptotes x— ±1 and an inclined asymptote y x. The minimum =
is y ( V 3) = +3 — — , the maximum y{—V 3)--= 2~ ' point of
inflection is (0, 0); (e) the function is defined everywhere, it is even. The
Answers and Hints to Ch. Ill 437
graph is symmetrical about the (/-axis and has a horizontal asymptote y = 0. The
minimum is y (0) = j[/ 4, the maxima y (± }^ 2) =
2 jj/ 2. The points of inflec-
tion are (±2, \/ A)\ (f) the function is defined in ( — 2, +oo). The vertical
asymptote is x = — 2. The minimum is i/ (0) = 0, the maximum y ( — 0.73)^0.12.
The point of inflection is ( — 0.37; 0.075); (g) the function is defined everywhere.
Ae
The points of inflection are (0, 0),
-v~$ t
(tzVjy vi-sy (*±_o, (ttfiy e e
-z-vi
(h) the function is defined and continuous everywhere. The horizontal asymptote
is y—\. The minimum is i/(0) =
0, the point (0, 0) being a corner point on the
graph: y_ (0) = -i ,
f
y +{ 0) = +~.
3.12.6. 4.4934. x2 3.12.8.
0.202; jc 3 ^=—2.330;
2.128. 3.12.11. 0.6705. = =
3.12.12. (a) 0.27; 2.25; (b) 0.21. 3.12.13. (a) 1.17; (b) 3.07. 3.12.14. 1.325.
3.12.15. 0.5896 and 2.2805. Hint. To approximate the smaller root more precisely
write the equation in the form x e°- 8x =
- 1 to find a more accurate value of the
i
3.13.7. Hint. From the correct equality limcos-r =0 (0 < £ < x), where g is
x ~> o s
determined from the mean value theorem, it does not follow that lim cos— = 0,
x ~> x
since it cannot be asserted that the variable £ attains all intermediate values
in the neighbourhood of zero as x 0. Moreover, g takes on only such a sequ- —
ence of values E for which lim cos — =0 (£££).
3.13.8. Hint. The mistake is that in the Lagrange formula one and the same
point £ taken for / (x) and cp (x).
is
3.13.9. Hint. Apply the Lagrange formula to the function \nx on the interval
[by a]; (b) apply the Lagrange formula to the function zP on the interval [y, x].
3.13.10. Hint. With the aid of the Leibniz formula ascertain that the
coefficients of the Chebyshev-Laguerrc polynomial alternate in sign, the odd
powers of x having negative coefficients. Whence deduce that L n (x) > for
x < 0.
3.13.11. Hint. Using the Rolle theorem, show that inside the interval \x xn
there are at least n roots of the first derivative, n 1 roots of the second deri- — , \
factor sin* (which we cancelled in computing the limit ol the ratio of deriva-
tives) vanishes.
3.13.13. Hint. Write the Taylor formula with the remainder R2 :
6
h2 h'
f (a + h) = f(a) + hf' (a) + f" (a)+-^ /"' (a +6^).
Comparing it with the expansion given in the problem, get the equality
f" (a + Qh) — f" (a)
= }_f,„ a+^ n ( ) and pass over to the limit as h — > 0.
h 6
3.13.14. Hint. Prove by using the rule of contraries. Suppose that e== "~»
where p and q are natural numbers, p > q > 1, and, using the Taylor formula,
get for n> p
i =1+ _L + Jr+ ... + Jr+ _^r (£)«,„ <9< ,,
Multiply both sides of this equality by n\, and noting, that ~ n\ and
1
1
1!
1
. ..4 1
— n\ J
\- \ n\ are positive
r integers
b and —+r-r( —
/i
< —77 — <
n-\-\
•
q
1,
1
\ q J
obtain a contradictory result.
3.13.15. Hint. Verify that the function
sin* n
=}
i
x '
q x< 2 '
is continuous on the interval
f(x)
\ x=--0 1,
( /m 1
> for a <
= l-a<[ <Qfora>1> / 1,
/(0)
the points a: = 2~~» i- e - the derivative changes sign in any vicinity of the
origin.
3.13.19. Hint. Ascertain that the auxiliary function \p (x) / (*) cp (x) in- = —
creases.
3.13.20. Hint. Make sure that at all points of the domain of definition of the
function the derivative retains its sign if ad bc^O. But if ad bc 0, i.e. — — =
—— d
c
then the function is constant. 3.13.21. p — — 6, #=14.
3.13.22. A minimum /(* ) = if cp (x ) >0 and n is even; a maximum
f(x ) if =
(p (* ) < and n is even; the point x is not an extremum if n
is odd. Hint. At an even n, in a certain neighbourhood of the point x the func-
tion retains its sign and is either rigorously greater than zero or rigorously less
than zero, depending on the sign of cp (x ). At an odd n t lie function changes
sign in a certain neighbourhood of the point x .
Answers and Hints to Ch. IV 439
3.13.23. Hint. For x ^ f (x) > 0, hence / (0) is a minimum. For x >
= 2— sin — + — cos — = -^—
the derivative /' (#)
xx x
is positive at the points x
ZiTin
and negative at the points x = ——/ . The case x < is investigated ana-
(2/i+1)ji
logously. 3.13.24. (a) and 0; (b) 1 and —2.
1
3.13.25. (a) The least value is non-existent, the greatest value equals 1; (b)
the function has neither the greatest, nor the least value.
3.13.30. Yes. Hint. Since /" (x) changes sign when passing through the point x ,
Chapter IV
3
2 -2- 2 - -
4.1.7. / = — (*+l) + ^-* 2 2
+C. Hint. Eliminate the irrationality from the
denominator.
4.1.14. / = ~arctan~ + C.
4.1.15. ^
/=-!=arctan-^±J+C.
^3
4.1.18. / = ln|* + Vx* + iyx+_\ + C.
3-
|
V\Qx-V7
4.1.20. I —
= U= In
2 ^70 V 10*+ V~7
+c
x—3
4.1.21. (a) —arc tan C; (b)
T (*-4) y X +C; (c) 3tanx-f
(d) — .1(8
2 + 4a:
2
+ 3a: 4 ) ^T^7 +C. 2
-J
+ C.
4.3.17. xln(x+ + rO — |/"i+x + C.
J^l 4
+ 320 (
2 *+!) 3
+C '
4.4.2. (d) Hint. Apply the generalized formula for integration by parts and
express In from the relation thus obtained
i
n -- —™ sin""
e •
„ i
1 ,
x (a sin x
•
— n cos x)-\ x
n (n—\)
3 — ,
/„-•>
n1
r ' n-
Answers and Hints to Ch. V 441
4.4.3. /„ = -
(nS* 2);
(n—\) sin"- 1
x n—l
cos x I cos a; 1
,
+ C.
, .
,
tan
2 sin 2 x 2 sin 2 x
1
'
2 2
Chapter V
+y ln|*+2| + C.
5.1.5. 2 ln |
x— 1 — In |* + C.
(x-
|
2
I)
9x4-1
5.1.8.
2
t^t- arc tan —>^7
^ I
arc tan (x + 2) + C.
3 V7 3
5.1.10. 5x + ln x + a
|
^ —2 +C |
3
5.1.11.
9* a + 50x + 68 1
ln
(x+\) (* + 2) 16
+ C.
4 (x + 2) (x + 3) 2
(* + 3)H
5.1.13. — 1
Jn
(1+,).
—x +x
1
arc tanx-
6(1+*)^ 6 1
l 1
arc tan — + C.
3 V3 1^3
5.1.14.
x+2
2 arc tan x + In ^+1
2(jc . + l) •
_"V?+T _
5.2.2. 4 £/*+6 y x+24 1
$/' x+24\n \
^ x—\\ + C.
+ C, where
5.2.9. f
\--x V )
1 — arc sin * + C.
5.3.3. -2 arc tan + 1
) +C -
2(/* 2
r l)
442 Answers and Hints
1+l ^-**+2arc
5.3.6. tan
|/ j±^+C.
5.3.7. |- 5.3.8.
(*+/T+?)" +e .
^ ~
2xX
'
L
15
5 Yx* + 2x 1-5— ln(x+l + y X + 2x+5) + C.
'
i
5.4.2.
3* + *--l
a
5.4.5.
o
y 3x 2_ 2x+ + c. \
5.4.9. —
64
(32a- 2 — 20*— 373) Vw + bx+7 -\
128
329 '
/2
In 1
4x+5+-
+ 2 V\x*+ 10a:+14| + C.
5.4.10. +
5 .4. 1 ..-^- 4X
,
+ 3 -2arcsin-L + C.
2
1
5.4.12. — y j-p- +C -
5.4.13. nh + + X + 3^ +
2 1 ^ 4 1
+ C. ////i/. First make the substitution
I
x2 = t.
5
^ |/ (l + ^/^) _3 ^(i + C. 7
5.5.7.
V
a/ - + 77= + c
-
5.5.8. 3 In ,
-
5.5... C+^f-^+C.
5.5.10.
^+^^- -11 +C .
A 8
+ / T4 + C
3
5.5.11.
|V (' )
-
5.6.2. —L
3 sin^ #
_L—
5 sin 5 x
f_c. 5.6.6. tan* + 4-tan**-|-C.
3
5.7.4.
5.7.7.
In (s + V^TT)--
5.7.8. /==
*~ 1
-\-C.
4 ]/> — 2a: +5
5.8.2. / = 4 ^ — x+ 2 In (2 — x— 2 V —a:) — 2 (l +
1 1
V \ —x) In x + C.
cos + sin cc„
.- o , *
\-C where = arc tan x.
/ / , ,
a r^—
5.8.5. I'-'-e^ t t
/
2
-f-l
Chapter VI
4
175
2/i
— i
125
r"T~92
1
4/i
?
*
= 16-4—,175
—- ift
1
2/z
,125
1
—
\-—
4/z
t
2
.
(d ) -J- arc tan -J; (e) In 2; (f) 1; (g) arc tan e~; (h) i; (i)
j|
(J)4- ;
(
k) J^lziO. 6.3.1. (c) 3 < / < 5. Hint. M = /(0)=A m =
= /(2)=A. 6.3.11. (a)^HL^; (b) - V 1-f-x 4
. 6.3.14. (b) ^ . 6 3. 15. (b) ^=
= -e-»smx. 6.3.23. (a) In x; (b)~. 6.3.24. (a) ^ = jjL ; (b) yi=
6.3.25. (a) The maximum is at *=1, the minimum at # =— 1; (b) the
minima are at x = — 2; 0; 2, the maxima at x= ± 1.
^U+ln 2+]^ r
3
6.4.6. (a) ^2 ;
(b) 2(y 3-l); (C ) 8 + ?-^ji.
]A 3 2 1
+ K 2
. n
Sm
24
6.4.J5. (a) 2 — 2 1n2; (b) 0.2 In 112; (c) ; (d)V3— 0.5 In (2 +
sin | sin ^
444 Answers and Hints
+ x— cos x = t)\ —
V^3); (e)0.251n3 (substitution
—
sin (i) a3
^
(substitution x = acos/); (g) (substitution x = 2a sin 2
/); (h) -^-^-j •
tution x 2 =a 2
cos 2 / + 6 2 sin 2 /).
nuous at / = 0.
6.4.18. The substitution / = tan —x will not do, since this function is discon-
tinuous at x = 7i.
[0, 1].
6.4.21. It is possible; see Problem 6.4.12.
a a
6.4.23.
J
/ (arc sin 0^+ J
/ (n — arc sin /) dt+
J
/ (2ji + arc sin /)
o l -l
Hint. Represent the given integral as the sum of three integrals for the in-
= 2
^ / (x) sin /i dx.
6.6.14. Hint. Integrate by parts twice, putting u = (arc cos x) n the first
time and u = ~
(arc cos x) n l the second time.
6.6.15. Hint. Integrate by parts, putting u x. =
6.7.4. 0.601. Hint. Estimate 1V (x) on the interval and put
(a) | / \
yj
2/z = 6; (b) 0.7462. 6.7.5. 0.96
6.8.1.
for 0<x<l,
F(x)--
-i for 1 < *<2,
a
6.8.5. Hint. Putting for definiteness x > and
E (x) = n^ x < /z+1,
take advantage of the additivity of the integral
x \ 2 n x
1 n-l n
6.8.6. The antiderivative F 1 (x) will lead to the correct result and F 2 (x) to
the wrong one, since this function is discontinuous in the interval [0, jc].
X
6.8.8. l=-^\n
2 2b — 2a '
'<H I at, =
1_
2 sin x
6.8.11. Hint. Integrate the inequality
J
V x sin x dx ^ J
x dx
J
sin # dx — 8"
-=
"J/ o
2 VI
6.8.14. //i/i*. Apply the Schwarz-Bunyakovsky inequality in the form
b ~|2 b b
b
a -I a a
F(-x) = ^
f(t)dt = —^f(—z)dz = —F(x) (/ =— z).
since
—X X
all the remaining antiderivatives have the form F {x)-\~C and, therefore, are
also even functions.
6.8.17. Hint. The derivative of the integral / with respect to a equals zero:
~ = /(a + 7W(fl) = 0.
Chapter VII
7.2.2. (a) ~2 i
v
(
b) 4-4-4-
2^2 !n -t-t^ °- 283 -
7 2 5 * '
-
X- 7 2 - 10
- '
^
'
' e-j-1 4 '
'
'
h V& + h*
7.2.13.(a) (i=y; (b) (1=10 2; (c) (1=^ + 2. 7.2.15. y. 7.2.16.
^
7.3.4. ~. 7.3.6. ~+4- arc sin A- 7.3.11. A. 7.3.13.9. 7.3.16. —i-r
6 3 2 1
5 15 m-\-l
Answers and Hints to Ch. VII 447
r n
7.3.19. y. 7.3.20. y. 7.3.21. 2ji— (2 V 3)ln(2+V 3). 7.3.22. 0.75jt.
98 4 R Q
7.3.23. -i-
1
15
. 7.3.24. —o
1
. 7.3.25.
6
. 7.3.26. —
15
. 7.3.27. -i-
lz
1
. 7.3.28. —
oU
.
7.4.6. —5 . 7.4.8. 0.75jtafr. Hint. The curve is symmetrical about the coordinate
axes and intersects them at the points x= ±a, y = ±b.
7.4.9. (a) — . Hint. The curve is symmetrical about the #-axis, intersecting
it twice at the origin at / =± 1. The loop is situated in the second and third
quadrants; (b) — . Hint. The points of self-intersection ot the curve are found
x(t 1 ) = x(t
7.4.10. 0.25nab.
2 ), only if
D
x (t^ = x
coordinates and passes twice through the origin forming two loops. Therefore, it
is sufficient to compute a quarter of the desired area corresponding to the variation
7.4.11. ^-J-. Hint. The curve resembles an astroid extended in the vertical
Sab
direction.
The curve
7.5.2. (a) 5^.; (b) . Hint. is a circle of radius
y passing
through the pole and symmetrical about the polar axis, —^"^(p^y.
XI JX
7.5.11. —5 jt<2
2
. Hint. The curve passes through the pole, it is symmetrical
about the polar axis and situated in the first and fourth quadrants. It is sufficient
to calculate the area of the upper portion of the figure which corresponds to
variation of cp from to — ji
and double the result thus obtained.
(,+_£).
7.5.13.
zia^
— 1
2
7.5.14. Y~2 na Hint. The curve is symmetrical about the axes of coordi-
.
nates and the bisectors of the coordinate angles; it cuts off equal intercepts on
the axes. The origin is an isolated point. It is sufficient to compute the area of
448 Answers and Hints
7.6.10. 2n 2 a 2 b. 7.6.11.
y (see Problem 7.3.9). 7.6.14. 5jx
2
a3 .
3 2 / 11 _ 2£A
^ 3 9
1
tion are: x x1 = —~
3
tt
;
*2 = tt
. 7.6.23.
1Q
—ji.
48
7.6.24. ^
127
7
ji. 7.6.25.-^^2
Ifinr 6
3 105a/?
——
2
c
//t/i/. Represent the evolute of the ellipse parametrically as follows: x cos 3 /
y = — ^-s\n 3
t, where c= y a 2 — b 2 . 7.6.26. ^-na A . 7.6.27. -^L
3
jra
2 In ( 1
-f- |^ 2)
— |-j . Hint. Pass over to polar coordinates,
7.7.10. 10^+^5^. 7.8.2. 8a. 7.8.5. y. Hint. The curve intersects the
axes at t
x
= and =iV 8. 7.8.7. 4 j/~3.
*
2
7.8.8. 16a. 7.8.9. 8na. Hint.
2jt j/"3\ 3a
al
3
7.9.5. 1.5jxa. 7.9.9. In -|- . 7.9,10. 2 ^2 jia.
2
<
b
>
^~T arctan T- 71,n ' (a) (b) £(3 + 4 Y~2)\
3— 1). V2
(0 l(53i
o
+ 6 V 3). 7.11.13.
£
(2 In 7.11.14. (5 J/5-2 V 2).
7.11.20.
nabh
7.11.21. I2ji. 7.U.22. l
/
4 ^- 6
')^fl. 7.11.23. — jta 3 .
^5—1
7.11.24. (a) jx (^5-0) + (^2+l)
+ 21n ^±fL3) (c)2n ,fc
=f\ 7.12.9.^.
7.12.2. 7.12.4.
;
-f^.
7.12.11. nabhd. 7.12.12. nrdh'K 7.12.13. nR * H ' 7.13.3. 0.25:rc/? 3 .
j2
7.13.7. 7.13.8.
3
r
ft
V fl
2
+^ 2
;
M y =-j V'a 2Jt-b^ 7.13.9. /"2 + ln(l+ /"2). 7.13.10.0.15.
„ « rt « ,
7.13.11.
.
/* = f2 aft
3
. a3b
7.13.12.
(a + 3b) /i
3
7.13.16. x c = f/ = 0.4a.
; /jf r
12 12
4/? m—n m— n
7.13.31. x,
n 7.14.1. ; 4
m-\-n
if both m and n are
m-\-n
m—n
even; 2 if both m and /z are odd; if m and n are of different
m-\-n m-\-n
evenness. Hint. The curves y m =x n and y = n x m have two common points (0, 0)
and (1, 1) in the first quadrant. The area of the figure situated in the first
ing in the formula for the volume V= ^ S (x) dx is the same and, consequently,
a
the values of the integrals are also equal.
7.14.5. Hint. The formula follows directly from Simpson's formula
h
2
x2
for a sphere S(x) = n(r — x 2 2
)\ for a cone 5 (x) — nr 2
; for a paraboloid of
^
revolution S (#) = 2jip# and so on.
Hint. Divide the curvilinear trapezoid into strips A# wide and write
7.14.6.
an expression for the element of volume AV 2n xy Ax. =
7.14.8. Hint. Use the formula for calculating the length of a curve represen-
ted parametrically.
tangent corresponds to / — y.
7.14.13. 7.14.14. V2-z. 7.14.16. (a) 0.5 In (x+y);
(b)
JX
- — 0.5 arc sin x.
Chapter VIII
8.1.2. (b)
y ln2; (c) 1; (d) 1 — In 2; (e) ji; (f) 1.
* n
8.1.6. (a) It diverges. Hint. ^* ^ > y for X > Y'e—l; (b) converges;
M^
2 *
(c) diverges. Hint. > _J__ .
converges; (e) diverges.
Vx
8.1.17. (a) 0. Hint. Represent the integral as the sum of two items:
CO 1 cc
n X nX nX
f
^
o o i
CO 1
" X " X
cond summand and show that \ . dx=- — \ .
, dx; (b) ^
1 o
_2_ _
3
8.2.2. (a) 9a ; (b) it diverges; (c) diverges; (d) 6^2; (e) y;
(f) converges for p < 1 and diverges for 1.
(e) diverges; (f) converges. 8.2.11. (a) It diverges; (b) 2 V^ln 2; (c)
y.
8.2.14. (a) It converges; (b) diverges; (c) diverges; (d) converges;
-1
(e) converges. 8.3.7. (a) y ;
(a) 2ji. 8.3.8. 3jw 2 . 8.3.9. . 8.3.10.
^p.
8.3.14. mgR. Hint. The law of attraction of a body by the Earth is deter-
= niP R 2
mined by the formula f -^y— 2
, where m is the mass of the body, r is the
distance between the body and the centre of the Earth, R is the radius of the
Earth.
8.3.15. e v Hint. Electric charges interact with a force -jf- , where ex and e2
are the magnitudes of the charges and r is the distance between them.
8.4.1. Hint. Represent the integral in the form of the sum
+ oo a +co
dx _C dx C dx
f
J
1
xp \n9 x~ j
1
xp \n^x J
a
x? M x
{a > l)
Answers and Hints to Ch. VIII 451
and apply special tests for convergence, taking into consideration that in the
integral In a:= ln[l
first + —
and in the second integral 1)] — a: — 1 as x — 1,
the logarithmic function increases slower for q < than any power function.
8.4.2. Hint. Making the substitution xQ=t, reduce the given integral to the
form ± —y
I
\ t
£+l
q
~ x
smtdt. Represent the integral
y\ t
p±i
q
~ l
sin / dt as the sum
o o
1 + GO
ges absolutely for 1 <a<2 and conditionally for 0<a^l. Note that at
-f CC +X
^jl-dt, and at —— 1 to the diverging integral ~^-dt.
J J
o o
1/2
-}- xP~ l
(1 — x)*!- 1
dx and apply the special comparison test.
1/2
T
8.4.4. Hint. If |
a |
7= | p |, then sin ax- sin fix dx is bounded.
ji 2
P dx C dx
8.4.9. Hint. Represent \ —as the sum of two integrals
5 \
k -f-
J (sin x)* J (sin x)
n
P dx
+ \ ; reduce the second integral to the first one by making the substi-
J (sin*)*
ji
X C0S
4 ^ ^'^s
^ dx '
The inte & rand of tne first summand on the right side is
y
an infinitely large quantity of order s 3 as x >0. By the special comparison — —
test the first integral converges absolutely for s 3 < 1, i.e. s < 4, and diverges —
for s^4. The second integral in the right side converges absolutely for s > 1,
since the function sin#(l cos a:) is bounded. But if < s —
the second in- < 1 ,
^s\nx-josx
integrals *-^dx and dx (§ee Pro5 em i 8>I j 3 ).
^
TT JT
2 T
8.4.11. Hint. Integral (2) can diverge. For example, let
( 2/ux<A:<(2/z4-i)ji,
YV(x) —
1,
cp <
1—1, (2/i H- 1)ji < x < (2/z4-2) jt.
CD CO
^^-^^(x)dx =
S
The integral -^^- dx converges (see Problem 8.1.13). But
J
o o
00 CO
S *" X
= ^ I
verges absolutely, then the integral f (x) cp (#) dx also converges absolutely: if
J
a
|
cp (#) | < C, then | / (a:) cp (a:) |
< C \ f (x) |, and it remains to use the comparison
theorem.
ji
T" x
8.4.12 //m*. Transform the integral f (x) into f (x)= ^ In sin z dz by the
JT
substitution
Jl
— z. Taking into account that sin z — 2 sin —
Z
• cos —Z
,
reduce
T
p sin
=— x
1
the equality
J /,, \ sin 2/?a
cos*
dx, n ^ 0. Since
' 2/2 J
sin 2/2* = sin (2n — 2) x-cos 2a, 4- sin 2a -cos (2/z — 2) a:,
Answers and Hints to Ch. VIII 453
r n_
2
ln
~2n
-
f /0
\ sin (2n
.
— nx2) ^—
sin a: , ,
cos #
L
n
2 2
J
Check by direct calculation that for n ^2 the second and the third summands
equal zero. Therefore, for n^2
\ sin (2/z
v
— 2) 7
A'
cos x
dx =
n
/„_,.
n 1
2/i J
n n
Since /,---—
2
\ sin 2jt
cos
4* = ^ we have
4
/2 == — —2 4 *
2
/ 3 =.-.-.
1
2
'
T~^3^4
J a: <j